You are on page 1of 114

Ma

the
mat
ic
s121/
1,2

KCSEMAT
HSPREDI
CTI
ONSF
ROMVARI
OUSCOUNT
IES
Ke
nyaCe
rti
f
ica
teo
fSe
cond
aryEd
uca
ti
on(
K.C.
S.E.
)

121/
1/2- MAT
HEMAT
ICS -Pa
per1/
2
COUNT
YANDSUBCOUNT
YPREDI
CTI
ONS

KIRI
NYAGA
121/1
MATHEMATI
CS
MOCKSKCSEPREDI
CTI
ONS
Pape
r1
2½h ou
rs
SECTIONI(50MARKS)
AnswerALLque
sti
onsi
nthi
sse
cti
on
1. Witho
u tu
sin
gacalc
ula
toro
rmath
emat
ic
alt
abl
e,e
val
uat
e

(
3ma
rks
)

2. F
indt
hep
oss
ibl
eva
lue
sofxi
nth
eeq
uat
io
n.

(
3ma r
ks)
3. Pai
ntxco
stKs
h50p erli
tr
ewh i
l
epain
tycostKs
h70pe
rli
tr
e.I
nwhatpr
opor
tio
nmustxbemix
edwit
hyt
oprod
uceamixt
ure
co
sti
ngKsh58perl
it
re? (3mark
s)
4. Cal
cul
at
ethear
eaoft
h eshad
edreg
iongi
ventha
tACi
sanar
cofac
ir
c l
ecen
treB.Gi
vent
hat AB=BC=14cm,CD=8c mand
an
gleABD=750 (
3ma r
ks)
A

5. Ahal
fth
ei n
ter
iora
ngle
so fanns
ide
dirr
egul
arpo
lyg
onmeasu
res1350ea
ch.Th
ere
mai
ni
ngh
alfme
asu
res1650e
ach
.Fi
nd
6. Us
esquareroo
ts,r
eci
pro
ca l
ands
quar
etabl
estoe
val
uat
eto4si
gnif
ic
antf
igu
re.

(
4ma
rks
)

7. Ast
rai
gh tl
i
n ewhoseequat
ionis4y+3x=12cut
sthexandyaxi
satPa
ndQ.
Cal
cul
atetheco-or
dina
tesofPa n
dQ. (
3ma
rks
)
8. Conv
ert154.500int
oradia
na ndwri
tey
oura
nswe
rinter
msofC. (
2ma
rks
)
0
9. Int
hefigur
eb el
owNP=11. 6c
m, <PMN=70 and<PNQ=450. (
3ma
rks
)

F
ORMARKI
NGSCHEMESI
NBOX0724351706 Pa
ge|1
Ma
the
mat
ic
s121/
1,2

10.Wr
it
edo
wnt
hei
neq
ual
i
ti
est
hatd
efi
net
heu
nsh
ade
dre
gio
nma
rke
dRi
nth
efi
gur
ebe
low.
(3ma
rks
)

11.Thele
ngthofar o
omi s4ml ongert
hanit
’swidt
h.Fi
ndthel
engthoft
heroomifi
t’
sare s32m2.
ai (
4mar
ks)
12.AKenyantouri
stle
ftGermanyforKenyathr
oughSwit
zer
lan
d.Wh i
lei
nSwitz
erl
and,hebo
ughtawat
chwort
h52Deut
schemar
ks.T
he
exc
hangeratewasasfol
lows:
1SwissFrance=1.28De ut
scheMa rk
1SwissFrance=45.21Ke nyaShil
l
ings
Fi
ndthev a
lueofthewatchinKenyaShil
l
ings. ` (
3marks
)
13.Twosimil
arsoli
dhavesurfa
cea r
eaof48cm2and108cm2resp
ecti
vel
y.F
indt
hevolumeofth
esmal
le
rsol
i
difth
ebi
ggeroneh
asa
vol
umeo f162cm3. (
3mar
ks)
14.Giv
ethata=2i +3j –5kandb=i –5j+7k.Evalu
ate|2a+b| (
3marks
)
~~ ~ ~~ ~ ~ ~ ~
15.Atr
iang
lewit
hvert
i
cesA(1,
1)andB(1,5)an
dC( 4,
5)i
srot
at
e dhal
ftu
rnaboutt
heori
gi
n.Det
ermi
netheco-
ord
ina
teso
fit
simage
.
(4mark
s)
16.Mutu
neb o
ughtt
hreepe
nsandf
ou re
xer
cis
ebooksfo
ratot
alofKsh155whil
eBingobou
ghtt
wos i
mil
arpe
nsandfi
vesi
mil
arex
erc
ise
bo
oksforato
tal
ofKsh150.De
termi
nethecos
tofapenan
dth ato
fanexe
rci
sebook. (3mar
ks)

SECTI
ONII(50MARKS)
Answe
ron
lyfi
vequ
est
ion
sfr
omt
hi
sse
cti
oni
nth
esp
ace
spr
ovi
ded
.

17.a
)Fi
ndt
hee
qua
ti
ono
fas
tr
ai
ghtl
i
nep
ass
ingt
hro
ught
hep
oin
ts(
3,2)a
nd(
¬-3,
6)g
ivi
ngy
oura
nswe
rint
hef
orm

whenaa n
dba reconstant
s. (4mar
ks)
b) St
atetheco-
ordi
nat
e sofAa ndBatwh ichthel
inei
n( a)ab
o vecro
ssth
ex–axisa
ndy–a xi
sres
pec
ti
vel
y. ( 2ma
rks
)
c) Usi
ngtheinf
ormati
oni n(a)and(
b)ab ovefi
ndthear
e aoft
riangl
eAOB,whe
reOistheori
gi
n. (2ma
rks
)
d)F i
ndtheacut
ea ng
lethel i
nein(a
)abo vemakeswit
ht heXa x
is. (
2ma r
ks)
18.Th
et a
blebel
ows howsma rksobt
ainedb y100candi
datesatEastsi
dehi
ghs
chooli
nbio
logyexa
minat
io
n.
Marks 15-24 25-34 35-44 45-54 55-
64 65-
74 75-
84 85-
94
Fre
quency 6 14 24 14 x 10 6 4

a
) Deter
min
eth
eval
ueofx
. (
2ma
rks
)
F
ORMARKI
NGSCHEMESINBOX0724351706 Pa
ge|2
Ma
the
mat
ic
s121/
1,2
b) St
atet
hemo dalcl
ass (
1ma r
k)
c) Cal
cul
atethemedianmark (
3ma r
ks)
d) Cal
cul
atethemeanma r
k (
4ma r
ks)
19.Awate
rtan
ki sre
prese
ntedbyt
hesol
i
ds h
ownb
elo
wwh
ichi
sah
emi
sph
ereo
nac
yli
nde
r.T
heb
aser
adi
uso
fth
ecy
li
nde
ris7mand
22
t
hehei
ghtofthetan
kis15m. (
use= )
7

a
) Ca
lcu
lat
eth
evol
umeo fthehemis
pher
ic
alpar
t. (
2ma
rks
)
b
) Ca
lcu
lat
eth
evol
umeo fthecyl
in
dri
calp
art
. (
2ma
rks
)
c
) Ca
lcu
lat
eth
ecapa
cit
yo fthet
ank. (
4ma
rks
)
d
) Ca
lcu
lat
eth
eto
tals
urf
a ceare
aexcl
udi
ngthebas
e. (
2ma
rks
)

20.a
)Gi
vent
hema
tri
x f
i
ndt
hei
nve
rseo
fA. (
2ma
rks
)

b)T wou ni
versiti
es,TECKa ndKCTp urchasedbeansandr i
ce.TECKb ought90b agso fbeansand120b agsofri
ceforatot
alofsh360,
000. KCTb o ught200b agso fbeansa nd300b agsof r
icefor at ot
alofsh850,000. Us ethema t
rixme t
hodtofi
ndthep r
ic
eo fone
bago feachi tem. (
5ma rks
c)T hep ri
ceo fb eanslaterdecreasedint herati
o4:5wh il
ethep r
iceofri
cei nc
reasedb y20%.
Ab usinessma nb ough t20b agso fbeansand30b agso fri
ce.Howmu chd i
dh epay? (3ma r
ks)
21.Theb ound ari
esPQ, QR, RSa ndSPo farancha r
es t
rai
ghtsu c
hth a
t:Qis16k mo nb earingof0400f r
omP; Risdire
ctl
ysouthofQ
ande astofPa ndSi s12k mo nab earing1200fromR.
a) Usingas cale1c mr epresent2km, showt heaboveinfor
ma ti
oninas cal
ed ra
wing . (3ma r
ks)
b)F romt hes caledrawin gdetermine
(i
)t hed i
sta nceink i
lome t
reso fPfromS. (2ma r
ks)
(i
i)T h ebea ri
ngo fPf romS. (2ma r
ks)
c) Calculat
et hea reaofth eranchPQRSi nsq uar
ekil
ome tr
es. (3ma r
ks)
22.An umb ero fnu r
seswo rk
inga tKiberahealt
hc e
ntred e
cidedtorai
seSh .144,000t obu yaplotofland.Eachnurs
ewa stocont
rib
ute
thesamea mo unt.Befo r
ethec ontri
bu t
ionwe r
ecolle
c t
edfiveofthenursesret
ire
d .Thisme a
ntthattheremaini
ngn ur
seshadtopay
mo r
et ome e tthetarge t
.
a)I ft
h erewe r
enn urs
eso ri
ginall
y,findthee xp
ress
iono ft
h eincr
easeincon t
ri
buti
o npern urs
e. (3ma r
ks)
b)I ft
h eincreaseint hec ontr
ibutionpern urs
ewa sSh .2,
400, fi
ndthen u
mb erofnu rse
so ri
ginal
lyattheh e
alt
hcentre
.
(3ma r
ks)
c) Ho wmu chwo ul
de achn urseh avecontri
butedift
he5n urs
esh a
dn otret
ired? (2ma r
ks)
d)F indthep ercentagein crea
sep ern u
rseb ecaus
eo fth
er et
irement
. (2ma r
ks)
2
23.a)Us ethetra pezi
umr u l
etoe sti
ma t
et heareaunderthecu r
vey=x +x¬–6o vertheinterv
al
0<x<8u si
n g8t rape
zia. (4ma r
ks)
b)Findt hee xactareau nderthec ur
vein( a)abov
e. (4ma r
ks)
c)Findt h
ep ercenta
g ee r
rorinthee sti
matedareain(a)above. (2ma r
ks)
24.Inthef i
gureb elow, DAi sad iameterof t
hec i
rc
leABCDEc entr
eO. TCSi satangenttot h
ec i
rcl
ea tC,AB=BCa ndangleDAC=380.

F
ORMARKI
NGSCHEMESI
NBOX0724351706 Pa
ge|3
Ma
the
mat
ic
s121/
1,2

O
D
B

T C S
Gi
vi
ngrea
son
s,d
ete
rmi
net
hef
ol
lo
win
gan
gle
s.
a) <DCT (
2ma
rks
)
b) <DEA (
2ma
rks
)
c) <ACB (
2ma
rks
)
d) <BDC (
2ma
rks
)
e) <BOA (
2ma
rks
)

KIRI
NYAGASCHOOLBASEDF ORM4EXAMINATI
ON.
121/2
MAT HEMATICS
MOCKSKCSEPREDI CTI
ONS/.
Paper2
2½h our
s
SECTIONI(50MARKS)
AnswerALLques
ti
onsint
hiss
ect
io
nint
hesp
acesp
rov
ide
d.

1. Us
elo
gar
it
hmst
o4d
eci
mal
pla
cest
oev
alu
ate
. (
4ma
rks
)

2. At
rai
nmo
vin
gat20m/
sta
ke15s
eco
ndst
oco
mpl
et
el
ycr
ossab
rid
gewh
ichi
s30ml
ong
.

Whati
sthele
ngthoftr
ai
ninm. (
3ma r
ks)
3. Th
ewidthofaroomis8mles
sth
anlengt
h.F
indthemeas
ureme
ntofth
eroo
mi ft
hear
eais48m2. (3ma
rks
)
4. Fi
ndth
ep er
cent
ageerr
ori
nthet
ota
lle
ngthoff
ourrod
sme a
sur
ing12.
5cm,24.
5cm,
12.9cmand10.
1cmal
lt
hene
are
st0.
1cm.
(
3mar
ks)
5. Si
mpl
i
fyt
hef
ol
lo
win
gsu
rdsl
eav
ingy
oura
nswe
rint
hef
orm

(
3ma
rks
)

6. F
indt
hev
alu
eofxg
ive
nth
at

(
3ma
rks
)

7. Th
equa
nti
ti
esP,
Qan
dRa
res
ucht
hatPv
ari
esd
ire
ctl
yast
hes
qua
reo
fQi
nve
rse
lya
sth
esq
uar
ero
oto
fR.
Whe
nP=24,
Q=2a
nd
R=9.
F
ORMARKI
NGSCHEMESI
NBOX0724351706 Pa
ge|4
Ma
the
mat
ic
s121/
1,2
Det
ermi
ne
i
)thel
in
ec o
nnect
ingP,QandR (
3mar
ks)
i
i)t
heval
ueofPwh enQ=5a n
dR=36. (
1mk
)
8. Makeythesub
jec
tofth
efo
rmu l
a. (3ma
rks
)

9. T
hefi
gur
ebel
ows
howsac
ir
cl
ewi
thc
ent
rePa
ndr
adi
us c
m.I
fth
ele
ngt
hoft
hec
hor
dSTi
s8cm,
showth
att
hes
had
eda
reai
s
(
8-16)
cm2 (
3mark
s)

4 5
10.Ex
pan
d u
ptot
het
ermi
nx .He
ncee
val
uat
e(0.
95)gi
vi
ngy
oura
nswe
rco
rre
ctt
o4S.
F (
3ma
rks
)
11.Marya
ndJan
ework
ingt
oget
herc
ancu
lt
iv
ateapi
eceofl
andi
n6day
s.Ma
ryalo
neca
ncompl
etet
hewor
kin15da
ys.Af
te
rth
etwo
ha
dwo r
kedf
or4d
aysMarywi
thd
rewt
heser
vi
ces
.Fin
dtheti
met
ake
nbyJan
etoco
mp l
et
eth
eremai
ni
ngwork
.
(
3mark
s)
12.De
ter
min
eth
eeq
uat
io
noft
het
ang
entt
oth
ecu
rve a
tth
epo
intx=2. ` (
3ma
rks
)

13.Thefi
ft
htermofanari
th
me ti
cpro
g r
ess
ionis11a ndthetwent
yfi
ft
htermi
s51.
Calc
ulat
ethef
irs
tte
rma ndtheco
mmo nd i
ffe
renceof t
hepro
gres
sio
n. (
3mar
ks)
14.Thecoordi
nat
esoft
heendp oi
nt
so ft
hediamete
ro facirc
lear
eA(3,-
2)an
dB(5.1).
2 2
Fin
dt h
ee q
uat
ionofaci
rclei
nthefor
ma x +by +c x+d y+e=0 (3mark
s)
15.Inthefi
gur
ebelowQTisat a
ngentt
othecirc
leatQ. PXRTandQXSa r
est
rai
ghtl
i
nes.PX=6c
m,RT=8c
m,QX=4.
8cmandXS=
5cm. (
4mar
ks)

F
indt
hel
engt
hofQT (
3marks
)
00<x<2cf 2
16.F
indi
nra
dia
nsint
heva
lue
sofxi
nth
ein
ter
val orwh
ich2c
os x–s
inx=1(
le
avi
ngy
oura
nswe
rint
ermsof)
(
4marks
)

SECTI
ONII(50MARKS)
Answe
ron
lyfi
vequ
est
ion
sfr
omt
hi
sse
cti
oni
nth
esp
acep
rov
ide
d.

17.Ke
n,T
oma
ndPe
terc
omp
etet
ohi
tth
ebu
ll
’se
yei
nag
ameo
fda
rt.
Thep
rob
abi
l
it
ie
sofe
achh
it
ti
ngt
heb
ull

sey
ear
eKe
n ,
Tom

a
ndPe
ter r
esp
ect
iv
ely
.

a
) Useth
et r
eediag
ramtoshowthep
oss
ibl
eou
tco
mes
. (
2ma
rks
)
b
) Cal
cul
atetheprob
abil
it
yth
at:
-
i
) All
thr
eeh i
tthet
arget
. (
2ma
rks
)
i
i
) Onlyonehit
sthetar
get
. (
3ma
rks
)
i
i
i)Twohitth
etarget
. (
3ma
rks
)

F
ORMARKI
NGSCHEMESI
NBOX0724351706 Pa
ge|5
Ma
the
mat
ic
s121/
1,2
18.Thef
i
gur
ebe
lowr
epr
ese
ntsari
ghtp
yra
midwi
thv
ert
exVa
ndar
ect
angu
larb
asePQRS.
VS=VP=VQ=VR=18c m.PQ=16cmandQR=12cm.
MandOaret
hemidpoin
tsofQRa
ndPRr
esp
ect
iv
ely
.
V

S R

M
O 12c
m
P
16c
m Q

Det
ermine:
a)T h
el e
ngthoftheproj
e c
tionoflineVPo nthepl
anePQRS. (
3ma rk
s)
b)T h
es i
zeoftheangl
eb etweenlin
eVPa n dt
heplanePQRSto2d .
p. (
3marks
)
c)T h
es i
zeoftheangl
eb etweenthep l
anesVQRa ndPQRS (
4marks
)
19.Atthebegi
nningoftheyear2004, Kamaub oug
httwoh o
uses,on
einThik
aa ndtheotheri
nNai
rob
i,ea
chatKs
hs3,
750,000.
The
val
ueofthehouseinThikaap p
reci
atedattherat
eof12%p .
a.
a) Ca l
cula
tetheval
ueoft hehouseinThikaaf
ter9year
stothenear
ests
h i
l
li
n g
s . (
2ma rk
s)
b) Afternyears
,theval
u eoftheh o
useinT hi
kawasKsh8,290,
055wh i
letheval
ueo ft
heho
useinNai
rob
iwasKs
h
8,
822,271.
i)F i
ndn (
4ma rk
s)
i
i)F i
ndtheannualrat
eo fappre
ciat
ionofthehouseinNai
robi
. (
4ma rk
s)
20.a)Complet
ethetabl
eb el
owf orthefunct
iony=2s inxandy=3s i x+300)f
n( orthe
-1800<x<1800t o2d .
p. (
2mk s
)

x-1800 -
150 -
120 -
90 -
60 -
30 0 30 60 90 120 150 180
y=2sinx -
1.73 0.
00 2.
00 0.
00
0.
00
y=3sinx+300)
( -
2.60 0.
00 1.
50

b)o nthes amea xi


sd r
awt hegrap
hso fy=2s inxan dy=3s n(x+300)f
i or-1800<x<1800. (4mark
s)
c)F romtheg raphdeterminetheperi
oda nda mpli
tudeo ft
hegraphy=3s i
n(x+300). (
2ma r
ks)
d) Usey ourgraphtoso l
vetheequati
on2s inx=3s in(x+300) (2ma
rks
)
0 0
21.Apla
neSf li
esfromap ointP(40N, 45W)t oap o
intQ( 350N,450W)andthe
no ntoapoi
nt T(350N,
1350E)
.
a) Giventhattheradiusoftheear
this6370k mf i
ndt hedist
ancefro
mPt oQinKm. (
2ma r
ks)
b)F indinn m
i)t heshorte
s tdi
sta
n c
eb et
we e
nQa ndT . (2ma
rks
)
i
i)t helongestdis
tancebetweenQa ndT( tothen e
aresttens) (2ma
rks
)
c)F indthediffe
renceinti
met ake
nwh enSfliesal
ongt heshort
estandl
onges
troute
sifi
tss
pee
dis420knot
s
(4ma
rks
)
22.Thet
ablebelows howsthed i
str
ibut
ionof marksscoredby60s tudent
sinate
st.

Mark
s 11-
20 21-
30 31-
40 41-
50 51-
60 61-
70 71-
80 81-
90
Fr
equen
cy 2 5 6 10 14 11 9 3

a
) Ont hegridprov
idedd rawac umula
ti
vefr
equen
cycurveoft
hedat
a. (
3mar
ks)
(
Take1cmt orepr
e s
e nt5s t
udent
sonthever
ti
calsc
alean
d1c mtore
pre
sent10mar
ksonth
ehori
zon
tal
scal
e)
.
b
)F r
omt hecurvei
n( a)ab o
ve
i
) e s
ti
ma t
etheme di
anma rk (
1mar
k)
i
i
) De t
erminethequart
iled ev
iat
io
n . (
2mar
ks)
i
i
i) Det
erminethe10thand90thp e
rcent
il
erange
. (
2mar
ks)
c
) I ti
sgiventhats
tude ntswhoscoreove
r45ma rk
spassth
etest
.Usey
ourgr
aphin(
a)ab
ovetoe
sti
matet
heper
cen
tag
eof
st
udent
sth a
tpass. (
2marks
)

23.T
hed
iag
rams
howsat
ri
ang
leOPQi
nwh
ichMa
ndNa
rep
oin
tso
nOQa
ndPQr
esp
ect
iv
elys
ucht
hatOM= OQa
ndP
N= PQ.

F
ORMARKI
NGSCHEMESI
NBOX0724351706 Pa
ge|6
Ma
the
mat
ic
s121/
1,2
L
ine
sPMa
ndONme
eta
tX.
P

⃗ ⃗
a) Give
nt hatOP=pa n dOQ=q .
Ex p
res
sinter
mso fPandqt
~ ~ heve
c
~to
rs.

) PQ
i (
1ma
rk)

i
i) PM (1ma
rk)

i
ii) ON (
2ma
rks
)
⃗ ⃗ ⃗
b) Yo ua r
efurtherg i
venthatOX=k ONandPX=hPM

i
) Ex presOXi
s ntwod iff
erenc
ewa ys. (
2ma
rks
)
i
i)F in
dt hevalueofha n dk. (
3ma
rks
)
i
ii)F in
dt herat
ioPX:XM (
1ma
rk)
24.Ap a
rtic
lemo vesalongas t
rai
ghtli
nesucht
hatit
sdis
plac
eme
ntSmetr
esf
romag
ive
npo
inti
s
3 2
S=t –6t +9t+3wh eretistheti
meinsecon
ds.Fi
nd:-
a)Thed i
spl
aceme nto ft
h epart
ic
leatt=3s. (
2ma
rks
)
b)Thev e
locit
yo fthep a
rtic
lewh er
et=4s (
3ma
rks
)
c)Thevalueoftwh ereth epart
icl
eismo me
ntar
il
yatres
t. (
3ma
rks
)
d)Thea c
c e
ler
ationo ftheparti
cl
ewh ent=4s. (
2ma
rks
)

F
ORMARKI
NGSCHEMESI
NBOX0724351706 Pa
ge|7
Ma
the
mat
ic
s121/
1,2
CEKENAS

121/
1
FORMFOUR
MATHEMATI
CSPAPER1
.MOCKSKCSEPREDI
CTI
ONS/.
.
TME:21/
I 2HOURS

SECTION1
1 3 1
1. Ev
alua
te ( /4 -/ 8)÷3/ 2+ 15/6
3 2 4 -
5
/7of3/ 3-3/ 7-( /4) (3mks
)
2. T
hreeang
leso
fap ol
ygona r
e125, 140a nd160.
Ther
ema
ini
nga
ngl
esa
re1450e
ach
.Ca
lcu
lat
eth
esumoft
hei
nt
eri
ora
ngl
eso
fth
e
p
oly
g o
n (
3mks
)

3. Ad e
ale
rhast
wotyp
eso
fgr
adesoft
eaAan
dB. Gr
adeAc
osts
Ksh.140pe
rkg.g
radeBco
stsKsh
.160pe
rkg
.Ift
hemi
xt
urei
sso
ld
atKsh.
180perk
gandt
hede
alermak
esapr
ofi
tof
20%.F
indther
ati
oinwhi
chthet
wogra
d e
sshou
ldb
emix
ed
(3mk s
)

4. Ev
alu
ate
1 2/
3 16 -1/
2
( /125) x(/ 25)
-
3/5
32 (
4mk
s)
5. L
ista
llt
hei
nterg r
alva l
ueso f Xwh i
chs
ati
sf
yth
ein
equ
ali
ty

4+x ˃3x+2˃-13 (
3mks
)
-
3
6. T
heequa
ti
o nofl
i
neL1,
is10y=4x–20.Li
neL1i
nte
rse
ctsa
not
herl
i
neL
2pe
rpe
ndi
cul
ar
lya
tth
epo
int
P(10,
2).
Findt
he
c
oor
din
atesofpo
intMwhereL
2int
ers
ect
swit
hli
nex=12 (
3mk s
)

7. Us
eta
ble
sofs
qua
rer
oot
san
dre
cip
roc
alst
oev
alu
ate (
3mk
s)

10 + 4
0.625 164

8. I
nth
etr
ia
ngl
ePQRb
elo
wPQ=13c
m,PR=15c
man PQR=750
d ˂

P
m

15c
m
13c

750 750
Q R
Cal
cul
ateco
rre
ctto3s
ign
if
ica
ntfi
gur
esth
eareaoft
hetr
ia
ngl
ePQR (3mks
)
9. Asect
orofaci
rcl
eofr
adi
us42cmsubt
endsa
na n
gleo1200a
f tt
hec
ent
reo
fth
eci
rc
le.
Thes
ect
ori
sfo
lde
dint
oanin
ver
tedr
igh
t
co
ne.

Cal
cul
at
e:
-
(i
)T heba
ser
adi
uso
fth
eco
ne (
3mk
s)

(
ii)T oonedecimalpl
acethever
ti
calhei
ghto ft
hecone (
1mk)
2
10. Si
mp l
if
ytheexpres
sion: x–3 -x -3x
2
x+3 x -9 (
3mk s)
11. Mu r
imibought50p h
ysic
sterxtb
ooksand60ma t
hemati
c st
extb
ooksf
oratota
lo fKs
h.85,
500.
Ha dMuri
mibou
ghtha
lfnumbe
r
ofphysi
cste
xtbo o
ksando neandahalft
imesthenumbe rofmat
hemat
ic
stextbookshewoul
dhavepa
idKsh.
5250more.
(
i) Form2e quati
onstorepr
ese
ntthea bov
einf
orma ti
on (
1mk)
(
ii) Usema tr
ixmethodtofi
ndthepri
ceo faphys
icsb oo
kandamathe
ma t
icstex
tbook
s (
3mks)

F
ORMARKI
NGSCHEMESI
NBOX0724351706 Pa
ge|8
Ma
the
mat
ic
s121/
1,2
12. Acommerci
alba
nkbu y
sandsell
sfore
igncur
ren
ciesatt
her
ateshownbel
ow:
Buyi
ng Sell
i
n g
100Japa
neseye
n 84.0 84.2
1Usdoll
ar 88.
6 88.
7
Atour
is
twentt
oKe n
yawith500,000yen.Hechange
dthemoneytoKsh.
Inaban
kwhic
hch
argedacommis
si
onof5%.Hesp
ent
¾o ft
hemoneyandconve
rte
dt h
er e
sttoUsdol
la
rswh e
nle
avingthec
ount
ry.
Howmuchmone
ytoth
en e
ares
tdo
ll
arsd
idhego
bac
kwith? (3mks
)

13.XYZi
sat
ri
ang
le.
Dra
wth
elo
cuso
fap
oin
tMs
ucht
hatXYZi
seq
ual
toa
ngl
eXMZa
ndYmu
stl
i
eont
hel
ocu
so fM
(3mks)

X
14.Apoi
ntPh asthecoo
rdi
nat
es(
1,2,3)ifPQ =5i+j+2k,f
in
d:-
(a
)T h
ec o
ordina
tesofpoi
ntQ (2mks)
(b
)T h
emo dulu
so fPQ (1mk)
15.Th
efigur
eb e
lowrepr
esen
tsar
igh
tp y
ramid.
As t
ri
ngisf
ix
edatA,t
henp
ass
est
hro
ughmi
dpoin
tsofe
dgesDC,VC,VBandf
ina
ll
yatA.
Gi
venthatt
hed i
mensi
onoft
hebasei
sas qu
areofsi
de5cmandVA=VB=VC=VD=6cm.Dra
wthenetre
pre
sent
ingt
heri
ght
py
ramid (
3mk s
)

16.I
nth
efi
gur
ebe
lowABi
spa
ral
l
elt
oCD,
BCa
ndADi
nt
ers
ecta
tE.
Giv
enDE:
EA=5:
1an
dBC=12c
m.Ca
lcu
lat
eth
ele
ngt
hofEC
(
3mks)

C D

B
F NGSCHEMESA
ORMARKI I
NBOX0724351706 Pa
ge|9
Ma
the
mat
ic
s121/
1,2

SECTIONI I
(Ans
we ronlyfi
veq
uesti
onsinthi
ssect
ion)
17.Thre
ep ar
tnersMut
ua,Muthokaan
dMwi kaic
ontr
ibut
edSh.600,
000,
Sh.400,
000andSh.800,
000res
pect
iv
elytos
tar
tabu
sin
ess
ofama t
atuplyi
ngMbumbuni–Ma ch
akosrout
e.Themata
tucarr
ie
s14pass
ang
erswi
thea
chp ay
ingKs
h.250.Themat
atuma
kestwo
round
st r
ipseac
hday,Sh.6000isusedt
oc ov
errunni
ngcost
sandwage
s.

(
a)Ca
lcu
lat
eth
eirn
etp
rof
i
tpe
rda
y (
2mk
s)

(b
)T hema tat
uwo rksfo
r25d ayspermonthandse r
vi
ceev e
rymo n
thatacos
tofKsh.10,000.Cal
cula
tethei
rmo nt
hl
yp r
ofi
ti
nJ une
(
1mk )
(c
)T het hreepartner
sagreedtos av
e40%o f
the24%t ob eshar
edint
herat
ioofth
eircontri
but
ion.
Ca l
cula
teMu t
hoka
’sshar
einmo nt
h
of. MOCKSKCSEPREDI CTIONS (4
mk s)
(d
)T hema tat
ud ev
e l
opedame ch a
nica
lprobl
ema ndtheyd e
cide
dtosel
lth
roughagentwh ochar
gedac ommissi
onof5%o nsel
li
ngpri
ce.
Eachp artnerrece
ive
d475, 000, f
romtheagentaft
erheh adt
akencommis
sio
n.De t
erminethepr
iceatwh i
chtheag
entsol
dthema t
atu
(
3mk s)
2
18.(a)Co mp let
ethetablebelowfortheequat
iony=x –6x+5
X 0 1 2 3 4 5 6
2
X 0 4 9 25
-
6x 0 -
36
5 5 -6 -
24
Y 5 0
(
2mk s)
2
(b
)Dr awt h egraphofy=x –6x+5u sin
gv al
uesinthetabl
e (3mks)
(c
)Us et heg r
ap htosol
vethee quat
ion
2
(
i) x –6x+5=0 (
1mk )
2
(
ii
) x =6x–7 (2mks
2
(
ii
i
) x –6.5x+5=0 (2mks)
19.Fourto wnsA, B,Ca ndDa r
es i
tua
tedsuchthatt
ownAi s40kmf r
omBo nab e
aringof3100.Cis60kmf r
omBo nab ear
ingof5700E.
0
An ot
he rtownDi s50k mf r
omAo nab e
ari
ngo fN70E.
(a
)Us ingas caleof1cmt orepres
ent10km, s
howt herel
ati
veposi
ti
onoft
hetowns (4mks)
(b
)F romy ourscal
ed r
awingd et
e r
mine:

(
i) Beari
nga ndd i
s t
anceofDf romC (2mks
)
(
ii
) Beari
nga ndd i
s t
anceofBf romD (2mks
)
(
ii
i
) Beari
nga ndd i
s t
anceofAf romC (2mks
)
20.Ama ta
tulefttownKa t7. 00a.mandt r
avel
ledtoward
stownMa tanave
ragespe
edof60km/
hr.
Ac a
rleftt
o wnMa t9.00a .
ma ndtrave
ll
edtowa r
dsKa ta
naver
agespee
do f50k
m/hr.Th
edi
sta
ncebe
twee
nthet
wotownsi
s324k
m.
Fin
d:-
(a
)T heti
mee ac hv e
hicl
ea rr
ivedatthe
irdest
inati
on
(
i) Ma t
atu (
2mks)
(
ii
) Car (2mks
)
(b
)( i)T hedistancethema tat
uc ove
redb e
forethecarst
art
edt
omo vef
romtownMt otownK (1mk)
(
ii
) Theti
met het wov e
h i
clesme tonthewa y (3mks
)
(
ii
i
) Howf artheca rwasfr
o mt ownKwh enthe
yme t (2mks
)
21.Inthefigu
reb elow,Oi sthec e
ntr
eo fth
ec i
rcl
e.PQisatang
enttoth
ec i
rc
leatN.Angl
eNCDis100a
n dang
leANPi s300

F
ORMARKI
NGSCHEMESI
NBOX0724351706 Pa
ge|10
Ma
the
mat
ic
s121/
1,2

Gi
vi
ngrea
son
sfi
nd
(
a)An
gleDON (
2mk
s)

(
b)An
gleDNQ (
2mk
s

(
c)An
gleDBA (
2mk
s)

(
d)An
gleONA (
2mk
s)

(
e)An
gleODN (
2mk
s)

22.T
heh
ist
ogr
amb
elo
wre
pre
sen
tst
hed
ist
ri
but
io
nof
mar
kso
bta
ine
dinat
est
.Th
efr
equ
enc
yoft
hes
eco
ndc
las
sis3.

(
a)Co
mple
tet
hetab
leb
elo
w (
3mk
s)
Mark
s 35–49
Fr
equen
cy 3
(
b)Us
eth
etab
lein(
a)a
bovet
ofi
nd

(
i)T
heme
anma
rk (
4mk
s)

(
ii
)Me
dia
nma
rk (
3mk
s)

23.Ame
tal
sph
ereh
asar
adi
uso
f5c
man
dde
nsi
tyo
f2.
4g/
cm3

(
a)Ca
lcu
lat
eth
ema
sso
fth
eba
lli
nkg (
4mk
s)

(
b)T
hebal
li
sdr
oppedi
ntoc
yli
ndr
ic
alc
ont
ai
nerwhi
chi
spa
rtl
yfi
l
ledwi
thwa
ter
.Th
eba
lli
sfu
ll
ysub
me r
geda
ndt
hec
yli
nde
rha
sar
adi
us
b
aseo
f8cm.Ca
lcul
at
eth
echangei
nwat
erl
eve
l (3mks)

(
c)T
hes
phe
rei
sme
lt
edd
ownt
ofo
rmame
tal
cyl
i
nde
rof
bas
era
diu
sof5c
m.Ca
lcu
lat
eth
ehe
igh
toft
hecyl
i
nderf
orme
d
(
3mk s
)

24.T
hed
isp
lac
eme
ntSme
tre
sof
amo
vin
gpa
rti
cl
eaf
te
rts
eco
ndsi
sgi
venb
y
3 2
S=2t –5t +4t+2
Det
ermin
e:-
(
a)Th
eveloci
tyoft
hep
art
ic
lewh
ent=2 (
3mk
s)

(
b)T
hev
alu
e(s
)oftwh
ent
hep
art
ic
lei
smo
men
tar
il
yatr
est (
3mk
s)

(
c)T
hed
isp
lac
eme
ntwh
ent
hep
art
ic
lei
smo
men
tar
il
yatr
est (
2mk
s)

(
d)T
hea
cce
ler
ati
ono
fth
epa
rti
cl
ewh
ent=5 (
2mk
s)
F
ORMARKI
NGSCHEMESI
NBOX0724351706 Pa
ge|11
Ma
the
mat
ic
s121/
1,2

CEKENAS
121/2
FORMF OUR
MATHEMAT I
CSPAPER2
MOCKSKCSEPREDI CT
IONS/..
TIME:21/
2HOURS

SECTI
ONI( 50MARKS)
Answeral
lque
sti
onsi
nthi
ssect
io
n
1. Uselo
g a
ri
thmsto4dec
imalp
lac
etoe
val
uat
e
1
/
3
0.7841x 0. 1356
Log84.92 (4mks)
2. T
hetopo fata bl
eisar egu
larpent
agon
.Ea c
hs i
deofth
ep e
ntag
onmeasu
res40.0c
m.Fin
dt h
emax
imu
mpe
rce
ntag
ee rr
ori
n
c
alc
ula
ti
n gt
hep er
imete
roftheto
po ft
hetab
le (
3mk
s)
3. Withoutus
ingac al
cul
ato
rorma t
hemat
ica
ltabl
es,
Simp
lycompl
et
el
yle
avi
ngyoura
nsweri
nfor
m,a+b c

s600
1-Co (
3mk
s)
1 +tan300

4. Us
eth
etr
ape
ziu
mru
lewi
ths
eve
nor
din
ate
stof
i
ndt
hea
reab
oun
dedb
yth
ecu
rve
2
Y=x+1, l
i
nesx=-
2,x=4a
ndx–a
xis (
3mk
s)

5. Gi
vent
hat x= +p ma
kePt
hes
ubj
ecto
fth
efo
rmu
la

3 2u+p (
3mk
s)

6. T
hef
i
gur
ebe
lows
howsac
ir
cl
ewi
ths
eca
ntsABEa
ndCDE,
itAB=4c
man
dBE=6c
man
dDE=4c
m,F
indt
hel
eng
tho
fCD
(
2mks
)

tdy/ 2
7. Ac u
rvep asse
sthroughthepoint(1,-
2).Giv
entha dx =3x –4x+1,
Fin
dth eequ a
ti
ono fthecur
ve (
3mk s)
8. Wa t
erflowsfromap ipeatt
hespe e
do f250l
it
resperminu t
e.Ift
hepip
eisusedtodra
inatankfu
llofwate
rme asu
rin
g3.2mb y2.5m
by2m. Ho wma nymi nut
eswouldittaket
od r
ainthetankcomp l
et
ely (3mk s)
2
9. Thedatab e
lows howstheageof10s tudent
spick
eda tra
n dominasecondar
yschool
,6,11,13,14,8,7,12,20,Pand9.IfƸfx =
1360.De te
rminethev al
ueofPh encefi
ndthesta
nd a
rdd evi
at
ionto3d .
p (
3mk s)
10.Solv
efo rxintheequatio
n
2 2
2s i
n x–1=c osx+s i
nxforO0≤ x≤360 (
4mk s)
11.Solv
efo rxi n 3log3x+4=l og324 (
3mk s)
12.Theresi
s t
ancetothemo ti
onofac ari
sp a
rtl
yvari
esasthes q
uareofthespe
ed.At40km/
ht here
sist
ancei
s530Na ndat60km/hi tis
730N. Wh atwil
lbetheresi
sta
ncea t70km/h (
4mk s)
13.Inatransfo
rma t
ion,anobjec
twitha r
ea4cm2isma ppedontoanimagewh os
eareai
s48cm2b yatra
nsfor
ma t
ionmatr
ix y+1 y
(
3mk s)
4 2

Fi
ndtheva
lueo
fy
14.Marg
areta
ndOti
enoh
adah
all
eac
h.T
hec
apa
cit
yof
eac
hha
llwa
s1920p
eop
le.
Whe
nfi
l
ledc
omp
let
el
yMa
rga
retwo
uldh
avee
qua
l

F
ORMARKI
NGSCHEMESI
NBOX0724351706 Pa
ge|12
Ma
the
mat
ic
s121/
1,2
numbero
fpeop
lei
nxrowsofs
eat
swhi
leOti
enowoul
dha
veequa
lnumb
erofp
eopl
einx+4rowsofs
eat
s.T
henumberofp
eop
lei
n
ea
chrowinMarga
reth
all
is2mor
eth
anthenumbe
rofpe
opl
eineac
hrowi
nOtie
nohal
l.
Cal
cul
atet
henumbero
fpe
oplepe
rro
win
Ot
ien
o’shal
l
. (4mk s
)
2 2
15.Wr
it
eane
quat
io
no fac
ir
cl
eth
ath
asad
iame
terwh
osee
ndp
oin
tsa
re(
2,7)a
nd(
-6,
15)i
nth
efo
rmx +y+ax+b
y+c=0wh
erea
,
ba
ndcar
ein
tege
rs (
3mks)

d(1+1/x7 4
16.(
a)Ex
pan 4)u p
tot
het
ermsi
nx (
1mk
)
7
(
c)Us
eth
eex
pan
sio
nab
ovet
oes
ti
mat
eth
eva
lue(
0.975)co
rre
ctt
o4s
ign
if
ic
antf
i
gur
es (
2mk
s)

SECTI
ON2( 50MKS)
Att
emptonl
yfiveque
s t
ionsi
nthisse
cti
on
17.TheHir
epurchas
e(H.P)pri
ceofapubl
icadd
res
ssys
temwa
sKsh.276,000.Ade
posi
tofKsh.
60,
000wa
spa
idf
ol
lo
wedb
y18e
qua
l
month
lyins
tal
lment
s.Thecashpri
ceofth
epubl
ics
yst
emwas10%lessthant
heH.Ppr
ice.

(
a)Ca
lcu
lat
e

(
i)T
hemo
nth
lyi
nst
al
lme
nts (
2mk
s)

(
ii
)Th
eca
shp
ric
e (
2mk
s)

(
b)Acust
omerdeci
dedtobuyt
h esys
temi
ncashandwasal
lo
weda5%d i
sco
u n
tonthec
ashpri
ce.Hetoo
kab a
n kl
oant
ob u
ythe
s
y s
teminca
sh.Theban
kc h
a r
gedcompou
n di
nte
res
tonthel
oana
tth
erateof20%p.
a.Th
eloanwa sr
epa
idin2year
s.Cal
cul
at
eth
e
amountr
epai
dtotheba
nkb ytheendo
fthese
condyea
r. (3mk s
)

(
c)Ex
pre
ssa
sap
erc
ent
ageo
fth
ehi
rep
urc
has
epr
ic
e,t
hed
if
fe
ren
ceb
etwe
ent
hea
mou
ntsr
epa
idt
oth
ebanka
ndth
ehi
rep
urc
has
epr
ic
e.
(
3mks)

18.(
a)Co
mpl
et
eth
eta
bleb
elo
wfo
rgr
aph
sof
Y=Si
nxa
ndY=2s
in(x+30)

x 0 30 60 90 120 150 180 210 240 270 300 330 360


Si
nx 0 0.
87 0.
5 -0.
87 -0.
5
2s
in(
x+3 1 0.
5 1.
74 0 -
1 -
1
0)

(
b)Ont
heg
ridp
rov
ide
ddr
awt
heg
rap
hso
fY=s
inxa
ndY=2s
in(
x=30)f
orO x 3600 (
4mk
s)

(
c)St
at
eth
etr
ans
for
mat
io
nth
atma
psY=s
inxo
ntoY=2s
in(
x+30) (
2mk
s)

(
d)F
indt
hev
alu
eso
fxwh
ichs
ati
sf
yth
eeq
uat
io
nSi
nx–2s
in(
x+30)=0 (
2mk
s)

19.(
a)Ana
ri
th
met
icp
rog
ess
ioni
ssu
cht
hatt
hef
i
rstt
ermi
s-5,
thel
asti
s135a
ndt
hes
umo
fth
epr
ogr
ess
ioni
s975.

Cal
cul
at
e
(
i)Th
en u
mbero
fte
rmsi
nth
ese
ri
es (
4mk
s)

(
ii
)Th
eco
mmo
ndi
f
fer
enc
eoft
hep
rog
res
si
on (
2mk
s)

(
b)T
hesumoft
hefi
rs
tth
reet
ermso
fag
eome
tri
cpr
ogr
ess
ioni
s27a
ndf
i
rstt
ermi
s36.
Det
ermi
net
hec
ommo
nra
ti
ona
ndt
hev
alu
eof
t
hef
ourt
hte
rm (
4mks
)

20.Abo a
tPlea
vesp
o r 450N,
tA( 500W)andsa
il
satanav
era
gespee
dof10k
not
s.Itsa
il
sd uee
astal
ongap
a r
al
le
lofLa
ti
tudet
oB(450N,
0 0 0 0 0
42W)a ndth
ensai
lsd
uen o
rthtoC(
48N, 42W).Anot
herb
oatQl
eave
sD(55N, 10W)a tt
hesamet
imeasPle
avesA.i
tsa
il
sd ue
westandt
hendueSou
thtome e
tboa
tPa n
dC.

(
a)Ho
wlo
ngd
oesi
tta
keb
oatPt
ore
achp
oin
tC? (
4mk
s)

(
b)I
fbo
atQs
ail
satt
hes
ames
pee
dasb
oatP.
Howl
ongd
oest
hef
orme
rta
ket
ore
achp
oin
tC (
4mk
s)

(
c)Atwh
ats
pee
dwo
uldb
oatQh
aves
ail
edt
ore
achp
oin
tCa
tth
esa
met
imea
sbo
atP(
2mk
s)

21.Th
eprob
abil
i
tythatourschoo
lwil
lhosts
occe
randr
ugbyt
our
name
ntt
hi
syeari
s0.
8.I
fwehost
,th
eprob
abi
li
tyo
fwinni
ngsoc
ceri
s
0.
7.I
fwedon’thostth
ep r
obabi
l
it
yo f
winni
ngso
cce
ris0.
4.I
fwewi
nsoc
certh
epr
obab
il
it
yofwi
nnin
grugb
yis0.8,
oth
erwi
sei
fwe
l
oset
hepro
bab i
l
it
yo fwin
ningrugb
yis0.
3.

F
ORMARKI
NGSCHEMESI
NBOX0724351706 Pa
ge|13
Ma
the
mat
ic
s121/
1,2
(
a)Dr
awat
reed
iag
ramt
ore
pre
sen
tth
isi
nf
orma
ti
on. (
2mk
s)

(
b)Us
eth
etr
eed
iag
ramt
ofi
nd:
-

(
i)T
hep
rob
abi
l
it
yth
atwel
oseb
othg
ame
s (
2mk
s)

(
ii
)Th
epr
oba
bil
i
tyt
hatwewi
l
lwi
non
lyo
neg
ame (
3mk
s)

(
ii
i
)Th
epr
oba
bil
i
tyt
hatwewi
l
lho
sta
ndl
oseb
othg
ame
s (
2mk
s)

(
iv
)Th
epr
oba
bil
i
tyt
hatwewi
natl
eas
ton
ega
me,
ifweh
ost (
1mk
s)

22.T
hef
i
gur
ebe
lows
howsar
igh
tpy
rami
dVABCDwi
thas
qua
reb
aseo
fsi
de6c
m.

VA=VB=VC=VD=12c
m

(a
)Ca l
cula
te:
-
(i
)T hehei
ghtofthepyr
amidcorr
ectt
o2d .
p (
3mk
s)
(i
i
)T heangl
ebetweenthepl
anesVADandVBCc or
rectt
o2s i
gni
fi
can
tfi
gur
es (
3mk
s)
()B1a
b ndC1arepoint
sonVBa ndVCresp
ecti
vel
ysuchthtVB1:
a VB=VC1:VC=1:3
Cal
cula
tetheangl
ebetwee
np l
aneABCDa dAB1C1D
n (
4mk
s)
23.Inth
efigu
reb e
low,Eist
hemid-po
into
fAB.OD: DB=2: 3an
dFisthepoi
nto
fin
ter
sec
ti
ono
fOEa
ndAD

(a
)Givent hatOA=aa ndOB=be x
pressintermso f aan db
(i
) OE (
1mark)
(i
i
)AD (
1mark)
(b
)Givenf ur
thert hatAF=tAD a ndOF=sOEwh eresa ndtarescalar
s,
Fi
ndthev alu
e sofsandt (
5mks)
(c
)Sh owth a
tO, Fa ndEa r
ec ol
li
near (
3mks)
24.Arel
ieforganizat
ionhastotrans
portatleast80p eopleanda tl
east18tonnesofs
uppli
est
osit
e.Ther
ea r
etwotype
so fv
ehi
cl
e
av
ail
able,typeAa ndtypeB. T
ypeAc anc ar
r y900k gof s
u p
plie
san d6peoplewhi
lety
peBc a
nc a
rry1350kgofsu
ppli
esand5peop
le.
Ther
ea tmo s
t12v eh
icl
esofeachtypea va
ilabl
e. Bytakingxtoreprese
ntthenumberofve
hic
lesoft
ypeAa ndyt
or e
pres
entt
he
numbero fvehicl
esoftypeB.
(a
)Wr it
ed owna llt
heinequali
ti
estorepre
sentt heab ov
ein f
orma t
io
n (
4mk s)
(b
)Ont heg ri
dp rovi
dedd r
awa l
linequ
a l
it
iesin( a
)a bove (
4mks)
(c
)Us ethegr a
phi n(b)abovetodet
erminethel eastnumb erofvehi
clerequi
re
da tt
hesit
e (
2mk s)
FORMARKI NGSCHEMESI NBOX0724351706 Page|14
Ma
the
mat
ic
s121/
1,2

IGEMBE
Paper1
.MOCKSKCSEPREDICTI
ONS.
Time:2½Hou
rs
ICSSEFORMFOURTRIALEXAM.
MOCKSKCSEPREDI
CTI
ONS.
 
SECT
IONA:
1.Ev
alu
ate
:
(
3ma
rks
)
 
 
 
2.So
lvet
hee
qua
ti
on:
(4ma
rks
)
 
 
 
3.40womenc
andoap
ieceo
fwo
rkin60day
swo
rki
ng9h
our
sad
ay.Ho
wma
nymo
rewo
mens
hou
ldb
ead
dedi
nor
dert
odot
hesame
wo
rki
n48day
swor
kin
g10hour
saday
. (2ma
rks
)

4.I
f .Fin
dwitho
u tus
ingta
ble
sorcal
cul
at
ors
,si
n(90-x)+cos
(90-x)
.Leav
eyour
 
an
sweri
nsur
dandsimpl
estfo
rm.(3marks
)
5.Areg
ula
rpo
lygo
nh ast
hesumo fal
li
tsi
nte
ri
orangl
e s1260o.
sa Findt
hesiz
eofeac
hexte
ri
oran
gleo
fth
epo
lyg
on.
(3ma r
ks)
6.Inthefigur
ebelowABCDi sac yc
licquadr
il
ate
ralinwhichAD=DCa ndABisp a
ral
lelt
oCD.
 
 
 
 
 
 
 
 
 
 
 
 
Giv
enthatangeABC=80o,
l f
indthesiz
eo f:
a)DAC (1mark)
b)BAC (1mark)
c)BCD (1mark)
7. Alineisdra
wnt hr
oughp oi
nt(2,1)perp
endic
u l
artoli
n ex+2y=4. Fi
ndtheequat
iono ft
heli
ne. (3mar
ks)
8. Findtheval
ueofxif:
32x+3+1=28 (3ma r
ks)
9. Giventha
ta=2i -3j+2k ,b=3i -4j-kandc=i +3ka ndR=3a-b+c .Findthemagnitu
d eofRc or
rec
tto3si
gni
fi
cantfi
gur
es.
(4ma r
ks)
10.Thea r
eaofarect
anglei
s24c m2,ifi
tsl
engthis5cml o
n gert
hanit
swidt
h,f
indit
slengthofthere
ctan
gle. (
4ma r
ks)
11.Findthegre
ates
tn u
mb erwhichwh e
nd i
vid
edb y247, 367and607lea
vesarema i
nderof7ineachcase
. (3ma r
ks)
12.Findtheint
egr
alval
uesthatsat
is
fytheinequa
lit
ies.
  4x-9<6+xa nd8-3x<x (3ma rk
s)
13.Thec o
stof3p ai
rsoftr
ouser
sa nd3s hi
rt
sissh.2400.T h
ecostoft
wop a
ir
so ft
rouse
rsa nd3shir
tsi
s1975.Fi
ndth
ec o
s tofo
nepai
r
oftrous
ersand4s hi
rt
s. (
3ma r
ks)
 
14.Thec u
rrenc
ye x
changerat
esatag iv
enb a
nkinKe nyaareasfol
l
ows.
 
 
Cu
rre
ncy Bu
yin
g Se
ll
in
g
1St
erl
i
ngp
oun
d 135.
50 135.
97
1USd ol
la
r 72.
23 72.
65
Ato
uri
starri
vedi
nKeny
awit
h5000USdol
lar
swh
ichhecon
vert
edt
oKe n
yas
hil
l
ing
sup
ona
rri
val
.Hes
pentKsh
.214,500a
nd
c
onver
tedther
emai
ni
ngt
oSte
rl
in
gpou
nds
.Howmanypo
und
sd i
dhere
cei
ve? (2mar
ks)

F
ORMARKI
NGSCHEMESI
NBOX0724351706 Pa
ge|15
Ma
the
mat
ic
s121/
1,2
 
15.Th ecircl
eb elowwh o sea reais18. 05cm2c ir
c umsc ri
b esat r
iangl
ePQR, wh er
ePQ=6. 3cm, QR=5. 7cma ndPR=4.8cm.
 
 
 
 
 
 
 
 
 
 
 
 
F
indt hea r
eao fth eu n shadedp art. (
4ma r
ks)
 
16.Ap o
intTd i
videslineABi nterna
llyinth era t
io5:2. Giv enthatA(-4,10)andBi s(10,3)f i
ndthec oordin
ateofT.
(
3ma rk
s)
 
  
 SECT IONB:  
17.Ag r
ou po fpeo plep l
a nn edtoc on t
rib
u tes h.2000, 000t os ta
rtab usi
ness.Howe v
erb eforetheco ntr
ibut
ionswer
ema de40me mber
s
p
u l
ledo ut.Asar e sulte acho ftherema iningme mb erswe r
etoc ontr
ibut
es h
.2500mo r
e.
  a )F i
n dtheo riginal nu mbe rofme mb ersint heg ro up. (
3ma r
ks)
b
) Af t
ere achme mb e rc ontri
buti
n gha l
fth ee x
p ecteda mo untofmo neythememb erso p
tedtog eta ban
kloantha
twou l
d
of
fset¼o fther e ma i
n i
nga mo unt.Ho wmu chwo ulde ac
hme mbercontr
ibut
ea f
tertheb ankl
o an. (
4ma r
ks)
  c )T wen t
yn e wme mb ersjoin
edt heg roupj ustb eforeth ebusine
ssstart
ed.Asaresulttheo l
dme mb ersweretobere
funde
d
somemo ney. Ho wmu chwa sther efund . (
3ma r
ks)
18.Th etableb el
ows howsg r
o upedma r
kso btainedb y100c andi
d at
esandfrequenci
esfora l
l gr
oups.
  a ) Us et heinforma tiont od rawa no gi
v ec urve. (
6ma r
ks)
 
 
Ma r
ks 1-10 11-20 21-30 31-40 41-50 51-60 61-70 71-80 81-90 91-100
Frequency 2 6 10 13 20 25 10 8 4 2
 
b
) Us ey ourg rapht oe st
ima t
eth e:
i
) me diano fthed at
a (
1ma rk
)
i
i) upp erq ua r
til
e (
1ma r
k)
i
ii
) lowerq uartil
e (
1ma r
k)
i
v) 80thp ercen t
il
e (
1ma r
k)
19.As trai
gh tli
nep asse sth rought hep oi
nts( 8,-2)a nd( 4, -4)
a
) Wr iteitse quationi nt heforma x+b y+c=0wh e
r ea,ba ndca r
einteger
s. (
3ma rk
s)
b
)I fth eli
n ein( a)a bo vecu t
sth ex-
ax i
sa tp ointP, dete r
minethec oor
dinat
esofP. (
2ma r
ks)
c
) An o t
herl i
newh ic hisp erpe
n dicul
art oth eli
n ei n(a)a bo
v epasse
sthroughpo i
ntPa n dcut
sth ey-axi
satthepoi
ntQ.
Determin ethec o ord i
nateso fpoi
n tQ. (
3ma rk
s)
d
)F i
n dthel engtho fQP. (
2ma r
ks)
20.a) Co mp letethet ab l
eb el
owf ory=s i
n2xa ndy=s i
n( 2x+30)g i
vi
ngy ouranswerto2d .
p. (
2ma r
ks)
 
0 15 30 45 60 75 90 105 120 135 150 165 180
sin2x 0       0. 87       -0.87       0
s
in(2x+30) 0. 5       0. 5       -
1       0.
5
b
) Dr awt heg raph so fy=s in2xa ndy=s in(2x+30)o nthea x
is. (
4ma rk
s)
c
) Us ey ourg rapht os ol
ves i
n( 2x+30)-s in2x=0. (
1ma r
k)
d
) De termin ethet ran sf
o r
ma ti
onwh i
c hma pssin2xo ntosi
n( 2x+30) (
1ma r
k)
e
) St at
et hep eri
o da n dthea mp li
tudeo f y=s in(2x+30) (
2ma rk
s)
21.Int hefi
g ureb el
o wDAi st h
ed iame t
ero fac ircl
e. ABCDEc ent
reO. AB=BCa nda ngeDAB=36o.
l
 
 
 
 
 
 
 
 
 
 
 
 
F
ORMARKI
NGSCHEMESI
NBOX0724351706 Pa
ge|16
Ma
the
mat
ic
s121/
1,2
 
  a ) Givingre
asonsfindt h
es i
zeo fangl
e:
  i ) CDB (
2ma
rks
)
ii
) DBC (
2ma
rks
)
ii
i
) DOC (
2ma
rks
)
iv
) OCA (
2ma
rks
)
v) DEB (
2ma
rks
)
 
 22.
Thefi
gurebel
owi sri
gh tpyrami
dwh oseslant
ingedgeis14c
m, ABis8cmandBC=6cm.
 
 
 
 
 
 
 
 
 
 
 
 
  a )F i
ndthesurf
acea reatothepyramid . (
6ma
rks

b)F i
ndthevolumeo fpyramid. (
4ma
rks
)
23.a) Wr i
tedownt heinequali
ti
eso fL1,L2,L3,
L 4t
hatsat
is
fyt
heregi
onindi
cat
edb
ylet
te
rR. (
8ma
rks
)
 
 
 
 
 
 
 
 
 
 
 
 
 
 
 
 
 
 
 
 
 
 
 
 
 
 
 
 
 
 
 
 
 
 
b)F i
ndtheareaoftheu nshadedregionlabell
edlet
te
rR. ( 2marks)
24.Ap a
rti
cl
emo vesalon
gas tr
aightl
ines uchthatit
sdis
plac
ementsmetr
esfr
omagive
npoin
tis
3 2
s=t -5t +3t+4wh eretisti
mei nsec o
nd s
.
Fi
nd:
a)t hedisp
lacemento fthepart
icl
ea tt=5 (
2ma
rks
)
b)t hevelo
cit
yo fthep ar
ti
clewh ent=5 (
3ma
rks
)
c)t hevalu
eo ftwhe nthep ar
tic
leismo me nt
a r
il
yatre
st (
3ma
rks
)
d)t heaccel
erat
iono fthepart
icl
ewh e
nt=2 (
2ma
rks
)

F
ORMARKI
NGSCHEMESI
NBOX0724351706 Pa
ge|17
Ma
the
mat
ic
s121/
1,2
 
IGEMBE
Paper2
.MOCKSKCSEPREDI CTIONS.
Time:2½Ho urs
ICSSEF ORMF OURT RIALEXAM. MOCKSKCSEPREDI
CTI
ONS.
 
SECT IONI:(50MARKS)
1. So lvet
heeq ua
ti
o n:
Log3(x+3)=3l og3+2 (
3ma
rks
)
 
2.Ma k
enthes ubj
ectofthefo
r mul
a (
3ma
rks
)
 
3. At rans
lat
ionma pspoi
ntA( 6,4)onoA1(
t 10,
8)
  a ) De t
erminethetra
nsla
ti
o nvec
tor
. (
1mar
k)
b) Ap oi
n tB1isthei
ma g
eo fB(4,10)u
nde
rth
esa
metra
nsl
at
io
n.F
indt
hel
eng
thA1B1l
eav
ingy
our a
nswe
rinsu
rdform.
(
2mar
ks)
4. Thepoi
nts(5,5)and(-3,-
1)a r
eendsofad i
amete
ro fac i
rc
lece
ntr
eA.
  De te
rmine:
a)t hecoordi
nate
so fA. (
1ma
rk)
2 2
b)T heequ a
tio
no fac i
rc
leexpre
ssi
ngitinfor
mx +y +a x+by+c=0 (
2ma
rks
)
5. Witho
utusi
n gasetsquareorap r
otr
act
orc o
nstr
uct:
a)T ri
angleABCs uchthatAB=8c m,BC=6c ma n dABC=30o (2ma
rks
)
b) Me a
surethelengt
hAC. (
1ma
rk)
c) Drawac irc
let
h a
tt o
uc h
esside
sAB, BCandAC. (
1ma
rk)
6. I
nt h
efig
u r
eb el
owPQRSi satra
pezi
umwi t
hQRp arall
eltoPS.QR=6cm,RS=4c
m,QS=9c
man
dPS=10c
m.
 
 

 
 
 
Cal
cula
te:
a)t hesizeo fangl
eSQR (2ma r
ks)
b)t heareao ftr
ianglePQS (1ma r
k)
 
7.T hegradesA,Ba ndCo fri
cewe remixedinthera
ti
o3:4:5.T hecos
tperkgofea
chofth
egr
adesA,Ba
ndCwe r
eKsh.120,Ks
h.90
andKsh.
60r esp
ecti
vely.
  Ca l
cula
te:
a)t heco stofonekgo ft
h emixture
. (2ma r
ks)
b)t heselli
ngp ri
ceo f5kgo fthemixtur
ewa ssol
dat8%p r
ofi
t. (2ma r
ks)
8 3
 
8.a ) Expan d(1+2x ) ina scendi
ngp owerofxuptoincl
udi
ngtheter
mx . (1ma r
k)
8
  b ) He ncee val
uat
e(1. 02) to3d eci
malplac
e s
. ( 2ma r
ks)
 
9.Ex pr
essins urdfor
ma n dsimp l
i
fyb yrat
io
nali
zingt
hedenominat
or. (
3marks
)
  1+c os30o
1-s i
n60
 
10.PandQa ret woma tr
ice s
.I f fi
ndQg i
venthatP2=(P+Q) (3ma r
ks)
 
 
2 2
11.Fact
ori
zea -b.
Hencefi
ndt hee x
actvalueo f25572-25472 (2ma r
ks)
12.Inthefi
gureb elowBTi sat a
ngenttothec i
rcl
eatB.AFCTa ndBFDa r
estr
ai
ghtl
ine
s.AF=6cm,CT=8cm,BF=4.8cma ndFD=
5cm.

 
 
 
 
 
 
 

F
ORMARKI
NGSCHEMESI
NBOX0724351706 Pa
ge|18
Ma
the
mat
ic
s121/
1,2
 
 
 
Fi
ndthelengt
ho f:
  a )F C (
2mar
ks)
b) BT (
1mar
k)
13.Ab ankchargesc ompoundi nt
e r
estonmo neyb or
rowed.Ab usi
nes
sma nb o
rro
wedKs h.
16000fro
mt heb a
nk.Hepa
idbac
k
Ks
h .
25000a f
te
r2y e
ars.Fi
ndt h
ei n
ter
estra
teperannum. (
3mar
ks)
14.Sol
veforxintheeq ua
ti
on
l
og(5x+75)-2l o g3=l og(2x-9) (
3mar
ks)
 
2
15.Usethemid-
ordi
n at
erulewithsixst
rip
stofi
ndtheareabounde
db ycur
vey=x +1,theli
nesx=-4,x=8andthex-
axi
s.
(
3marks
)
16.Atra
inmo v
ingata naver
a g
es peedof72km/htake
s15s e
condstocompl
ete
lyc
rossabri
dgetha
tis80met
reslo
ng.
a) Expres
s72k m/hinme trespersec
ond. (
1mar
k)
b)F indthel
eng t
ho ft
h etr
aininme t
res
. (
2marks
)
 
 
SECTIONI I
:
 
17.Thef
ollo
wingtableshowsther a
teatwh i
chinc
omet a
xwasc h
argeddur
ingace
rta
inyear
.
 
 
 
Mon
thl
yta
xab
lei
nco
mei
nKs
h. T
axr
ate%
0-9860 10
9861-19720 15
19721-29580 20
29581-39440 25
39441-49300 30
49301-59160 35
o
ver59160 40

Ac i
vilservante arnsab asi
cs alar
yo fKs h .
35750a ndamo nthlyh ou seallowa nceo fsh.
12500.Thec i
vilser
vanti
se nt
it
le
dt oa
person alr
eliefo fsh .
1062p ermo nth. Ca l
culat
e :
  a )t ax a
blein co me (2marks)
b)c al
culateh isn etmo nthlyt ax (
5ma r
ks)
c) Ap artfromt hes a
la r
yth efo l
lowi ngd eductiona r
ea ls
oma def
r omh ismo nthlyin come.
 
WCPSa t2%o f t
h eb asics al
ary
Lo anr e
p aymen tKs h.1325
NHI Fsh.480
 
Calcul
a t
eh i
sn e
tmo nt
h l
ye ar
nin g. (3marks)
18.Th eprod uctoft hef i
rstthreete rmso f ag eome tr
icp r
o gr
essionis64. I
fth efi
rsttermi saa ndthecommo nrat
ioisr.
a) Ex pressri nt ermso fa. (3marks)
b) Gi ventha tth esumo fthet hreet ermsi s14. Findthev al
uesofaa ndra ndh enc ewr i
tedowntwo
po ss
ibleseq uenc ese a
c hu ptot he4t ht erm. (7marks)
19.Aq u
a dri
lat
e r
al ABCDh asverticesA( 4, -
4),B(2, -4),C(6,-6)andD( 4,-2).
  a ) Ont heg ri
dp rovi
d ed,drawt h
eq uad ri
lat
eralABCD. (2marks)
  b ) A1B1C1D1i st heima geo f ABCDu nd erpositi
veq uart
erturnab o uttheo r
igino nt hes a
meg r
id,dr
awt h
eima ge
A1B1C1D1. (2marks)
  c ) A11B11C11D11i sth eimag eo fA1B1C1D1u nderthet r
ans f
ormationg ivenb yth ema t
ri
x
 
  i) De terminet h ec o
o rdi
n at
e so fA11B11C11D11 (2ma rks)
  ii) Ont hesa meg ri
d, drawt heq ua dril
a t
eralA11B11C11D11 (
2ma r
ks)
  iii) De termineas i
nglema trixth a
tma psABCDo n oA11B11C11D11
t (
2ma r
ks)
20.Th ep r
ob a
b i
li
tyt ha tap upilgoe stos c hool byab o d
ab odais2/3an db yama tat
ui s1/ 4.Ifheuse
sab odabodathepro
b a
bil
i
tythathei
s
l
a teis2/5an difh eu sesama tatut hep ro babi
li
tyo fbe i
nglateis3/10. I
fh eu s
eso therme ansoft
rans
porttheproba
bil
it
yofbei
ngl a
tei
s
3
/20.
  a ) Dr awat r eed iagramt ore presen tthisinforma t
ion. (3marks)
  b )F i
n dthep ro babil
itythath ewi llb elateforsch ool. (3marks)
c)F i
n dthep ro babil
itythath ewi llb elateforsch oolifhed oe
sn o tusema t
atu. (
2ma r
ks)
 
d) Wh atisth ep robabil
itytha thewi l
l notbel at
efo rschool? (2marks)
o o o o 22
21.Twot ownsPa ndQa r
el ocateda tP( 60N, 29W)a ndQ( 60N, 31E)t a
k i
n g= / 7a ndR=6370k m.
F
ORMARKI
NGSCHEMESI
NBOX0724351706 Pa
ge|19
Ma
the
mat
ic
s121/
1,2
  Ca l
cul
atethed i
sta
nc ebetweenPa ndQ.
i
) a longthep ar
al
leloflat
it
ud ei
nk m. (
2ma
rks
)
i
i)a longthelongit
u dei
nn m. (
2ma
rks
)
i
ii
)I ft
h elocalt
imea tPis1200h oursd e t
ermin ethel
o c
alti
mea tQ. (2ma
rks
)
b) Ana e
ropl
aneflewd ueSou t
hfromA( 60oN, 45oE)toBad i
sta
n c
eo f8008k
mf r
omA. F
indth
ep o
sit
io
nofB.
(4ma
rks
)
22.OABCisap a
ral
lel
ogramMi sthemidp o
in tof OAa ndAX=2/ 7AC,OA=aa ndOC=c
 
 
 
 
 
 
 
a)i ) Ex p
ressthefoll
owingintermso f vectorsaandc
  i) AC (
1ma
rk)
i
i) AX (
1ma
rk)
i
ii
) MX (
2ma
rks
)
b)I fAY=h ABa n
dMY=k MX. ExpressMYi nt wod i
f
fere
ntwaysh e
nc ef
indt
hescal
ar
sha ndk. (4ma
rks
)
c)F indther a
ti
ono fAYa ndYB.AY:YB. (2ma rks
2
23.a) Comp l
et
ethetab l
efory=3x -2x-8f o r-3<x<3 (
2ma rks
)
 
x -3 -2 -
1 0 1 2 3
x² 9 4 1 0   4 9
3x² 27     0     27
-2x 6     0      
-6
-8 -8 -8 -
8 -
8 -
8 -8 -8
y 25     -
8      
2
b) Dra wtheg r
apho fy=3x -2x-8f o r-3<x<3 (4ma
rks
)
c) Uset hegraphtos ol
vethefol
lowinge q
u ati
on s
:
2
i
) 3x -2x-8=0 (
1ma
rk)
2
i
i) 3x +x-5=0 (
3ma
rks
)
24.Thedi
ag ra
mb elowrepresen
tsac uboidPQRST UVWi nwh i
chUV=4. 5c m,VW=8c mand
WR=6c m
 
 
 
 
 
 
 
 
 
 
 

Ca
lcu
lat
e:
  a)thelengt
hofUR (
2ma
rks
)
b)i) t hesi
zeoft
heang
lebet
weent
hel
inesURan
dUW (
2ma
rks
)
i
i)t hesi
zeofa
ngl
ebetwe
enthel
i
nesPQa n
dPV (
2ma
rks
)
c)T hesi
zeoft
heangl
ebet
weenth
epl
anesPQWTandUVWT (
2ma
rks
)
d)L eng
thofsi
deVR (
2ma
rks
)
 
 
 

 
KANGEMA 
FORM4ENDOFT
ERM2EXAM

121/1
MATHEMATI
CS
Pape
r1
.MOCKSKCSEPREDI
CTI
ONS.
Ti
me:2½Hour
s
F
ORMARKI
NGSCHEMESI
NBOX0724351706 Pa
ge|20
Ma
the
mat
ic
s121/
1,2
 
SECTI
ONI:(50MARKS)
Answe
ral
lqu
e s
ti
onsi
nth
esp
ace
spr
ovi
ded
.
 
1.Wi t
houtu si
ngac alc
ulat
orormathemat
ica
ltabl
essi
mpli
fy. (
3ma r
ks)
 
36-8x-4-15÷- 3
3x=3+- 8( 6-( -2)
2.Simp l
i
fyc omp let
ely: (
3ma r
ks)
2 2
3x -5x y-2y
2 2
y -9x
3. Fin
dt hevalu eofxinthefoll
owi
nge qu
ati
on:
64x+1+82x=1040 (
4mar
k s
)
4. Theima geo fA( 5,5)underanenl
arg
ementscal
efact
or- sA1(
2i 8,
7).Fi
ndthecoo
rdi
nat
esoft
hecen
treofenl
ar
gement
.
(3mark
s)
5. Thi
rtytwome nwo rk
ingatther
ateof9hoursadaycancomple
teapie
ceofworkin7days
.Howma n
ymo remenworki
ngatt
herat
e
of8h oursad aywo ul
dcomplet
ethesamewo r
kin6d a
ys. (
3ma r
ks)
6. Inthefigureb elowtri
angl
eDECi ssi
mil
artotr
ian
gleDABa n
dECi sp
aral
le
ltoAB.Gi
venth
atDE:DAis3:5,f
indt
herat
ioo
ftheare
a
oftri
angleDECt othatoft
rapez
iumABCE. (
4ma rk
s)
 
 
 
 
 
 
 
 

 
7. F
indt
hei
neq
ual
i
ti
est
hatd
efi
net
her
egi
onRs
howni
nth
efi
gur
ebe
low.
(3ma
rks
)
 
 
 
 
 
 
 
 
 
 
 
 
 
 
 
 
 
 
 
 
 
 
 
 
 

8. Fouri
nte
rio
rangl
esofahexagona
re100o,140o,125oand105o.T
hefi
ft
hint
eri
oran
glei
sfo
urt
ime
sth
esi
xt
han
gle
.Fi
ndthef
if
th
an
g l
e. (3mar
ks)
9. Gi
venthats
in=2/3an
di sanac
utea
ng l
e,f
indwithou
tusi
ngtabl
eso
rcalc
ula
tor
.
a)t an (1mar
k)
b)c os(90-) (2mar
ks)
10.Thecur
renc
yexc
hangera
tesofagi
venba
n kar
ea sfol
l
ows.
F
ORMARKI
NGSCHEMESI
NBOX0724351706 Pa
ge|21
Ma
the
mat
ic
s121/
1,2
 Cur
rency Buying(Ksh.
) Se l
li
ng(Ksh)
1Sterl
ingpo u
nd 145.80 146.20
1USd o
ll
ar 100.80 101.00
Atouris
tarriv
edinKe nyawi th7000USd oll
arswh ichh econvert
edtoKe
nyashi
l
li
ngsup
ona r
riv
al.Hespe
ntKsh.332790and
conv
erte
dth eremaini
ngp oun ds.Howma nypoundsd i
dh erece
ive? (
3ma r
ks)
11.Twoline
sL 1andL2a resu c
ht hatbot
hp a
ssthroughthep ointA(x,x)
.Giv
ent
hatL
1hasagr
adi
entof3a n
dal
sopasse
sthroughB(
6,
8),
fi
nd:
a)t hevalueofx (2ma rk
s)
b)t heequati
ono fL2ifital
sop ass
esthr
ou ghpoi
n tC(2x,3x)i
nt hef
ormy=mx+c. (
2ma r
ks)
12.Div
idetheli
neb el
owi ntothreeequalpa
rts
. (2ma rk
s)
 
 
 
13.Giv
enthatOP=3i -2ja ndOQ=8i -5j,f
i
nd|PQ|t o3s i
gn i
fi
cantf
igur
es. (3ma rk
s)
  ~ ~ ~ ~ ~ ~
14.Usethetab
lesofrecipr
o cal
sa ndsquar
erootstoevalu
ate:
 
0.1 + 0. 498 (3ma rk
s)
0.0351
15.Uselogar
it
hmst oevaluate: (4ma rk
s)
 

16.Atai
lorboug
h tapai
roftr
ouser
satsh.
1600.Hemarkedthepri
ces
u c
hthataf
terall
owi
ngh i
sdis
counto
f20%h ewo u
ldsti
l
lmak
ea
pro
fi
tof30%o nthecos
tpric
e.Dete
rmin
ethepr
iceatwhi
chthepai
roftr
ouser
swa smark
ed. (2marks
)
 
SECTI
ONI I:(50MARKS)
Att
emptfi
vequest
ion
sonlyfr
omthisse
cti
on.
 
17.Thefi
gur
eb el
o wre
pres
entsamodelofasol
ids
truc
turei
nt h
eshapeofafru
stumo faconewit
hh e
misphe
ri
calto
p.Thedia
met
erof
th
eh e
mispher
ical
parti
s70cma ndise
qualt
othedi
a met
erofth
etopofthef
rustu
m.T h
efrust
umh a
sab a
sediamet
erof28cma
nda
sl
anthei
ghtof60cm.
 
 
 
 
 
 
 
 
 
 
 
 
 

 
Ca
lcu
lat
e,t
aki
ng=22/7
  a)theare
aofth
eh e
misp he
ri
c a
lsu
rfa
ce. (
4ma
rks
)
b)thesur
fac
eareaoft
h ecur
v e
dsurf
ace (
4ma
rks
)
c)thetot
als
urf
aceare
ao fthemodel
. (
2ma
rks
)

18.T
hed
iag
ramb
elows
howstr
iang
leOABi
nwhi
chOM:MB=1:3a
ndAN:NB=1:2.
T
hel
i
nesONandAMmeetatX.OA=aa
ndOB=b.
  ~ ~
 
 
 
F
ORMARKI
NGSCHEMESI
NBOX0724351706 Pa
ge|22
Ma
the
mat
ic
s121/
1,2
 
 
 
 
 
 

a
) Expr
essthefoll
owi
ngvect
orsinte
rmsofaandb.
i
) A
~B (
1ma r
k)
i
i
) AM (
1ma r
k)
~
i
i
i)ON
~ (
1 mar
k)
b
) Youhavealsobeengi
vent
ha tOX=hONa n
dAX=k AM.Ex
pres
sOXi
ntwowaysa
ndh e
ncefi
ndtheval
uesofhan
dk .
~ ~ ~ ~ ~ (5mark
s)
c
) ShowthatA, XandMarecolli
nea
r. (2mark
s)
19.T
hedi
agr
amb elo
ws howsahis
togramrep
res
ent
ingmar
ksob
tai
nedi
naMat
hst
estb
yaF o
rmo necl
assofKa
ngemahig
hschool
.
 
 
 
 
 
 
 
 
 
 
 
 
 
 

 
 
  a ) De velopaf reque nc
yd i
str
ibut
iont a
bleforthed at
a. (4ma rk
s)
b) Stateth emo d alfr
equency. (1ma rk
)
c) Esti
ma teth eme a nusinganassume dme ano f33. (5ma rk
s)
 
20.Johnbou ght3b r
an dsofteaA,Ba ndC. Thec ostpri
ceofthebrandsweresh.
25,sh.
30a ndsh.
45p e
rkil
ogra
mr e
s p
ect
ively.Hemix
ed
thebr
and sinth era t
ioof5:2:1r especti
vel
y.Aftersel
li
ngthemix t
ureh
ema deaprof
itof20%.
  a ) Ho wmu chp ro f
itdi
dh ema keperk i
logr
amo f
themi xt
ure. (4ma rk
s)
b) Afteron ey ear,t
h ecostpri
ceo feachbrandwa sincr
easedby12%.
i) F o
rh owmu chd idhesellonekil
o gr
amo fthemi xt
uretomake20%p rof
it
. (
3ma rks
)
ii
) Wh a two uldh avebeenh i
spercen t
ageprofi
tifhesoldonekil
o gr
amo ft
hemixt
ureatsh.
40.25. (
3ma rks
)
 
 21.
Thre
eb oats;X, Ya ndZa r
esuchthatYi s500kmo nab ear
ingof030ofr
omX.Bo atZis750kmf r
omYa ndonab ear
ingo f140of
rom
Y.
a) Dr a
was k
e tc
ho ftheposi
ti
onso fthethr
eeb oats.
b) Us i
ngy ourske t
cho rother
wise,calcul
at
e:
i) thed is
tanc eo fZfromX
ii
)t heb earingo fXf ro
mZ
ii
i
)t hes hortestdist
a ncefr
omXt othed i
rec
trou t
eb et
weenYa ndZ .
c) Bo atSi ssighted900k md ueEa stofY.Calcul
a t
ethedist
anceofSfromZ.
 
t23/ 2
22.Thefi
gureb el
ows howsav el
oci
ty-t
imeg ra
pho fjo ur
neyofac ar.Thecarst
art
sfr
omr estandacce
ler
ate
sa 4m/s f
ortseco
nds
unti
li
tsis22m/ s.
 
 
 
 
 

F
ORMARKI
NGSCHEMESI
NBOX0724351706 Pa
ge|23
Ma
the
mat
ic
s121/
1,2
 
 
 
 
 
 
Br
a k
e sarea pp
liedb r
ingi
n gituniformlytor est.Thetotal
jour
neyi s847ml ong.
  a )t h evalueoft,thea ccel
erat
iont i
me (2mar
ks)
b)t h ed i
sta
ncet r
a v
e l
le
dd uringthef irs
tts econ d
s. (2mar
ks)
c)t h evalueofx ,th
ed ecel
e r
ati
o ntime (4mar
ks)
d)t h erateofd ec
e l
erat
ion (2mar
ks)
23.Thetransfor
ma t
ionTa ndSa rede f
ine dasfo ll
ows:
T=r e f
lect
iono nthel i
n ey=x
S=p osit
ivequarterturna bou ttheo r
igi
n
  T hepo i
ntsA(3, 7),B(3,4)an dC(-1, 4)aret heve r
ti
cesoftr
ia
ng l
eABCwh oseimag
eunderTi
str
ia
ngleA1B1C1
11 1
  a ) Ont heg r
idp r
o v
ided,drawt ri
ang leABCa nditsimageABC u n dert
rans
for
mati
onT. (3mar
ks)
b) St at
et heco o
rdinatesofA1B1C1 (1mar
k)
c) Dr awt ri
ang eA11B11C11t
l hei mag eo fA1B1C1u ndertr
ansf
orma ti
o nS. (2mar
ks)
d) St at
et heco o
rdinatesofA11B11C11 (1mar
k)
e)F indas i
nglema tri
xtransforma t
iont hatma psABCo ntoA11B11C11 (3mar
ks)
1
24.Astr
a i
g htl
ineL 1w hi
c hpassesthro ughA( -1,2)h asag r
adi
ento f- /2.Anot
herst
rai
ghtl
ineL
2pas
sesthr
o ughth
epoi
ntsB(
2,-
3)an
d
C(
4, 6).De t
ermine:
  a )t h eequ a
ti
o nofL 1intheformy=mx+c (2mar
ks)
b)t h eequ a
ti
o nofL 2intheformy=mx+c (2mar
ks)
c)t h ecoo r
dinate
so fthepo in
to fintersecti
ono fl
i
nesL1andL 3 (2mar
ks)
d)t h eequ a
ti
o nofal in
et hroughca ndp aral
leltoL1inthefo
rmy=mx+c (2mar
ks)
e)t h eequ a
ti
o nofth eli
nep assi
n gth r
oug hD( -2,-
2)andp e
rpen di
cu l
artoL2int
heform
y=mx+c (2mar
ks)

F
ORMARKI
NGSCHEMESI
NBOX0724351706 Pa
ge|24
Ma
the
mat
ic
s121/
1,2
KANGEMAF ORM4ENDOFT ERM2EXAM
121/ 2
MAT HEMAT ICS
Pap er2
.MOCKSKCSEPREDI CT I
ONS.
Time :2½Ho urs
 
    SECT IOI NI:( 50MARKS)
1. Givent h aty=2p-re xp resspi nte r
mso fya n dr
p+3r (3ma rks)
2. Aq u antityvi sp artlyc on stanta ndp ar
tlyva ri
e sasu .Ifu=1wh env=12a ndu=12wh env=23.
Findt hev a l
u eo fvwh enu=5. (3ma rks)
3. Solvef orxi nth ee q uation
log10(3x+2)-1=l og10(x-4) (3ma rks)
4. Solvet hef o l
lowi ngi ne aqu ali
tya ndstateth ein t
egralvalue s (3marks)
2x-1<7+x<3x+2
5. Thec irclewh o sea rel eng t
hi s2. 2ms ubten dsa na ngleo f60oa tthec en
tre.Calcula
tet heareaoftheminorse gmentoft
hec irc
le.Tak
e
22
= / 7 (4marks)
6. Theb asea ndp erp end i
cu l
a rh eightofat ri
a nglea reme a sureda s8.2cma nd6. 3cmr esp
ecti
vel
y.Ca lc
u l
at
et hepercent
a geerro
rin
calculatingt hea re ac orrec tto3d .p. (3ma rks)
7. Twot apsAa ndBc a nfillawa terbathin8mi nute
sa nd10mi nutesrespect
ive
ly.Ta pAi sopenedfor2mi nu t
esthenclosed.TapBi s
lat
ero p en edf oro n emi n utet henc l
osed. Ho wl ongwi llthet wotap sta
keru nni
n gtogethertofi
llt
here maini
ngp ar
tofthewa te
rb at
h?
(
3ma r
ks
8. Ch ordsABa n dCDo fac i
rc l
eme etatXa ss howni nt hef i
g ur
eb elow.IfAB=8c m,BX=5c ma ndCX=6c m,cal
c u
lat
eth elengt
ho f
cho rdCDc orrec tto1d .p. (2marks)
 
 
 
 
 
 
 
 9.Exp r
e sst hefo l
lo win gins urdfo rma nds i
mp l
ifyb yrati
on ali
zingthed enominatorgivi
n gyoura n
swerint h
eformo fa+b c
(3marks)
1
cos60o-s i
n450
5 3
10.i) Ex p anda nds imp lify(1- 3x) uptoth etermi nx (2marks)
5
ii
) He nc eu sey o ure xpa nsiont oesti
ma te( 0.97)c or
rec tto4d .
p . (2ma rks)
11.Inat ran sforma tion ,a no bjectwi tha na reao f10c m2 i sma ppe dontoa nima gewh osea r
eais60c m2.Gi ve
nt hatthema tri
xo f
tra
n sforma tioni s findthev al
u eo fy. (3ma rks)
 
12.Givent h atOA=3i +2j -4ka ndOB=4i +5j -2k. PdividesABi ntherati
o1:2. Determinetheposit
ionvectorofpintermso fi
, ja
nd
k. (3marks)
13.Solvet hee quation :
2c os4x=- 1f o r0<x<180o (3ma rks)
14.Then umb ers8, xa nd2a ret hef i
rstthreeter mso faG. P
i) F indt wop o ssiblev alueso f x (2marks)
ii
)F indt hes u mo fth ef i
rstfivetermso f t
h eG. Pif t
hec o mmo nr ati
oisne ga
ti
v e. (2ma rks)
15.Wa n j
ikup aysf orac aro nh irep urchasein15mo nthlyin st
almen t
s.Thec ashpriceo fthecarisKsh.
300,000a ndtheinteres
trateis
15%p .
a .Ad epo sito fKs h.75, 000i sma de. Ca lc
u l
ateh ermo nt
hlyr ep
ayme nt
s. (3marks)
16.Poin t
sAa ndBh a vec o ord i
n a
tesa s(1,5)a nd( -3,7)r espe ct
iv
e l
y.IfABi sthed i
ame terofthecir
cle,fi
ndth eequat
iono fthi
sc i
rcl
e.
(3marks)
 
  
SECTIONI I:(50MARKS)
An swe ra nyf i
v eq ue sti
o nsi nth i
sse c
ti
o n. 
17.Josh uah astwoc h il
d renwh o s
ea ged i
fferenc eis5y ears. Twiceth esumo fthei
ra gesisequaltotheageo fthepare
n t
.
a)T ak i
n gYt ob et hea geo fthee lde
rc hild, writ
ea ne xpressi
o nfor
i) t hea g eo fth ey ou nge rc h
ild (1mark) 
ii
)a geo fthep aren t (1ma rk)
b)I n20y earst ime ,thep roducto ft
h ech il
d ren’
sa geswi llbe15t imesthea g
eo fthe i
rparent
.
i) F orma ne qu ati
o ni nya ndh enced etermi nethep resen tpossi
b l
eageso ftheelderch i
ld
. (4ma rks)
ii
)F indt hep resen tp ossiblea gesofth ep arent. (2ma rks)
  i i
i) De termi net hep ossiblea geso fthey oun gc hil
din20y ear
st i
me . (2marks)
18.Thef ollowi ngt ab l
es h owst hed i
s t
ri
butiono f ma rksob t
ain edby50s t
udentsinat est
.
  Ma rks 45-49 50-54 55-59 60-64 65-69 70-74 75-79
No . ofs tuden ts 3 9 13 15 5 4 1
Byu si
n ga na ss ume dme ano f62,calculate:
FORMARKI NGSCHEMESI NBOX0724351706 Page|25
Ma
the
mat
ic
s121/
1,2
a)t h eme an (
5ma r
ks)
b)t h evariance (3mar
ks)
c)t h estanda r
dd eviati
on (2mar
ks)
e2/ 3
 
19.InaMa thema ticstest,
th eprobabi
lityo f3s t
udentsKa mau,Otien
oa ndMwa l
apassi
nga r 3,/ 4a
nd5/6resp
ect
ive
ly.
  a ) Dr awat reed i
agramt orepresen tthi
sin f
ormatio
n . (3ma
rks
)
b) Us eth etreed ia
g r
amt ofindthep robabil
it
ythat:
 
i
) a l
l t
het hrees t
ud e
n t
swi l
lfai
l (2mar
ks)
i
i)a tlea
s ttwos tudentswillpass (3mar
ks)
i
ii)o nlyo nes t
u dentwillpas
s (2mar
ks)
 
20.Ap a
rticl
emo vesinas traig
htlinesu cht ha
ti t
sdis
p l
aceme n
tsme t
resfr
omag i
venp oi
ntis
3 2
s=t -6t +2t+3wh eretistimei ns econd s
.Find:
  a )T hed i
s p
lace mento ftheparti
clea tt=3 (2mar
ks)
b)T heve lo
c i
tyo fthep ar
ti
clewh eret=4 (2mar
ks)
c)T heva lu
eo ftwh enthep ar
tic
lei smo me nta
ril
ya tres
t (
3ma r
ks)
d)T hea cceler
a t
iono fthepart
iclewh ent=4 (3mar
ks)
21.a) Ap l
an eleav e
sa i
rportA(40oS, 36oW)a t1400h r
so nMo ndayandfli
esdueNo r
thtoai
rportB(50oN,36oW).
Calculat
et hed i
stancethep l
an ec oversinkil
ome tre
s.(
T a
k e=22/7andR=6370k m) (
3ma r
ks)
b) Af t
er45mi nute
ss toppageatB, theplanefl
iesdu eEastt
oa i
rportC,di
st
anceof2550n auti
calmil
esfr
omB. Fi
nd:
  i) t h epo si
ti
o no fC (4mar
ks)
i
i)t h elocaltimet hep l
anelandsa tCi f
itsavera
g espeedforthewh ol
ejou
rneyis1200km/hr.
(Take1n au t
icalmile=1.854k m) (3mar
ks)
 
22.Fig
ureb el
o wi sap yramido nar ec
ta ngu l
arb a
se.PQ=16c m, QR=12c ma ndVP=13c m.
 
 
 
 
 
 
 
 

Fi
nd:
a)theleng
thofQS (2ma
rks
)
b)theheig
htofthepy
ramid (2ma
rks
)
c)theangl
eb e
tweenVQa ndth
ebase (2ma
rks
)
d)theangl
eb e
tweenpl
aneVQRa ndth
eb as
e (4ma
rks
)
 
 23.
a) Us
ingarule
randapairofcompa
ssesonl
y,con
s t
ruc
ttr
ia
ngl
eABCinwhic
hAB=9c m,AC=8c
ma nda
ngeBAC=60o
l
(
2marks)
b) Onthesamesi
deofABa sC,dr
awthelocusofapoi
ntPsuc
hthata
ngeAPB=60o
l (
3marks)
c) Are
gioniswit
hint
hetri
angl
eABCs uc
hthatAT>4c mandang
leACT>angl
eBCT
. Sh
o wt
her
egi
onTbysha
din
git.
(
5marks)
24.
 
 
 
 
 
 
 
 
 
 

Thef
i
gur
eabov
eshowstwocir
cl
esABPQa n
dABSRint
ers
ect
in
gatAandB.QART
,PBSa
ndABUa
res
tr
ai
ghtl
i
nes
.Th
eli
neT
SUi
sa
t
ange
ntt
othec
irc
leABSRatS.Gi
vent
hatBPQ=80o,
PBU=115oa
ndBUS=70o. Fi
ndsta
terea
son
sth
efol
lowi
ngang
les
a) BRS (
2marks)
b) BSU (
2marks)
c) STR (
2marks)
d) BAR (
2marks)
e) ARB (
2marks)

F
ORMARKI
NGSCHEMESI
NBOX0724351706 Pa
ge|26
Ma
the
mat
ic
s121/
1,2
MURANGASOUT H
ENDOFT ERMI IEXAMINAT I
ON
KenyaCert
if
i
ca t
eo fSeco
nd a
ryEducat
io
n
121/1
MAT HEMATICS
FORM4
.MOCKSKCSEPREDI CTI
ONS.
1
TI
ME:2 HOURS
2
SECTIONI(50MARKS)
Answeral
lquesti
on si
nthissect
io
n
1. Duri
n gaf
ootbal
l ma t
ch,
sh.1,462,
800wasr
eal
i
zedf
roms
tad
iume
ntr
anc
efe
es.
Ift
hee
ntr
anc
efe
ewa
ssh
.80perp
erso
n,c
alc
ula
te
howma nyfa
n spaidtowatchma t
ch. (
2ma r
ks)

1 3
2. Thr
eewomenKez
ia,
MaryandAli
ceco
ntri
but
edmone
ytobuyaveh
icl
e.Kezi
acont
ri
bute
do fthet
otal
amount
,Mar
ycont
ri
bute
do f
3 8
th
eremai
ni
ngamounta
ndAli
cecont
ri
butedt
here
sto
fthemone
y.Thedif
fe
renc
einthecon
tri
buti
onbet
wee
nMa r
yandAl
icewass
h.
60,
000.
Calc
ula
teth
ecostp
ric
eofth
ev e
hicl
e. (
3ma rks
)

3. Simp l
ywit
houtus
ingmat
hemat
ic
alt
abl
esorc
alc
ula
tor
s. (
3ma
rks
)
3 41
2744× 2
64
4. Us elo
gar
it
h mtos
olvet
abl
est
oeval
uat
e; (
4ma
rks
)

( )
1
45.
3×0.
00697 3

0.
534

5. Achor
dABofl
eng
th13c
msu
bte
ndsa
nan
gleo
f67°a
tth
eci
rc
umf
ere
nceo
fac
ir
cl
ece
ntr
eO.
Fin
dth
era
diu
so ft
heci
rc
let
oth
e
ne
are
stwh
olen
umber
. (3mark
s)

6. Ameta
lweig
hs83.16gra
mswhento
tal
l
yimme
rse
dinac
yli
nde
rofr
adi
us3c
m,t
heh
eig
hto
fwa
terl
eve
lint
hecy
li
nderr
is
esb
y0.
49
cm.
Calcu
lat
eden
sityo
ftheme
tal
. (3mar
ks)

7. T
hef
i
gur
ebe
lowi
sas
emi
cyl
i
nde
rof
len
gth18c
man
dra
diu
s3.
5ass
hown
.

Drawalabelednetofthesol
i
d. (
2ma rks
)
8. Asale
sma nispai
das ala
ryofsh
.25,
000.Heget
sacommissi
onof3.
5%o nsal
esabov
esh. 100,
000a ndanadd
it
ional1.
5%ift
he
sa
lesexc
e edsh.500,000.I
nthemont
hofDecembe
rthesa
lesmanre
cei
vedear
nin
g st
ota
li
ngs h.61,
500.Cal
cul
ateth
et o
tal
sal
esh
e
madethatmo nt
h. (
3ma rks
)
9. Thesumo fint
eri
oran gl
eofare
gul
arpol
ygo
nistwe
lvet
imesit
sext
eri
ora
n g
le.
Findth
en umb e
ro fs
ide
softh
ep ol
ygonandnamet
he
pol
ygon. (
4ma rks
)

10.I
nab
ota
nic
ale
xpe
ri
men
tth
ele
ngt
hof
60l
eav
eso
fac
ert
ai
nty
peo
fat
reewe
reme
asu
redc
orr
ectt
oth
ene
are
st0.
1cm.
Le
ngt
h(cm) 3.
0-3.
4 3.
5-3.
9 4.
0-4.
4 4.
5-4.9 5.
0-5.4 5.
5-5.9 6.
0-6.
4 6.
5-6.
9 7.
0-7.
4
No.o
fle
ave
s 1 4 9 14 12 10 6 3 1

a) Sta
tethemod a
lclas
s . (1mar
k)
b) Cal
culat
etheme di
anh ei
gh t
. (3ma
rks
)
11.Giv
enthata=5i +4j,b=3i -2jandc=7i
+10j.Fi
ndt
hescal
ar
sma ndns u
cht
hatma+nb=c (3marks
)
12.Si
mp l
if
ytheexpress
ion
2
x -
9
2 (3mar
ks)
6-11x+3x
2 2
13.Acirc
leOh astheequati
onx+y=4. T
hear
eaoft
heci
rcl
einf
irs
tquadr
anti
sdi
vi
dedi
nt
o5ver
ti
cals
tr
ipsea
cho
fwi
dt
h0.4cm.Th
e
ta
blebel
ows howsthev a
lueso fxa dyf
n ort
hefi
rs
tquad
ran
toftheci
rc
le.

x 0 0.
4 0.
8 1.
2 1.
6 2.
0
y 2.
0 1.
96 1.
83 1.
6 1.
2 0

F
ORMARKI
NGSCHEMESI
NBOX0724351706 Pa
ge|27
Ma
the
mat
ic
s121/
1,2

Useth etrap
ezo i
dalru
let oes t
imateareao f t
hecircl
e. (
3ma rks)
14.Ab usiness
ma nb ought100t extbooksa nd80p ensforsh.25,600.Ifshehadboughtt
wiceasmanyt e
xtbooksandh al
fasman ypens
shewo u l
dha vepai
ds h.7,400l ess.Findth ecostofonetextbookando nepen. (3ma r
ks)
15.Thear eaoccupiedbyal akeo nama pis28c m2.Ifthesc
a l
eo ft
h emapis5cmt o1km
Calculat
ethea ctua
lareao fthel a
keo nth eg r
oundi nhect
ares. (
3ma rks)
16.Wa weruh adwa lke
dtwot hi
rdsa waya cr
o ssab ri
dgewh enh esawa napproac
hingt
rai
n60ma way.Heranb ackonlytorea
cht he
bri
dgea tthes ameti
mea sthet ra
inwa smo vi
nga t25m/ sandWa wer
ur anat10m/s,fi
ndthele
ngthoftheb ri
dge.
(3ma r
ks)
SECTI ONB( 50Ma rk
s )
Answe ranyfivequest
ion sinth i
ssection.
17.Mes h
a ckandKe lvi
nco ntri
bu t
eds hs.60,000a nds h.90,
000r espect
ive
lyinordert
ostar
tbusi
ness.
T he
ye mployedama nagerand
agree
dt op ayh i
ms h.4,500p e rmon t
hf romt hep r
ofi
tma deeachy e
ar.Theyal
soagree
dthat20%o fth
ep ro
fitmadee ac
hyea rwoul
d
bep utbackintothebu si
n esswh il
ethere stwou l
db eshar
edb etweenthemintherat
ioofth
eiri
nit
ialcont
ri
bu t
ion.Duri
ngthef i
rs
t
yeartheyma deap rof
itofs h.365,000. Ca l
c u
lat
e:-
a)T hema nage
r’sannualsalaryforthaty e
ar (1ma r
k)
b)T hemo neyputb ac
kin t
ob usinessthatyea r. (
2mar ks
)
c)T heb usines
sn etprof
itforthaty ea
r. (
2mar ks
)
d) Howmu c
he achp ar
tnerreceivedthatyea r. (
3mar ks
)
e)T heca pit
alf
o rthefol
lowingy ea r
. (2marks)

18.a
)Gi
vent
hatA= ( ) ( )
2 3
4 4
,B=
x 1
2 3
an
dth
atABi
sas
ing
ula
rma
tri
x,s
olv
efrx
o . (
4ma
rks
)

b) Acl
othesd e
ale
rs ol
d3s hi
rtsand2trouse
rsfors
h.840and4s hir
tsa n
d5t r
ouse
r sf
orsh
.1,680.Fo
rmama tr
ixequ
ati
ontorep
res
ent
t
heaboveinfor
ma t
ion.Hencefin
dt h
ec o
stof1shir
tandt
h ecostof1trous
er. (6marks
)
19.Usi
ngrulerandcomp a
sson l
y:-
a) Cons
tru
c ttr
ia
ngleABCs uchthatAB=5c m,AC=6c ma ndangleBAC=67. 5°.Meas
ureBC. (
4ma rks
)
b) Onthesamed i
agram,constru
ctac i
rc
lewh i
chpas
sest
hroughtheverti
cal
softh
et r
ia
ngl
eABC. Measu
reth
era d
ius
.
(3mar
ks)
c) Cal
cul
atetheareaofthepartt
hatisout
sidethet
ri
angl
ebutwith
inth eci
rc
le. (
3ma rks
)
20.Th
efigur
eb el
owi saplotofla
ndABCDwh ereAD =84c m,BC=75. 8cm,∠ABD =27°, DABisari
ghtang
ledtr
ian
glean
d
∠DCB=82°.
C

820

D 75.
8cm

84m

2
A B

L
eavin
gy oura nswerto1d ecimalpla
cedeter
mine;
a)ThelengthAB. (
2marks)
b)Thelengtho fDB. (
2marks)
c)Thelengtho fDC. (
4mark
s)
d)Thea r
eao ftheplotinhe c
tares
. (
2ma r
ks)
21.T
heL 1andL 2l i
neswh osee q
u a
tion
sa r
eax+b y+c=0a n dqy=k int
ersec
tatP(8,8).
a)Iftheli
nesma k
ea ngl
eso f135°a nd45°respec
tiv
elywihxa
t x i
s.St
atethe
irgra
dient
. (
2ma rk
s)
b)Findtheeq uati
ono ft
hel i
nesabo v
e (
4mark
s)
c)Findtheeq uati
ono fanotherl
ineL3p as
sin
gthroughthe(1,2)a ndpara
ll
eltol
ineL1(4marks)
22.(a
)PʹQʹ
Rʹ i
st heimageo fPQRwi thver
ti
cesP(4,2)
Q( 1,
1)an dR( 5,
1)underarota
ti
on.IfPʹi
s(-6,
-4),
Qʹ(-3,-
3)andRʹ(-
7,-3)
,
d
rawtri
ang l
e sPQRa ndPʹQʹ
Rʹ. (2marks
)
(b
)Pʹʹ
Qʹʹ
Rʹʹ
i
sth eimag eofPQRu nderatr
ansl
ati
onT= .
-()
1
3
Wr i
ted o
wnt hecoordi
nat
esofPʹ
ʹQʹʹ

ʹa
nddrawit
.(3marks
)

F
ORMARKI
NGSCHEMESI
NBOX0724351706 Pa
ge|28
Ma
the
mat
ic
s121/
1,2

(
c) Pʹ
ʹ
ʹ
Qʹʹ
ʹ
Rʹʹ
ʹwi
thv
ert
i
ces
(-6,
2),
(-3,
1)a
nd(
-7,
1)r
esp
ect
iv
elyi
sth
eima
geo
fPQRu
nde
rtr
ans
for
mati
onT
.Dr
awPʹ
ʹ
ʹ
Qʹʹ
ʹ
Rʹʹ
ʹa
ndd
esc
ri
be
itf
ul
ly
. (3marks
)

(
d) P4Q4R4i
sth
eima
geo
fPQRu
ndra+900r
e ota
ti
ona
bou
tth
eor
igi
n.St
at
eth
eco
ord
ina
te fP4Q4R4.
so (
2mk
s)

23.T
hef
i
gurebe
lowi
sasol
idwhi
chi
spa
rtl
yac
yli
nder
,fr
ust
uman
dh e
mis
pher
e.Theto
pradi
usi
s3cmwhi
let
her
adi
uso
fth
e
h
emi
sph
erepa
rti
s10c
m. T
hel
eng
thoft
hef
rus
tumis14cmwhi
let
hehei
ghto
fthecyl
i
nderi
s14cm.

Dete
rmine:
-
a)T hecur
veds u
rfa
c eareao fth
ec yl
indri
calpa
rtof
thesol
i
d. (
2ma
rks
)
b) Curv
edsurf
a c
eo ftheh emispher
icalpar
t. (
2ma
rks
)
c)T hecur
veds u
rfa
c eareao fth
ef r
ust
u mp ar
toft
hesol
id
. (4ma
rks
)
d)Thet
otalsur
faceare aofthesoli
d. (2mar
ks)
24.Theequat
io
no fan orma ltoacurveat(0,3)i
s2y+x=6.
a)F i
ndtheequati
ono fitstangenti
nth efor
my=mx+c (
2ma
rks
)
b)T hegra
dientf
unc t
iono fthecurveat(0,3)i
s2- 2x
i
. F i
ndtheequati
ono fthec urv
e (
3mar
ks)
i
i.F i
ndthetur
n i
ngp oi
n to ft
hecu r
ve (
3ma
rks
)
c) Sket
chthecurve
. (
2mar
ks)

F
ORMARKI
NGSCHEMESI
NBOX0724351706 Pa
ge|29
Ma
the
mat
ic
s121/
1,2
MURANGASOUT H
121/2
MAT HEMATICS
PAPER2
FORM4
.MOCKSKCSEPREDI CTI
ONS.
SECTIONI(50Marks)
Answeral
lques
ti
onsi
nthi
ssect
io
n.
1. Thesurf
aceofs
pher
ic
alba
lli
sin
crea
sedb
y21%a
ft
erp
res
sur
ewa
spu
mpe
din
.

a
)Fi
ndt
hec
ir
cumf
er
enc
eoft
heo
rig
ina
lba
lli
fon
ewi
thi
ncr
eas
edp
res
sur
eha
sac
ir
cumf
ere
nceo
f55c
m. (
2ma
rks
)

b) Calc
ulat
ethep
erc
entag
einvol
umeofth
eb a
ll
. (
2ma
rks
)
43
2. Simpli
f
y l
eavi
ngyoura
nsweri
nthefo
rmab+c (
2ma
rks
)
3-
23
3. Inthefi
gur
ebel
ow,AOBisasect
orwhos
eareai
s7.
2cm2s
ubt
end
inga
nan
gle2.
5ra
dia
nsa
tth
ece
ntr
eO.

Cal
cul
ate:
a) Cal
cul
atethera
d i
usofth
esect
or. (2ma
rks
)
b)T h
elengthofthearcAB. (2ma
rks
)
4. Oisth
ec ent
eroftheci
rcl
e.Gi
venABi
spa
ral
l
elt
oOCa
nda
ngl
eABO =30°,
fi
nda
ngl
eACO. (
3ma
rks
)

1 5
5. Ama nc ant i
llo fap i
e ceofla
n din4d ayswh i
lehi
swifecanti
llo ft
h esamep ie
ceo fl
andin3d a
ys.Howma nyday
swillt
hetwo
2 8
ta
ketot i
ll t
hewh olepieceo f
thelandwh enwo rki
ngtogeth
er? (3marks
)
1
6. Amo ther’sageisthes quarerootofherd a
u ght
ers
,Afte
rfiv
eyears
,themo th
erwillbe4 t i
me sasoldasthedau g
hte
r.Fi
ndthe
2
daughter
’sp rese
nta ge. (4marks
)
7. Sol
veyf o rtheequatio
n ;
l
og (3y+2) -
1=log (y-4) (3marks
)

()
10 10
36 4
8. a) Ex pan d1-x u ptothetermi nx. (1ma r
k)
2
6
b) He nc eevaluat
et hefir
stfourt
e r
mso ftheexpans
iontoevla
ute(1.03)t ofoursi
g ni
fi
cantfi
gures (2marks
)
2 2
9. Writ
ed ownt heequati
ono fac i
rcl
ec e
n tr
e(0,
2)andraduis3unit
s,l
e a
v i
ngy ouranswerintheformax +b y +c x+dy+e=0.
(3ma rks)
10.PQRi sat ri
angleofarea9c m2. I
fPQi sthefix
edb a
seo ft
hetra
ingl
ea nd6c mlongd rawitandd e
scri
bethelocusofpoi
ntR.
(3marks
)
11.Aclassha s18b oysa nd12g ir
ls.Threepup i
l
sarechosenatrando
mf romt hec l
ass
. Whatistheprobabi
li
tyofchoosi
ngallt
hegir
ls.
(2marks
)
12.Afarmerwi shestop urchasesomeg oatsandc o
ws.Hec anbuyatmo st3e it
herani
ma ls
.Ona verga
eac owa ndag oa
tcostsh.
12,000
andsh s
. 4,000r e
specti
v el
y.Heh asshs.240000t ospendandthen umbe rofcowss hou
ldb eatmosttwi
c eth
en umberofgoat
s.He
mustb uymo r
ethanf i
vec ows.Lis
ta l
l t
heinequal
it
iestorep
rese
ntthea boveinf
orma t
ion. (
4ma rks)
d y 2 1
13.Giv
ent hat =2x+3a ndthaty=3wh enx=0, f
indtheval
ueofywh e
nx= . (3ma r
ks)
d x 3

F
ORMARKI
NGSCHEMESI
NBOX0724351706 Pa
ge|30
Ma
the
mat
ic
s121/
1,2
14.T
heposi
ti
ono ft
wotownsXan
dYareg
ive
ntothedeg
reeasX(
45°N,10°E)a
ndY( 70°E)
45°N,
F
ind
:-
i
. Thedif
fe
renceinl
ongi
tude
. (
1ma
rk)
i
i.Thedis
tanc
eb et
weenth
etwot
own
sinnaut
ic
almil
est
othenea
restwho
lenumbe
r. (
2ma
rks
)

15.Solv
et heeq uati
ons n(
i 5x-20)°=-0.8660f orvlaueso fxbetween0°an d90° (2marks
)
16.Vect
orAO =6i –2j+3ka ndOB= -2i+3j –k.Ap ointCd i
vi
desABi nther at
ioo f3:1.Fi
n dACintermso fi,j
andk .
(3marks
)
SECTIONI I(50MARKS)
Answero nlyF I
VEq u est
ionsint hi
ss ect
ion .
17.Hali
mad eposit
edsh .500,000i naf inanciali
nsti
tutionwh i
chp ai
dsimp l
eint
erestatar ateof6%p e
rannum. Attheendof2y ear
s,
Shewith dr
e wtheni nvest
edt hemo neyins har
es.T heval
u eofthesharesdepr
ec i
ateda tt
h erat
eo f4%d u
rin
gt hefir
styearoft
he
i
n v
estme nt
. Inthen ext3y ears,thevalueo ft
her ateofthesharesappreci
at
eda ttherateo f6%e ver
yfourmo nths.
a) Calc
ulateth eamo untthatHa li
mai nvestedinsh ar
e s. (3ma r
ks)
b) Calc
ulateth eval
ueo fHa li
ma ’sshares.
i
. Att hee ndo ftheyear. (2marks
)
i
i. Atthee ndo fthefourthyea rtothen ear
e stshi
ll
ings. (3ma r
ks)
c) Calc
ulateHa l
i
ma ’
sg ainfromt hes har
esa sap ercentage. (2ma r
ks)
18.Anarit
h me t
icp r
ogressiono f41t ermsiss uchthatthes u
mo fthefi
rs
tfivete
rmsi s560a ndthesu mofthef
ivel a
stt
ermsi s-
250.Fi
nd:
-
a)T hefi
rstterma ndth ec ommo nd i
ff
erence (5marks
)
b)T helastterm. (2ma r
ks
c)T hesumo fp r
ogression. (3ma r
ks)
19.a)Comp let
et hetableb el
ow, givin
gv aluescorrecttod e
cimal p
laces
. (2marks
)
x° 0 30 60 90 120 150 180
2si
nx ° 0 1 2
1- cosx° 0.5 1 2
b) Ontheg r
idp rov
idedu si
n gthes ames caleanda xesd r
awt hegraphsofy=2s inx°a n dy+c o
sx °=1. (5ma r
ks)
Taketh escal
e: 2c mf or30°o nthex–a xis
2c mf or1u ni
to nthey–a xi
s
c) Usetheg raphin( b)ab o
vet o:-
i
. So lv
et heeq uati
on2s i
nx °=1-c osx° (1ma r
k)
i
i. Deter
mi neth erangeo fvaluesforwh ich2s i
nx °≥1-cosx° (1mark)
i
ii
. Sta
teth eamp li
tudeo f2sinx ° (1ma r
k)
20.Thediagramb el
owr epresen t
sap yramids ta
ndingo nr e
ctangula
rb as
eABCO. Visth ev e
rtexoft
h epyra
mida ndVA=VB=VC=
VD =26c m. Ma n dNa ret hemi dpoi
n t
so fBCa n dACr espect
ive
ly.AB=24c ma n dBC=18c m.

Calc
ulat
e:-
a)T heli
neo ft
h el
e n
g t
hAC (
2mark
s)
b)T helen
gtho fproj
e c
tiono ftheVAo nthepl
aneABCD. (
1mark
)
c)T heangl
eb etweenlineVAa ndtheplaneABCD. (
2mar
ks)
d)T hever
ti
ca l
h ei
ghto fthep yra
mid. (
2mark
s)
e)T hesi
zeo ft
h eangleb etweenthepla
n e
sVBCa ndABCD. (
3mar
ks)
21.Thre
eq uant
iti
esR, Sa ndTa r
esuchthatRvar
iesdire
ctl
yasSa n
dinver
sel
yast
hes
quar
eofT
.
a) Give
nthatR=480wh enS=150a ndT=5,writeanequat
io
nc onn
ect
ingR,Sa
ndT. (
4mar
ks
b)i .F
indthevalueofRwh enS=360a ndT=1.5. (
2mark
s)
ii
.Fi
ndthep ercenta
g ec hangeinRifSincr
eas
e sandTd e
crea
sesby20%. (
4mark
s)
22.(a)
Theaccel
erati
ono fap art
ic
letse
co n
d sa
ft
erp as
si
ngafixe
dp oi
nti
sgive
nbya=3t-
3.Gi
vent
hatt
hev
elo
cit
yoft
hepar
ti
cl
ewhent
=2s econdsis5m/ s,find:-
i
. I t
svelo
cit
ywh ent=4s e conds (
3mar
ks)
i
i.I t
sdisp
laceme n
ta tthistime. (
3mark
s)
3
(b
)Findtheareab ounde db ythegr
aphx=9y- y andtheyaxi
s. (
4mar
ks)
F
ORMARKI
NGSCHEMESI
NBOX0724351706 Pa
ge|31
Ma
the
mat
ic
s121/
1,2
23.Thetabl
ebelowshoesthema rk
sscore
db yfortyform4s tu
d e
ntsinama themati
calt
est
.
Ma rks 10-19 20- 29 30- 39 40- 49 50- 59 60- 69 70- 79 80- 89 90-99
F r
equency 2 4 5 7 10 6 3 2 1
a) Usi
nga nassumedme a
ns coreof55,c
alcul
ateth emeano fth
ed ata. (
3mark
s)
b) Cal
culat
ethelowerquart
ile
. (
2marks
)
c) Ontheg r
idprovi
ded,dr
awt hecumula
ti
vefrequ encycurv
etor e
p re
sentt
hea b
o v
ed i
st
ri
buti
on. (
3mark
s)
d)F r
omt hegraph,es
ti
ma t
e:-
i
. T he5thdec
il
e. (
1ma r
k)
Rangeofmarksofthemid d
le80%o ft
hestude nts. (
1mark)
24.ABCDi squadr
il
ate
ralwithvert
ic
esA( -
7,2)B( - 2,
1)C(2,8)D( -7,7).
A’B’C’D’ist
heimageofABCDu
nde
ratra
nsf
ormat
io
nma t
ri
x

(i
)Fi
n dth
ec oor
dinat
esofA’ B’C’D’.
( ) -1 0
0 1
(
2marks
)
(i
i
)Ont hegri
dp r
ovide
dd rawABCDa ndA’ B’C’ D’ (
2mark
s)
a)(
(
i
)F
i
i
i
n
)Ont
dA”B”C”D”t
hesameg r
i
h
dd
ei
ra
ma geofth
eo
wA”B”C”D”
fA’B’ C’ D’underthetr
an sf
ormat
io ( )
nma t
ri
x
0 -
-
1 0
1
(
2mk
(
1ma
s
r
)
k
)
b)( i
)Fi
n dth
es i
ngl
ema tr
ixthatmapsA”B”C”D”o ntoABCD. (
2mark
s)

(
ii
)De
scr
ibet
het
ran
sfo
rma
ti
onf
ul
ly
. (
1ma
rk)

KANDARAF
ORMF
OURENDOFT
ERMT
WO.
EXAMI
NAT
ION

121/1
MATHEMAT
ICS
PAPER1
TI
ME:2½HOURS

SECTI
ONI(50MARKS)
Answe
ral
lt
heque
sti
onsi
nthi
sse
cti
on.

1. Wi
th
outu
sin
gac
alc
ula
tor
,ev
alu
ate
, ,
lea
vin
gyo
ura
nswe
rasaf
rac
ti
oni
nit
ssi
mpl
estf
orm.
(3mk
s)

2. Fi
ndtheequa
ti
ono fap
erpe
ndic
ularbi
sec
torofal
i
neABift
hecoo
rdi
nat
esofAandBa
re(
-4,
-2)and(
6,2)re
spec
ti
vel
y.
(
3mks)
3. Uses
qu a
res
,squar
esro
otsandre
cipr
ocalt
abl
esonl
ytoe
val
uat
eth
efol
lo
win
gg i
vi
ngyo
ura
n s
we rc
orr
ectt
o2d ec
imalp
lac
es.
(
4mks)
1 3
+
√31.
47 8. 542

4. Fin
dthein
tegra
l v
aluesofxfo
rwh i
ch: (
3mks)
-6<3x+3
14–3x>2
5. PipeAempti
esafull
tankin8h o
u r
s,p
ipeBemp t
ie
sin6hour
sandpi
peCemp
ti
esi
n3h
our
s.I
fal
lt
het
hre
epi
p e
sareopenf
or30
minut
esandthenpipeAc l
ose
d, Howlongwil
li
ttaket
oemptyaf
ul
ltan
k. (
3mk s
)
6. Withou
tusi
ngac al
culat
orormathemat
ica
lta
ble,
eval
uat
e. (3mks)

7. Si
mpl
i
fyt
hee
xpr
ess
ion
: (
3mk
s)

8. T
wotra
insT1andT2,t
rav
eli
ngi
noppo
sit
edi
rec
ti
o n
so np
ara
ll
elt
rac
ksar
eju
stbeg
inni
ngtopa
sson
ea not
her
.Trai
nT1is72met
res
l
ongan
distra
vel
ingat108km/
hr.
T2i
s78me t
resLonga
ndtra
vel
i
ngat72k
m/hr.Fi
ndth
eti
meinseco
ndsthetwota
ketocompl
etel
y
p
assone anothe
r. (4mks)
9. I
nthef
igur
ebell
owedandEAaret
ange
ntst
othecir
cl
eatDandA.DCispa
ral
l
eltoABand <BDC=510.Ca l
cul
atet
heval
ue
o
fx. (3mks)

F
ORMARKI
NGSCHEMESI
NBOX0724351706 Pa
ge|32
Ma
the
mat
ic
s121/
1,2

10.Twosi
mila
rc o
n t
ai
ner
shavebas
ear
eas25cm2a
n d324cm2r
esp
ect
iv
ely
.Ca
lcu
lat
eth
eCa
pac
it
yoft
hel
ar
gerc
on t
ai
nerco
rre
ctt
oon
e
3
dec
imalpl
aceift
hecap
acit
yoft
hesmal
le
roneis8cm (3mks)
11.Sol
vefo
rmi n;
Log5–2+( 2m+10)=log(m-
4) (4mks)
a
12.Gi
vent
hat =1 , Findthev a
lueso faandb . (3mks
)
b
13.AKe nyanb ankbu ysandse l
lsfore
ignc ur
renci
e susi
ngtheratesshownb el
ow.
Buying Sel
ling
(Ksh
) (Ks
h )
1Eu r
o86. 25 86.97
100J apa nes
eYe n66.51 67.26
AJ a
pa nesetrave
lingfr
o mF rancearri
vesinKen yawith5000Eu ros
,wh i
chh econv
ert
stoKenyashi
l
lingsatt
h ebank.Wh i
l
einKeny
a
hespen tat o
talofKsh.289,850a ndthenc onv
e rt
edtheremainin
gKe nyashi
lli
ngst
oJ a
panes
eYe natth
eb ank.Cal
cul
atetheamoun
t
ofJapan eseYenthather ecei
ved. (3mk s)
14.Allprimen umbersbet
we ent e
na ndtwen t
ya r
ea rr
angedindescendi
n gorde
rtoforman u
mb e
r.
a) Wr it
ed ownth enumb er (1mk)
b) St atethetotalval
ueo fthethi
rdd i
giti
nt hen u
mb erfo
rmedi n(a)above
. (1mark)
15.Ana i
rcraftfl
yin
ga t800k no t
sleavesair
portA(60N, 12E)at9aml o
ca lt
imeforai
rpo
rtB(60N,88W)alongthep a
ral
l
elof l
at
it
ude
.Fi
nd
thel
oc altimeatBwh enthep l
anelan d
s. (3mks)
16.Thediag ramb e
lowr ep
r e
sentsar i
ghtpyramido nas quar
eb a
seo fs
ide3c m. T
hesla
nt
Heighto fthepyramidis4c m.

a
) Dr
awaneto
fthepyr
amid (
2ma
rks
)
b
) Ont
hen
etdrawn,
me a
sur
eth
ehe
igh
tof
atr
ia
ngu
larf
acef
romt
het
opo
fth
ePy
rami
d (
1ma
rk)

SECTI
ON11( 50MARKS)
Answe
ro nl
yFIVEques
ti
onsf
romthi
sse
c t
ion.
17.Theve
rti
cesofatr
ia
ngl
eareA(
2,5)
,B(
4,3)andC(
2,3)
.Hr
epr
ese
ntsah
alft
urnr
ota
ti
ona
bou
tth
epo
int(
0,2)
.
a) Drawtri
angl
esABCandA1,
B1,
C1underH (
4mk
s)

b
) Tr
epr
ese
ntsar
ef
le
cti
oni
nth
eli
nex=0a
ndKr
epr
ese
ntsat
ran
sla
ti
on F
indt
hec
oor
din
ate
sofA”,
B”,
C”o
fA’
,B’
,
C’u
nde
r

TK.Henced r
awt ri
an gl
eA”B” ,
C” . (4mks)
c)F i
ndthea re
ao ftri
an gl
eA”,B”,C’ (
2mk s
)
18.Bis102kmo ntheb eari
ngo f112°f r
omA. Cis94k mo nt
h ebea
ri
ngof062°f r
omBDi s073°fro
mAa nd336°f r
omC.
(a)Us i
ngas c a
leo f1cmtor eprese
nt20km,drawad i
agra
mt oshowtheposi
ti
onsofA,B,CandD. (
6mk s
)
b) Us i
ngy ourd i
ag r
am,determine;
i
) T heb ea
ringo fBf r
omDa n dAfromC. (2mks)
i
i)T hed is
tan c
eACa ndBD (2mks)
19.Ag r
oupo fpeopleplannedtoc ontri
but
eequal
lytowardsawa t
erpr
oje
ctwhichneede
dk shs. 2,000,
000t ocomplete
.Howev
er,
40me mbe r
so ftheg r
oupwi t
hd re
wf r
omt h
ep roj
ect
.Asar esu
lt
, eac
ho ft
heremainin
gme mbersweretocontr
ibu
tek s
h2500
more.
(a)Findtheo ri
gina l
numb ero fmembersi
nth egro
up. (5mks)
(b)Fortyfi
vep ercentofth
ev alueo ft
heproj
ectwasfundedbyConst
it
uencyDevel
opme ntFun
d(CDF )
.Calcu
lat
ethe
a
mo untthatwo uldbema d ebyeachoftheremai
ningme mber
softhegroup. (3mks)
(c)Me mb er
sc ontrib
uti
onwe rei nte
rmsoflabourpro
videdandmoneycontr
ibut
ed.Th
er at
iooft
hev al
ueoflab
o u
rtoth e
F
ORMARKI
NGSCHEMESI
NBOX0724351706 Pa
ge|33
Ma
the
mat
ic
s121/
1,2
moneycontr
ibut
edwa s6:19,cal
cul
atethetota
l a
mo untofmoneyc ont
ri
but
edbythemember
s (
2mks
)
20.Abusl
eftKi
sumua t9:30a .
mt owardsNair
o biata
na ver
agespeedof81k m/hr.
Ama ta
tul
eftNa
ir
obi
at10.
10a.mata
nave
rage
s
peedof72km/hr.Thed i
stanc
eb et
we e
nKis umuandNa ir
obiis360km.
(
a)De ter
mine
(
i)T h
et i
metakenb e
forethet wovehi
cle
sme t
. (
3mks
)
(
ii
)T h
ed is
tanc
eb et
we enthetwov ehi
cl
es40mi nu
tesaft
erme et
ing. (
2mks
)
b) Acarlef
tKi
sumut owa rd
sNa ir
obiat9.50a .
ma tanavera
g es
pe e
do f90km/hr.
Deter
mine
i
) T h
et i
mewh enthec arcaughtupwiththeb us
. (
3mks
)
i
i)T h
ed is
tanc
eo fNa i
robif
romt hepl
acewh e
rethecarcaughtupwiththebus
. (
2mks
)
21.I
nthef
igurebel
owEi sthemi dp oi
ntofBC. AD:DC3: 2andFi sthemee t
in
gpointo
fBDa n
dAE.

Ifab=bandAC=c ,f
i
nd: (
2mk
)
i
) BD
i
i) AE (
2mk
s)
b)I fBF=tBDa ndAF=nAE.Fi
ndthev
alu
eoftan
dn . (
5mk
s)
c) Stat
eth
er a
ti
oofBDtoBF. (
1mk
)
22.Asur
v e
yorr
ecord
edthemea
sur
ement
sofafi
el
dinaf
iel
dboo
kus
ingl
i
nesAB260ma
ssh
ownb
elo
w.

B
130 R40
70 Q10
50 P20
S50 10
A
a) Sk etc
hthema pofthefie
ld. (4mks )
b)F i
n dtheare
ao fth
efiel
dinh e
ctar
es. (6mks )
23.Atr
adersoldanart
icl
eatsh.4800a ft
erallowi
ngh iscusto
me ra12%d i
scountonthemarkedpr
ic
eo ft
hearti
cl
e .
Ins odoi
nghemade
apr
ofi
to f45%.
a) Calcul
ate
(
i)T hema rke
dp r
iceoftheart
icle. (
3ma r
ks)
(
ii
)t h
ep r
ic
eatwh i
chthetraderh a
db oughtthearti
cl
e (
2marks
)
b) Ifthetrad
erhads ol
dthesa mea r
ti
clewithoutgiv
ingad is
count
.Calc
ulatet
h epe
rcen
tagepr
ofi
thewo ul
dh avema d
e.
(3ma rks)
c)T oc l
earhi
sstock,t
hetraderd e
cidedtosellt
herema ini
ngart
icl
esatalossof12.
5%c al
cul
at
ethep r
ic
eatwh ichhesol
d
eacha r
ti
cl
e. (
2mks)
3 2
24.Thedi
stancesSme tr
esfr
omaf i
x e
dp oi
nt, co
veredbyap a
rti
cl
eaft
ertsecondsi
sg i
venbytheequ
atio
n,S=t –6t +9t+5
a) Ca l
c ul
at
ethegradi
enttothec urv
ea tt=0.5s eco
n ds. (
3mks)
b) De ter
minethevalu
eso fSa tth
ema ximuma ndmi nimumt ur
nin
gpo i
ntsoft
h ecur
ve. (4mks )
3
c) Ont hespac
ep rov
ided,St
retchthecu r
vesofS=t –6t+9t+5 (3mk s)

F
ORMARKI
NGSCHEMESI
NBOX0724351706 Pa
ge|34
Ma
the
mat
ic
s121/
1,2
KANDARAF
ORMFOURENDOFT
ERMT
WO.
EXAMI
NAT
ION
121/2
MATHEMAT
ICS
PAPER2
TI
ME:2½HOURS
SECT
ION50MARKS
Ans
werall
quest
io
nsinthi
sse
cti
oni
nthes
pac
esp
rov
ide
d.
1. Uselo
gari
thmtoeval
uate (
4mk
s)
358.
32x 98232
0.
693.5

2. Ma
kett
hes
ubj
ecto
fth
efo
rmu
la (
3mk
s)

3h(
3t
-h
x=
t
3.Si
mpl
i
fya
ndr
ati
onal
iz
eth
eex
pre
ssi
ong
ivi
ngy
oura
nswe
rth
efo
r fa+b
mo c,
whe
rea
,ba
ndca
rec
ons
tan
ts
(
3mk
s)
11 5
-
7-√3 7+√3
4. Themeasuremen t
so ftheradi
usan dheightofacyl
inde
raregiv
enas8c mand9.5cmre
spect
iv
ely
.Cal
cul
at
etheperc
ent
agee
rrori
n
t
hevolumeo fthec y
linder. (3mks)
6 3
5. (a
)Ex pand( 1–2x ) i
na s
cendin
gp owe r
so fxupox
t . (
2mk
s)
6
(b
)He nc eeval
u a
te( 1.
02)t o4d .p. (
2mks)
6. (a
)Co ns t
ructABCs uchthatAB=6c m,BC=5c ma ndAC=4c m. (
1mk )
(
b) Cons
tructthelocuso fpoin
tQi nsi
dethetriangl
eABCa bov
esuchthatthear
eaofABQ is6cm2.(2mks
)
7. Apil
otl
ea ve
sp ointT(600S,100W)a ndfl
iesdueEastf
orad i
st
anceof960n mtopoi
ntU.Dete
rminet
hepos
it
ionofU.
(
3mk
s)
2 2
8. Fi
ndtheradiusandc e nt
reoftheci
rcl
ewh oseequa
ti
onis 3x +3y –12x+18y–9=0. (
3mk
s)
9. Quant
it
yQp art
lyvariesasquanti
tyRa ndp ar
tl
yvari
esin
vers
elyasth
es qua
reofR.Gi
ventha
tQ=3wh enR=1a ndQ=5wh enR=
½
(i
)F in
da ne quat
ionc onnecti
ngQa n dR. (
3mk
s)

(i
i)Findtheva
lueofQwhenR=3/2 (
1mk
)
10.Dete
rminethei
nteg
ral
val
uesofxt
hatsa
ti
sfythef
ol
lo
wingi
nequal
it
ie
s. (
3mk
s)
3x
-18–3x2x-½20–
2
11.Thelong
estsi
deofari
ghtang
ledt
ri
angl
eis25cmandthet
wos i
desco
nta
ini
ngt
heri
ghta
ngl
ear
exc
madyc
n m.I
foneoft
he
shor
tersi
desex
ceedst
heothe
rby17cm,obta
intwoe q
uat
ion
sinxandya n
dso
lvet
hem. (4mk
s)
12. I
fOA=2i
–4ka
ndOB=-
2i+j
–k.
Fin
dth
ema
gni
tu
deo
fAB. (
2mk
s)
      
13.I
nth
efi
g.b
elo
wli
neCD=4c
m,l
i
neDT=8c
man
dAB=6c
m.ATa
ndCTa
res
tr
ai
ghtl
i
nesme
eti
nga
tpo
intT
.

Fin
dthev a
lueo fy. (2mks)
14.Inwhatrat
ios houl
dg r
adePo fteacost
ingsh
.450p e
rkgbemix
edwit
hg r
adeQo
fte
aco
sti
ngs
h.350pe
rkgs
oth
atapro
fi
tof10%
is ma debys ell
i
ngthemi x
tureatsh.451perkg? (3mk
s)
15 Findth
ev al
u esof between00a nd1800suchth
at2co
s3=3s in3 (3mks)
x 2y
+1 2x y
-3
16.Solv
e8=4 a nd27 =9 g iv
ingyourans
wersasa
nexac
tfr
act
ion
. (
3mk s
)

F
ORMARKI
NGSCHEMESI
NBOX0724351706 Pa
ge|35
Ma
the
mat
ic
s121/
1,2

SECTIONB( 50MARKS)
Answeranyfivequesti
o nsi nthissect
ioninthespac e
sprovi
ded.
17.Inc
o metaxisc ha
rgedo na n nualinc
omea ttheratesshownbelo
w.
Taxabl
eannua li
ncome Rateshpe rk£
1 - 2300 2
2301- 4600 3
4601- 6900 5
6901- 9200 7
9201- 11,500 9
11501a ndab ove 10
per
so n
alrel
iefksh.1056p ermo n t
h.
Ins
u r
ancereli
efksh.480p ermo nt
h.
Mr.On ya
ngoe ar
nsab asicsa l
arysh.13800p ermo nt
h.Inaddi
ti
ontoh
issal
ar
yheg
etah
ousea
ll
owan
ceo
fk s
h.8000permont
h
andme di
calall
owa nceo fsh. 5000p ermo n
th.
Cal
c ul
at
e;
(b
)Mr .Onyango’
st ax
ableinc ome (
2mks)
(i
i
)Mr .Onya ngo’
sn ettaxp ermo nthinKenyas h
ill
ings
. (
5mk s
)
(i
i
i)Ca l
cula
teMr .On yan go’snetmo nthl
ysalar
yinKe nyashi
l
ling
s.(
3mks)
18.Thetabl
eb e
lo wshowst h ema rks c
oredbys t
udentsinama t
hsexam.
Cl
a ss 30–39 40–49 50–59 60–69 70–79 80–89
No. ofst
ude nts 3 17 27 23 8 2

a
) Drawac u
mu l
at
iv
efrequ
enc
ycurv
e (
3mk
s)
b
) Useyo
u rgra
phtod e
ter
min
e
i)Th
eme dian (
1mk
)
i
i
)T h
eq u
a r
ti
ledev
iat
io
n (
1mk
)
i
i
i)Th
en umberofst
udent
swhosc
oreda
bov
e67. (
1mk
)

(c
)Us ea nassumedme ano f64.5tocalc
u l
a t
eth est
an da
rdd e
via
ti
ono ft
heaboved a
ta. (
4mks)
19.Ana ri
thmeti
cp r
o gr
essi
onh ast
h efi
rstter
ma sxa ndthecommo ndif
fe
renceasd,
(a
)Wr i
ted ownintermso fxandd ,
the3rd,9than d25thte
rmso ft
hep r
ogres
si
on . (1mk)
r
d t h
(b
)Thep rogre
ssionisincr
easi
nga ndthe3, 9a nd25thtermsfor
mst hef
irs
tthre
eCo ns
ecu
ti
v et
ermso
fag
eomet
ri
cse
rie
s.Ift
hesu
mo f
the7thandtwicethe6th t
ermo fth
ea ri
thme ti
cprogressi
onis78,cal
cul
ate:
(i
)T hefirs
tterma n
dt hecommo nd i
ff
erenceo ftheA.P (
6mks)
(i
i
)Thes umo fthefi
rstnineter
mso ftheA.P (
3mks)
20.T
hefigureb e
lows howsar i
ghtpyrami
dwi thas qua r
eblockati
tsbase.
Thes i
desoft
hebasear
e40cmandth
eh ei
ghto ft
hebas
eis
30cm. Mi sthemid-poin
tofQRs ucht haPM=29c m.

Calcul
ate
:
(a
)Thever
ti
calhei
ghtofver
texPf
romp l
aneABCD. (3mks)
(b
)Theangl
ebetweenpla
nesPQRa ndPST. (
2mk s
(c
)Thel
engthofthepro
ject
io
no f
theli
neRPont h
ep l
aneQRST (
3mk s
)
(d
)Theangl
ebetweenpla
nesQRCDa ndABCD (
2mks)
21.T
hemanagerofahotel
hasenoughmoneytobuyatot
alof120cra
tesofs
oftdr
ink
so ft
wob r
andsnovi
daandal
varo
.Then
umberof
n
ovi
dacrat
eshastobeles
sthant
wicethenumberofal
var
ocrat
es.T
h e
remustbeatmost90cra
teso
fnovi
dabutatl
eas
t15cr
ate
s
F
ORMARKI
NGSCHEMESI
NBOX0724351706 Pa
ge|36
Ma
the
mat
ic
s121/
1,2
butnotmo retha
n60c ra
tesofal
var
o .
(a
)L etxb ethenumb erofnovidacr
ate
sa ndybethenumberofa
lva
rocra
tes
(i
) Represe
n tt
heinfo
rma ti
onu s
ingi
neq u
ali
ti
es. (
3mks)
(i
i
)Plo
tth egraphofthea b
o v
einequal
it
ies
. (
3mks)
(b
)Thep r
o f
itfr
omac rateofnovi
d ai
s75a ndtha
tofal
var
ois50.Fi
ndthenumberofcr
ate
sofeac
hty
peth a
theshoul
dbuytomake
maximump r
ofi
t. (2mks)
(c
)Du et oinfl
ati
onthep r
ofi
tonac r
ateofnovid
aredu
cedby20%a ndtha
tonac r
ateofAlv
aroi
ncr
eas
e dby80%.Fi
ndthedi
ff
ere
ncei
n
t
hema x
imu mprof
itthatcanbema debythehot
elasres
ulto
fthes
echanges
. (2mks)
22.Gi
v e
nt haty=2s i
n2xa ndy=3c os(x+45o)
(a
)Comp let
et h
etabl
eb elo
w. (
2mks)
X 00 200 400 600 800 1000 1200 1400 1600 1800
2s
inx 0 1.
97 0.
68 -0.
68 -1.
73 -1.
28 0.00
3c
o x
s(+450) 2.
12 1.
27 -
0.78 -2.
46 -2.
72 -2.12

(b
)Usethedatatodra
wt h
egrap
hsofy=2s in2xa
ndy=3c os(x+45o)f
or0ox180o.
Onthesamea x
es. (
4mk
s)
(c
)Stat
etheamp l
it
udeandpe
ri
odo f
eachcurve
. (
2mk
s)
(d
)Uset
heg r
aphtosolvet
heequa
ti
on2s i
n2x–3cos(x+450)=0for00x1800 (
2mks)
23.(
a)Give
nth a
tBCi sat
ange
nttotheci
rc
leandt
hatan
gleABC=1100andangeCBD=500.
l Cal
cul
at
egi
vi
ngr
eas
ons
E

50o C
F B
(
i) AngleAED (2mks)
(
ii)Angl
eBAD (2mk
s)
(
iii
)Angl
eDCB (2mk
s)
(b
)Giventh
atBD=3c mandDC=5cm.Fi
ndthear
eaoft
ri
ang
leBDC. (
4mks
)
24.T
h ewat
ersuppl
yinat
ownde
pen
d se
nti
re
lyontwowat
erpumps
.Aan
dB.
Thep
rob
abi
l
it
yofp
umpAf
ai
li
ngi
s0.
1andt
hepr
obabi
l
it
y
ofpumpBfail
i
ngis0.
2.

[
a]Dr
awat
reed
iag
ramt
ore
pre
sen
tth
isi
nf
orma
ti
on (
2mk
s)

(
b)Ca
lcu
lat
eth
epr
oba
bil
i
tyt
hat
;

(
i) Bo
thp
ump
sar
ewo
rki
ng (
2mk
s)

(
ii
)Th
erei
snowa
teri
nth
eto
wn (
2mk
s)

(
ii
i
)On
lyo
nep
umpi
swo
rki
ng (
2mk
s)

(
iv
)Th
erei
sso
mewa
teri
nth
eto
wn (
2mk
s)

F
ORMARKI
NGSCHEMESI
NBOX0724351706 Pa
ge|37
Ma
the
mat
ic
s121/
1,2
NYERICENTRALSUBCOUNTY
MATHEMAT I
CSALTA
PAPER1
.MOCKSKCSEPREDICTI
ONS/.
,.
TI
ME:2½HOURS
1.Wit
houtusi
ngmath
emat
ic
alt
abl
esorac
alc
ula
tore
val
uat
e,l
eav
ingy
oura
nswe
rinp
owe
rfo
rm.

16 ¾
( ) x 512⅔
81
(
3ma
rks)

2. Wa teri
nap la
sti
crectangulartankislea
k i
nga tarateof2cm3perse
conds.F
ind
(a
)T h
ev olumeofwa t
erthatleaksfro
mt heta nkeveryhour. (
1mark
)
(b
)T heti
met otheneares
tmi nuteitwillt
akethele aka
getofil
lacyl
indr
ic
alcont
ai
nero
frad
ius22cmandhei
ght
35cm( Tak
eπ=3.142) (
3mark
s)
5
3. Iftanx= f i
nds i
n(90–x ).giv
ingy ouranswe ri
n3s .f
.g (
3mark
s)
8
2
4. Simp l
if
y 16x 9 (
3mar
ks)
2
12x +7x12
1 3
5.Theinter
iorangl
esofah ep t
agona re1200,1000,140,2x 3x200,
, 1 xand2x+50.
2 4
Fi
ndt hedif
fer
encebetwee nit
slargestangleandi t
ssma l
l
estang
le. (
3mark
s)
6.Lin
ey =3x+10i nter
sectsli
ne5x+y+6=0a tap oi
ntP.Findt
hec o
ordi
nate
sofP. (
3mark
s)
7.Giventheinequa
lit
ie
sx 5≤3x8<2x3
(
a)So lv
etheinequal
it
ies (
2mar
ks)
(
b)Re pres
entthesol
u t
iono nan umb erl
ine. (
1mark
)
8.Inthefigur
eb e
lowABCi sane qui
later
altri
angleo fs
ide15cm.ArcsAB,BCandACaret
hoseofci
rc
leswi
thc
entr
esC,AandB
respect
ive
ly.Calcul
atethear e
ao ftheshadedr egi
on. (
3mark
s)

9. Threecon s
ecuti
veoddn umbersx,ya ndza ddupto387.Fi
ndthethr
eenu mbers (
3mark
s)
10.IfOP=5i3j andOQ=3i +5j,fi
n dthema gnit
udeofPQtotwosig
n i
f
icantfi
gures
. (
3mark
s)
11.Ac omme rci
albankbuysandsell
sJ apaneseYeninKenyash
il
li
ngsatthe
shownb elow
Buying Sell
ing
Ksh0.8109 Ksh0.8433
Japanesetouri
stwaslef
twithKsh70,000a ttheendofhert
ourt
oKe nya
.I fsheconv
ert
edthemoneytoJap
anes
eYen
th
rou ghtheb an
kh owma nyJ apaneseYend i
dsheget. (
3ma
rks
)
12.Withoutu s
inglogar
it
hmso rcalc
ulatorsta
b l
eseva
lua
te (
3mark
s)
331. 24x0. 0576
60.84
13.Give
nt hatlog4=0. 6021andlog6=0. 7782,wit
houtusi
ngmathemati
calta
blesorc
alc
ula
torev
alu
atelo
g0.096.

(4ma r
ks)
14.Ac arto
ok135mi nute
stocove
radi
st
anceof(
3x6)kma ta
na v
era
gespeedof(
x+24)km/h.Fi
ndtheac
tuald
ist
anc
ethatwa
s
covered. (
3mar
ks)
15.G1(1,-
5)andH1(-
8,8)ar
ethei
mage
sofGa ndHre
spec
ti
vel
yunderat
rans
lat
io
n.Ifth
ecoor
din
ate
sofHis(-
5,6)
,fi
ndthe
coo r
dina
tesofG. (
3ma r
ks)
16.User e
cipr
ocal
a ndsqua
rest
abl
est
oeva
luat
e,t
o4sig
nif
ic
antfi
gur
estheexpre
ssi
on. (
3ma r
ks)

1 + 5. 3182
17.
62
F
ORMARKI
NGSCHEMESI NBOX0724351706 Pa
ge|38
Ma
the
mat
ic
s121/
1,2

Sect
ionIIansweranyfive(50ma rk
s)
17.(
(
a
b
)F
)I
i
n
nJ
dt
an
h
u
ei
ar
nv
yo
ers
eo
facer
f
t
a
( )
6 4
5 3
inyear,aret
ail
ersol
d18p a
nsan d12s u
furi
asatato
talofsh11400.InFebruar
y,hesol
d15
(
2mark
s)

pan
san d9s uf
uri
asatat o
talofsh9150.Usema t
ri
xme thodtofi
ndthesel
li
ngp ri
ceofeac
hite
m. (
4marks)
(c)Fromthes t
artofMOCKSKCSEPREDI CTI
ONStothee ndofDecembert
hatyear,t
hesell
in
gp r
ic
eo fea
chp a
nwa si
ncre
asedb
y
15%and
t
hatofas uf
uri
awa sreduce
db y6%.I nthi
sla
tt
erperio
d ,
hesoldfi
veti
mesasma nyp a
nsandsixti
mesasma ny
su
furi
asash ehadsoldinJanuaryandFebr
uar
y.Us ethematri
xmethodtocalc
ulat
eh i
stot
alsa
lesofpan
sa nd
su
furi
asfromthestar
to fMOCKSKCSEPREDI CTI
ONSt othee ndofDecember.
(4ma
rks)
18.Inthefi
gurebelo
w, Oisthecentreofth
ec i
rc
le.WOUi sas t
rai
ghtl
ine <WXV=480a ndVX=WX

(a)Gi vi
n gar eason( s)ine ac hc ase,f
indth esizesof
(i
) <UXV (
2ma
rks
)
(i
i)<VOW (
2ma
rks
)
(i
ii
)<UOV (
2ma
rks
)
(b)IfUV=8c mf indth ea reao fthec i
rcle. (
4ma
rks
)
2
19.(a)Sketcht heg rapho fy =2x +4x9f orthev alues-3≤x≤1. (
2ma r
ks)
2
(b)Usemi d– ordinat
esr u l
ewi t
h4s t
ri
p stoe s
tima t
eth eareaoftheregionb ounde dbyy=2x +4x9t hexaxi
s
andt helinex=- 3a n dx=1. (
3ma
rks
)
(c)Calculatethea reao fth es amer egi
onu s i
ngt heintegrat
ionme t
h od. (
3ma r
ks)
(d)Calculatethep ercen t
a gee rrorwh enmi do rdi
natesruleisusedtoe st
imateth ea r
ea. (
2ma r
ks)
3 2
20.Thedisplaceme nt,sme tre sof amo vi
ngp arti
clefromp o i
ntOa f
tertsecondsi sgivenb y S=t 5t +3t+10.
(a)Findswh ent=2 (
2ma
rks
)
(b)De t
ermi ne
(i
)T heve l
ocityo fthep articl
ea tt=5s ec onds (
3ma
rks
)
(i
i)Thev alueo ftwh ent h ep arti
cleismo me ntari
lyatrest. (
3ma r
ks)
(c)Findt hetimewh ent h eve loci
tyo ft
h eparti
c l
eisma ximum. (
2ma r
ks)
21.(a)Solve 2x+1=1
(4marks)
6 2x
(
b )Thewi dtho fKwa mb o ka’so ff
iceis3.1ml essth anitslengt
h.T heareao ftheflooris33m2.
(i
)F i
ndt hep erimetero fth eo ff
ice’
sfloor. (
4ma r
ks)
(i
i)Kwa mb okal aysar ectan gularca r
petinh ero f
fic
ele av
ingas paceof15m2o ff
loo rspa
cearoundit
.Gi vent
hatthe
widtho fthec arpetish alfleng t
ho fthec arpetfi
ndt heleng
tho fthecarpet. (
2ma r
ks)
22.A(1,
3)B( 5,5)C( 4,3)an dD( 4,1)arethev e
rti
ce sofq uadri
la
tera
l ABCD.
(a)Ont heg ridd ra
wq uad ri
lateralABCD (
2ma r
ks)
(b)Qu adri
lateralABCD i
st heima geo fABCDu nderap osi
ti
veq uar
terturna b
o uttheori
gin.Onth esamegri
dd r
aw
ABCD ands ta
tei t
sc o ordinat
es. (
3ma
rks
)
(c)ABCDi sma ppedo ntoq uadril
ateralA
BC
D byarefl
ec t
iononth exax i
s.Ont hesameg ri
dd r
awA
BC
D
ands t
atei t
sc oo rdinates. (
3ma r
ks)
(d)Un derar efl
e ct
iono nl inePQq u adr
ilat
eralAB
CDi
sma p p
edb acko ntoABCD.Ony ourgr
ap hdrawt
hemi nor
l
inePQa n dg i
veitse q uation. (
2ma
rks
)
23.Thefi
gu r
eb e l
owr e
p r
ese ntsa16c mh i
ghs oli
df rust
umwi t
hab aseradiusof 8. 4cma ndato pra
diusof14cm.
Thefrust
u mi sma deo fwa xwh osed ens
ityis0.82g /cm3.
22
(a)T aki
n gπ= f ind
7
(i
)T hev ol
u meo fthefru st
u m (
5ma
rks
)
(
ii
)T hema sso fth
ef ru st
u mi nk i
logramsc orrectto2d .
p. (
2ma r
ks)
(
b )L ucyme l
tsd ownt h efrus tuma ndre ca
s t
sitintoas oli
dc ubeofs i
dehc m.I f15%o fth
ewa xwa slo
stinmelt
ing
processfindh (
3ma
rks
)

F
ORMARKI
NGSCHEMESI
NBOX0724351706 Pa
ge|39
Ma
the
mat
ic
s121/
1,2
24.Thefol
lowingtabl
eshowstheagesi
nmo nt
hsofsomech
il
dre
nthatweret
ake
ntoAf
yacl
in
ici
nace
rta
inweek.
Age(months) 0≤x<10 10≤x<15 15≤x<25 25≤x<45 45≤x<75
fr
equency 24 32 48 66 30
(a)Esti
ma t
e( i
)theme anage (
4ma
rks
)
(i
i
)theme di
anage
(2mark
s)
(b)Stat
et h
emo dal
clas
s (
1ma
rk)
(c)Ont hegridprov
ideddrawahis
togr
amtorep
rese
ntt
hedis
tr
ibut
io
n. (
4mk
s)
NYERICENT RALSUBCOUNT Y
121/2
MAT HEMAT ICSAL TA
PAPER2
.MOCKSKCSEPREDI CT I
ONS/.,.
TIME:2½HOURS
Sect
ion1Ans werAll
TheQue s
ti
ons(50mark
s)
1. Solv
efori ntheequat
ion.
2
6Cos-Si
n-4=0inth
er a
nge00 1800 (
3Ma
rks
)
2 2
2. Th
eequa
ti
onofac
ir
cl
eisx–8x+y +12y+16=0

Dete
rmineth
ecoordi
nate
softhece
ntr
eo f
theci
rcl
eandit
srad
ius
. (
3Mar
ks)
3. Aq u
anti
tyPispar
tl
yco n
sta
ntandpart
lyv
ari
esasthes
quareofQwh
enQ=2,P=40a
ndwh
enQ=3,P=65.Det
ermi
net
hev
alu
e
ofPwhenQ=4. (
4Mark
s)
dy
4. Thegra
dien
tfun
cti
ono facur
veisgi
venby =2x–4
dx

Det
ermi
ne;
(
a)t
heequa
ti
onoft
hec
urv
egi
vent
hec
urv
epa
sse
sth
rou
ghp
oin
t(0,
3) (
2Ma
rks
)

(
b)t
hec
oor
din
ate
soft
het
urn
ingp
oin
tof
thec
urv
e. (
1Ma
rk)

5. So
lvef
orxi
nth
eeq
uat
io
nlo
g(x–1)=l
og12–l
og(
x–2) (
3Ma
rks
)

6. At
ri
ang
lewh
osea
reai
s6.
5cm²i
sma
ppe
don
toat
ri
ang
lewh
osea
reai
s13c
m²b
yama
tri
x
. F
indt
hep
oss
ibl
eva
lue
sof
. (
4

ma
rks
)
7. Gi
vent
hat f
i
ndt
hev
alu
eso
fP,
Qan
dR. (
4ma
rks
)

2
tv-
w
8. Ma
kett
hes
ubj
ecto
fth
efo
rmu
la.
V= (
3marks
)
m
9. ApointTdi
vi
desalineABi nt
erna
ll
yinthera
tio5:2.Give
nthatAis(
4,10)andBis(11,
3).
Fi
ndthecoord
inat
esofT. (
3marks
)
10.Ashopkee
permixescof
feecost
ingsh.160perkgwi t
hanothert
ypewhic
hc o
stssh.
240p e
rkg.F
indt
her
ati
oinwh
ichthet
wotyp
es
s
houldbemixedsothati
fo neoft
hemi x
tur
esissoldatSh.220,apr
ofi
tof10%isreal
iz
ed.(3mar
ks)
11.Atwod i
gi
tnumb e
rismad ebycombini
nganyo ft
h et
wod i
git
s1,3,
5,7and9a tr
andom.
a) Makeanarra
yo fpos
sibl
ec o
mb i
nati
ons (
2mar
ks)

b)F i
ndtheprob
abil
i
tyth
a tt
h enumberfor
medispri
me. (1mark
)
5
12.(
a)Exp
a n
da nds i
mpli
fythebinomina
lexpr
essi
on(2+2y) inasce
ndin
gp owe
rsofy (1mark
)
5
(
b) Uset
heexpansi
onuptoth ef
ourthte
rmtoe v
alua
te(2.
02) cor
rec
tto4d e
cima
lpl
ace
s. (
2mar
ks)
13.Amand ep
osit
edKsh.48,000inab a
nkthatcompound
edatthera
teofr%p.as
emi
-an
nual
l
yfo
rtwoy
ear
sandreal
i
zedKs
h.54,
968.
Fi
ndrgivi
ngyourans
we rcorr
ectto4s.f
. {3mark
s}

F
ORMARKI
NGSCHEMESI
NBOX0724351706 Pa
ge|40
Ma
the
mat
ic
s121/
1,2
14.Ari
ghtci
rc
ula
rcy
li
nderi
stobema
desot
hatth
esu
mofit
sra
diu
sandi
tsh
eig
hti
s6c
m.F
indth
emax
imu
mp oss
ibl
evol
umeoft
he
cy
li
nder
. (
3mar
ks)
15.Thef
igur
ebel
owshowsaci
rc
lewi
ths
eca
ntsABEa
ndCDE,i
fAB=4cmandBE=6c
mand DE=4c
m.fi
ndt
helengt
hofCD.
(
2mar
ks)

16.T
apsAan
dBc anfi
l
latanki
n4and9hou
rsres
pect
iv
ely
.Bo
tht
apsa
ret
urn
edo
nfo
r2h
our
saf
te
rwh
icht
apAisc
los
ed.
Fin
dho
w
l
ongt
apBta
kestofi
l
lth
er e
mai
ni
ngpa
rtoft
hetan
k. (
3mark
s)

Sect
ionIIanswerfivequest
ionsonl
y( 50marks
)
17.Theposi
ti
ono ftwotown sar
eA( 30ºS,20ºW)andB(30ºS,80º
E)fi
nd
(a
)t hedif
fer
e n
c einlongi
tu
d ebet
we e
nt h
etwotowns. (
2mar
ks)
(
b) (i
)th
ed i
sta
n c
eb etweenAa ndBa l
ongp ar
all
elofl
at
it
udei
nk m(t
akera
diu
soft
hee
art
has6370k
man
d )
.

(2mar
ks)
(ii
) i
nnm. (2marks)
(
c)Findl
oca
lti
mei
nto
wnBwhe
niti
s1.
45p
mi nt
ownA. (4mar
ks)
18.I
nth
efigu
rebe
lowMan
dNar
epoi
nt
sonOBandBAre
spe
cti
vel
ysu
cht
hatOM:
MB=2:
3an
dBN:
NA=2:
1.ONa n
dAMi nt
ers
ectatx
.

⃗ ⃗
a
)Gi
venthatOA=aa ndOB=be xpr
essthef
oll
owinginter
mso faandb. (
4ma
rks
)

AB

AM

ON
⃗ ⃗
b
) Bytaki
ngOX=hONa ndAX=kAM, wher
eha n
dka rescal
ar
s .F
indt
woexpr
ess
ion
sfo
rOXi
nte
rmso
faa
ndb.Hen
ce
det
erminet
heconst
antshandk (
5ma
rks
)
c
)F i
ndtherati
oinwhic
hxd i
vi
desAM. (
1ma
rk)
19 T
hef
ig
urebelowshowsacuboi
dABCDEF GH.(
Give
nt h
a tAB12cm,BC=8cma
ndBE=4cm)

(
a)St at
et hepro
jecti
ono fAGo nt
hep l
aneEFGH. (
lma r
k)
(
b)De terminetheanglebetweenth
elineAGa ndth
ep l
aneEFGH. (
3mk s
)
(
c)Gi venthatMi sthemid-poi
ntofFG,det
erminet
heang l
ebet
we e
ntheplane
sAMDa ndABCD. (
3ma r
ks)
(
d)F indthevolumeo fthecuboi
d (
1mar
k)
20.Ama na
ge rwis
hestoh i
retwotypesofmachi
ne.Hec on
sider
sthefol
lo
wingfact
s:
Ma ch
ineA Machi
neB
Fl
oorspace 2m3 3m3
No.ofme nrequi
redtoo per
ate 4 3
Hehasama x
imumo f24m2o ffl
oorspac
ea n
dama x
imumo f36me navai
l
able
.Inad
dit
ionhei
snota
ll
owedt
oh i
remoremac
hin
eso
f
t
ypeBth anoftypeA.
(a
)Ifheh i
resxma chi
n e
so ftyp
eAa ndyma chi
nesofty
peB, wri
tedownallt
heineq
ual
it
ie
stha
tsat
is
fyt
heab
ovecondi
ti
ons.
(
4mar
ks)
(b
)Re p
resentthein
eq ua
li
tiesonthegri
dandshadetheunwantedre
gion. (
4ma r
ks)
F
ORMARKI
NGSCHEMESI
NBOX0724351706 Pa
ge|41
Ma
the
mat
ic
s121/
1,2
(c
)I ft
heprof
itfr
omma chi
neAissh.4perhoura
ndthatf
romu
singBisshs
.8perh
our
.Wha
tnu
mbero
fma
chi
neofe
achtypes
hou
ld
themanag
e rchoo
setogiv
ethemaxi
mu mprof
it
? (
2ma
rks
)
21.T
hefreq
uencydist
ri
but
ionofmark
sof110stude
ntsi
sgiv
eni
ntheta
b l
ebel
ow.
Ma r
ks 1- 10 11-
20 21- 30 31- 40 41- 50 51- 60 61-70 71-80 81-90 91- 100
f 2 5 9 12 25 37 11 6 2 1

(
a)Ont heg r
idprov
ided,dr
a wanOg ivetoil
lus
trat
ethed
ata (
4mar
ks)
(
b)Fr
omy ourg r
aphest
imate
(
i) T
hemed i
anma rk (
1ma r
k)
(
ii
) T
heint
erquart
il
erange (
2mar
ks)
(
ii
i
) T
hepassma r
k,if68%ofthes t
uden t
saretopass (
3mar
ks)
22.(
a)T hefi
rsttermofanArit
h met
icProg r
essi
on(AP)i
s2.Thesumofth
efi
rst8te
rmsoft
heAPi
s156.
(
i)F i
ndthec ommond i
ffe
renceo ft
heAP. (
2ma
rks
)
(
ii
)Giv e
nthatthesumo fthefi
rstnter
mso ftheAPis416,fi
ndn. (
3mar
ks)
b
)T hethi
rd,fi
ft
handeighthter
mso f
a not
herAPformthefi
rs
tth
reecon
secut
iv
ete
rmsofaGe
ometr
icPro
gres
si
on
(
GP).Ifthecommond if
fere
nc eoft
h eAPis3, f
in
d
(i
) T hefi
rstte
rmo ftheGP; (
3ma
rks
)

(
ii
) T hes
umo fth
ef i
rs
t9termsoft
heGP,
to4si
gni
fi
cantf
ig
ure
s. (
2ma
rks
)
23.T
het
axat
io
nra t
e,f
or2007wereasi
nthet
abl
ebel
ow.
Income(Ksh.P.M) Taxr
ate(%)
1-2000 5
2001–4000 10
4001-6500 15
Above6500 20

I
nc e
rta
inmo nt
hin2007, Mu ta
ip a
idat a
xo fKsh.3000aft
erh
av i
ngbeengiv
enapers
onal
rel
ie
fKsh.1056p e
rmo nt
h.
(a
)Ho wmu chtaxp e
rmo nthc ouldh ehavepaidifhedidn o
tgett
axreli
ef
. (2marks
)
(b
)Ca l
cula
tetheh i
stot
altaxableincomei nKsh.P.m. (6marks
)
(c
)Gi ve
nthathi
stotalt
axa
b l
ea ll
o wancea mounttoKsh.4520P.m.
Ca l
cul
atehi
smo n
thl
ybas
ics
ala
ryinKenyashi
ll
ing
s.
(2marks
)
24.(
a)Usi
ngar ul
erandpai
ro fco mpas s
eso nl
y,const
ructt
ri
angl
eABCi nwhic
hAB=9cm,BC=8.
5cmand∠BAC=60o.
(3marks
)
(b)Ont hesamesid
eo fABa sC:
(i
) De t
ermi
nethelocuso fap ointPsu c
hth t∠APB=60o.
a (3marks
)
(i
i)Co ns
tru
cttheloc
uso fRs u chthatAR>4cm. (2marks
)
(i
ii
)Dete
rminebyshadi
n gther egionTi n
sidethet
ri
an g
lesuc
hthat∠ACT≥∠BCT . (2marks
)

F
ORMARKI
NGSCHEMESI
NBOX0724351706 Pa
ge|42
Ma
the
mat
ic
s121/
1,2
BUURIEASTST ANDARDS
121/1
MAT HEMATICS
.MOCKSKCSEPREDI CTIONS,.
PAPER1
TI
ME:2½HOURS
SECTIONI(50MARKS) :
Answeral
lt
heq ue
sti
onsint
hi
ssect
io
nint
hes
pac
esp
rov
ide
d.

1. Withoutusingac a
lcula
toreval
uat
e . (3mark
s)
-4(
3+1)-18÷6+5
-
3×-6+-1×11
2. Thedist
an cebe
twe e
nJ ane’
sh o
mea ndh erschools4/5o
i f8km.On edaysherun¼ofth
ewa ya
ndwal
kedtheres
tofth
ejo
urn
ey.
Wh a
td i
stancedidshewa lk
? (
3ma r
ks)
3. Oti
endewo r
ksforaco f
feeproc
essin
gc ompanyasas ale
sma n.Hei spai
donMo n
thl
ybasi
saspe
rag
ree
me ntbel
ow.
a) Ab a
sicpa yofsh.20,000p e
rMo nth.
b) Acommi ssionof2%f orgoodssolduptoama ximu mo f
sh.200,000.
c) Acommi ssionof4%f orgoodssoldoversh.200,000inthatMon t
h.
I
nac ertai
nMo nthh esoldgoodswo r
thysh.600,000.Ca l
cula
tehistot
alpa
yfort
hatMont
h. (
3ma r
ks)
4. Drawthen e
toftheso l
i
ds howninthefi
gu r
ebelow.Me asure
me nt
sa r
eincent
ime
ter
s. (
3ma r
ks)

y
+1 3y
+1
5. So
lvef
orYint
heequat
ion. 8 -2 =48 (
3ma
rks
)
2 2
12x +a
x-6a
6. Si
mpli
f
ytheex
pre
ssi
on; 2 2 (
3ma
rks
)
9x-4a

1
7. Ali
nePwh oseequ
ati
onisy= x+4i spara
ll
elt
oan
othe
rli
neQ.F
indt
heeq
uat
io
nofl
ineQi
nthefo
rmy=mx+cg ive
nthat
3
i
tpas
sesthroug
h Poi
nt(3,6) (
3mark
s)
1
8. Si
mpli
fywith
outusi
ngtabl
esorcal
cul
at
ors
; (
3ma r
ks)
1-sn450
i
0
9. Th
efig
u rebe
lowshowsatri
angl
eABCinwh i
chAB=6cm,BC=11cmanda
ngl
eABC=100.Cal
cul
atet
othed
eci
malpl
ace
sthe
l
engt
ho fAC. (3mar
ks)

10.T
hef
igur
ebe
lows
howsat
ri
ang
ula
r–b
ase
dpy
rami
d.I
fth
eso
li
disma
deu
pofama
ter
ia
lofd
ens
it
y1.
25g
/cm3,c
alc
ula
tet
hema
sso
f
t
hes
oli
d. (
3ma r
ks)

F
ORMARKI
NGSCHEMESI
NBOX0724351706 Pa
ge|43
Ma
the
mat
ic
s121/
1,2

11.Afo
o t
bal
lmat
chb et
weenArse
nalFCandChel
seaF
Cstar
tedat1500hrs
.Itla
ste
dfort
heof
fi
ci
al90min
uteswi
thaha
lft
imeb
rea
kof
15minut
es.TheRef
ere
ea dd
edfi
veext
ramin
utesf
ori
nju
riesandoth
ersto
ppages
.Fin
dth
etimethemat
chend
ed.
(3marks
)
12.Wri
tedownth
ei n
equal
i
tie
sthats
ati
sf
ytheuns
hadedr
egi
onl a
bell
edR. (
3mar
ks)

13.Thewidt
ho farect
angul
a rhal
lofRuir
iGir
lsSec
ondar
ySc h
ooli
s16mlesst
hani
tsl
eng
th.Ca
lcu
lat
ethel
engt
hofth
eh a
lli
fit
sare
ais
132m2.He ncecal
cul
ateitsperi
mete
r. (4mark
s)
14.TownAi s80kmd ueeastoftownB.T ownCisonabeari
ngof2340f
ormto
wnB.Ifto
wnCis100kmfro
mt ownA,bysca
ledr
awing
fi
ndthedis
tanceofto
wnCf romtownB. (
4ma r
ks)
15.a
b
)F i
) Us
ndt
i
n
hei
gma
n
t
v
r
i
er
seo
xmet
ft
h
o
h
d
ema
,sol
v
tr
i
et
x
hesi
( )
mu l
ta
n
7 4
3 2
eou
.
se
quati
ons.
(
1ma
(
2ma
r
k
r
k
)
s)
7x+4y=14
3x+2y=8
16.Useta
b l
esofsquar
erootsa n
dr ec
ipr
ocal
stofi
ndth
evalueofx. (3marks
)

1 3
x= +
15.
36 1.
302

SECT
IONI
I(
50ma
rks
).An
s ro
we nl
yfi
veq
ues
ti
onsi
nth
iss
ect
io
nint
hes
pac
esp
rov
ide
d.

17.a
) Ont heg ri
dprovideddrawthequadr
il
ate
ralPQRSwh oseve
rti
cesar
eP(- 5,
4),Q(-
3,4)
,R(-
5,3)a
ndS(-
4,3)
.
(
1ma
rk)
b
) Ont hes amegriddraw:-
i
) P’Q’R’S’t
h ei
ma geofPQRSu nderaref
le
cti
o nint
heli
ney=0 (
2ma
rks
)
i
i
) P”Q” R”S”theimageo fP’
Q’R’
S’underarot
atio
no f+1800about(
0,0) (
2mark
s)
i
i
i) P”
’Q”’R”’
S”’t
hei mageP”Q”R”S”underanenlar
gementsc
ale-
2,c e
n t
re(4,0) (
2ma
rks
)
c
) Na met hequadri
lat
eral
sthatar
e
i
) Directl
yc ongr
uen t (
1mark
)

F
ORMARKI
NGSCHEMESI
NBOX0724351706 Pa
ge|44
Ma
the
mat
ic
s121/
1,2
i
i
) Op
pos
it
el
yco
ngr
uen
t (
2ma
rks
)

18.T
hefig
ureb
elo
wshowsaf
rus
tr
um.T
het
opa
ndb
ott
omr
adi
iar
e5c
man
d 10c
mre
spe
cti
vel
y,wh
il
eth
eve
rti
cal
hei
ghto
fth
e
f
rus
tr
umis12cm.

F
indt
he:
-

a) Sl an th ei
g hto fthef r
ustum. (3ma r
ks)
b) Cu rveda r eao fthefrustum. (3ma r
ks)
c) Vo l
u meo fth ef r
ustum. (4ma r
ks)
19.Kir
iar
i i
sama r ketce ntr
e600k mf ro mKi sumut own .
Ab ussta rt
sf romKi sumuf orKi ri
a r
i at7.00a ma tana ver
ges p
e edof80k m/ h.At8.30a mac a
rsta
rte
dfromKisumutoKira
riand
mo v
eda tana verages peedo f120k m/ hr
.
i
) T hed i
s t
a nceb u scoveredb e f
o rethec arstart
edmo v
ing. (3ma r
ks)
i
i)T her el
atives pe edforthet wov ehicl
es. (2ma r
ks)
i
ii)T het imet hec aro vert
o okth eb us. (1ma r
k)
i
v) Di sta ncec o veredb ythec arb eforeo vertak
ingt hebus. (2ma r
ks)
v) Di sta ncef romKi r
iaritothec a ratthet imethec arwaso vert
aki
n gth ebu s
. (2ma r
ks)
20.Theh ei
g hto f36s tu dentinac lasswa sreco r
dedt othenearestcent
ime terasfo l
lows:
-
148159 158 163 166 155 155 179 158
161160 157 165 165 175 173 172 178
147168 157 172 165 154 170 157 167
155159 173 171 168 160 172 156 167
a) Ma k
eaf requ enc yd is
tri
b ut
iont ableu singac l
assinter
valof5a ndstarti
n gwiththec l
ass145–149. (2ma r
ks)
b)F romt het ablea b ove
i
) Ca lc ulateth eme anma rk (3ma r
ks)
i
i) Ca lc ulateth eme di
an (3ma r
ks)
d) Drawaf requ enc yp olygonu si
n gt heta b
lein( a)abo v
e. (2ma r
ks)
21.Kibi
ri
c hiaBo ysSe co ndarySc hoo l.I nt
end stob uyac ert
ai
nn u mbero fcha i
rsF o
rKsh.16,200.Thesupp
li
eragre
edtoo f
fe
rad i
scou
nt
ofKsh. 60p erc hairWh i
chwi lle nab l
ethes c h
o oltoget3ch ai
rsmo re.
Taki
ngya sth eo ri
g inal
lyintende dn umb ero fchairs:
-
a) Writ
ea ne xpre ssi
oni nt er
mso fyf o r
i
) Or ig
inal p r
icep e rch ai
r. (1ma r
k)
i
i) Pri
cep erc h ai
ra fterd i
scou nt
. (1mark)
b) Determin e
i
) T hen umb ero fch airthesch oolo ri
g inal
lyintendedt obuy. (4ma r
ks)
i
i) Pri
cep erc h ai
ra fterd i
scou nt
. (2ma r
ks)
i
ii
)T heamo un to fmo n eythes choo lwo ul
dh aves avedp e
rchairofit
gotthei ntend edn u mb erofch airsa tad i
sco untof15%. (2ma r
ks)
22.a) Wi thou tu si
n gap r
o t
ractor,c onstructtria
ng l
eABCs uc
ht hatangleABC=600, BC=8c ma ndAC=9c m.MeasureAB.
(3ma r
ks)
b) Drawap e rpen dicularfromAt oBCa ndme asureit
slength. (2ma r
ks)
c) Hencec a l
cu l
a tethea r
eao ftri
an gleABC. (
2ma rks
)
d)L ocateap ointDo nBCs ucht hatth ea r
eao f t
ri
a n
g l
eABCi sth r
eetime sthato ftr
ian
gleABD. (
3ma rks
)
23.Intr
iang leABC, sho wnb elow, AB=aAC=bp o i
ntMl ie
so nABs uc hth atAM:MB=2: 3andpoi
ntNlie
so nACs u
chthatAN:NC=
5:1li
n eBNi n tersectslineMCa tX.

F
ORMARKI
NGSCHEMESI
NBOX0724351706 Pa
ge|45
Ma
the
mat
ic
s121/
1,2

a) Ex
pres
st hefol
lowinginter
mso faandb
i
) BN (
1ma
rk)
i
i) CM (
1ma
rk)
b) Gi
venthatBX=k BNa ndCX=r CMwh e
rekandrar
escal
ars
i
) Wr itetwod i
ffer
entexpr
essi
onsforAXinter
mofa,b,
ka ndr (4ma
rks)
i
i)F i
n dthevaluesofka n
dr (
4mark
s)
24.Apar
ti
clemo vessuchthattseco
n d
safterp
assi
ngagi
venpoi
ntOisg
ive
nby
S=t(t–2)(t–1)
a)F i
n ditsve
locit
ywh ent=2s econ
d (3ma
rks
)
b)F i
n ditsmini
mu mv el
ocit
y. (
3mar
ks)
c
) Fi
ndthetimewh enthep ar
ti
cl
esismome nt
ari
l
yatre
st. (
3mar
ks)
d
) Fi
ndit
sa ccel
er
ationwh ent=3s ec
onds. (
1ma
rk)

BUURIEASTST
ANDARDS

121/2
MAT HEMATICS
.MOCKSKCSEPREDI CT
IONS,.
PAPER2
TI
ME:2½HOURS
SECTIONI(50MARKS):
Answeral
lt
heq u
est
io
nsint
hi
ssect
io
nint
hes
pac
esp
rov
ide
d.

y(
p-y)
1. Ma
kePt
hes
ubj
ecto
fth
efo
rmu
lai
n;x= (
3mark
s)
p-
i
2. T
hel
engt
hofar e
cta
n g
leis8.
3cmandit
swidt
his5.45c
m.Calcu
lat
e.
a
)T here
lat
iveer
rori
nt hea
reaoft
here
cta
ngle. (
3mark
s)
b
)T heper
centa
geerrori
ntheare
a. (
1mark
)
3. T
hef
i
gureshownb e
lowisacir
clec
ent
reO(notdra
wntosc
ale
).Fi
ndthev
alu
eofan
gle
sa,
ban
dcg
ive
nth
at<PSR=300.
(3marks
)

4. Thefi
rs
ttermo fanari
th
me ti
cs equen
ceis-5andthecommo ndi
f
fere
nceis3.
a)L istthefi
rst5te
rmso fthesequenc
e. (
1ma
rk)
b) De te
rminethesumo fthefir
st40ter
mso fth
es e
qu e
nce. (
2ma
rks
)
5. a) Givent h
ev ec
tosa=3i
r –j +2k,b=4i +2j–ka ndp=2a–b .Expr
esspint
er fi
mso j
,adk
n . (
2ma
rks
)
b) Hen cecalc
u l
at
e|p|co r
rectto3sig
nif
icantf
igu
res. (
1ma
rk)
6. Aquantit
yXv ari
esdi
rect
lyasYa ndt
hesquareofZ.wh enY=2andZ=5, X=150.
F
ind:-
F
ORMARKI
NGSCHEMESI
NBOX0724351706 Pa
ge|46
Ma
the
mat
ic
s121/
1,2
a
)T hel
awg ov
ern
ingX,Ya
ndZ (
2ma
rks
)
b
)T heval
ueofXwhenY=4and2=3 (
1ma
rk)
7. F
inda
ccu
rat
elyt
heshad
edar
ea. (
3ma
rks
)

8. Si
mpl
if
yth
efo
ll
owi
ngs
urdl
eav
inga
nswe
rint
hef
orma+b c (
3ma
rks
)
3-2
3+ 2

9. Ev
aluat
eth
efo l
l
o wi
nge
xpres
sio
nwi
th
outu
sin
gma
the
mat
ic
alt
abl
eso
ra c
alc
ula
tor
.
3l
og104+l
og10125–3l
og102 (
3ma
rks
)

10.Thedi
agr
amb e
lowre
pre
sent
sawedgewhos
ecros
s–sec
ti
oni
sar
igh
tan
glet
ri
ang
le.
PQVUisare
cta
nglea
ndthedi
men
sio
n sa
reass
hown.

Cal
cula
tet o2d eci
ma lplac
es:-
a)T hel engthofPV (
2ma rk
s)
b)T hes izeofthea ngl
eb e
twe enPVa ndplanePQRS. (
2ma rk
s)
6
11.a) Expan d(1+x ) uptotheforthte
rm. (
1ma rk)
b) Usingt hebinomi a
l e
xpan s
ionin(a)aboveesti
matethev al
ueo 96
f1. (
2ma rk
s)
12.Atra
nspo rt
erh a
stwot ypeso ftr
uckstotransp
ortma i
ze.Typ eAcarr
ie
s2000b ag
swh ol
etypeBcarri
es3000bagspertr
ip.The
tr
ansp
o r
terh astotranspor
t120, 000bags.Heh a
stoma ken otmorethan50tri
ps.TypeBtru
cksaretomakeatmostt
wicethe
numbero ftri
psma deb yty
peA.T aki
ngxt obethen umbero ft
ri
psma debytyp
eAt ruc
kandytob ethenumberoft
rip
sma debyt
ype
3.Wr i
ted owna l
lthein e
qualit
iesre
present
ingthisi
nformati
o n. (
3ma rks
)
2 2
13.Theequationofac i
rcleisx +y –4x+8y=5.F indthecen t
reandtheradi
usoftheci
rcl
e. (
3ma rk
s)
14.Sol
veforxi ntheeq uat
ionbe l
o wusi
ngt hecomp l
eti
ngthesq uar
eme t
hod. (
3ma r
k s
)
2
x –7x+10=0
15.MrSuda nwo rksforac ompan yearn
ingab asi
cs a
lar
yo f30,000andho use al
lowanc
eof12,000.Inacer
tai
nyearthe
gove
rnme ntch a
rgedt a
xo nPAYEb a
sisusingthetab
lebelow.

I
nco
meinKshperMont
h Ra
te(
%p erKs
h)
1–10,000 5
10,
001–20,000 10
20,
001–30,000 20
Over30,
0000 30

F
ORMARKI
NGSCHEMESI
NBOX0724351706 Pa
ge|47
Ma
the
mat
ic
s121/
1,2
I
fSudanisgiv
enap e
rson
alrel
i
efof
sh.3,
000perMo
nth
,fi
ndt
axhepay
sint
hatmo
nth
. (
3ma
rks
)
16.T
r
I
f
i
t
a
n
h
g
l
ea
eA’
re
ao
B’
C’
ft
r
i
i
a
st
n
h
g
ei
l
ma
eA’
g
B’
eo
C’
ft
D’
r
i
i
ang
l
s25.
eABCu
6c 2
m, f
i
n
n
de
rat
dt
hea
r
a
r
e
ns
f
ao
o
f
r
ma
t
h
t
i
eo
b
o
j
nma
ec
t.
t
ri
xT=
1 3
2 2 ( ) (
3ma
rks
)

SECTI
ONII
(50mark
s).
Ans ro
we nl
yfi
veq
uest
ionsi
nth
iss
ect
io
nint
hes
pac
esp
rov
ide
d.

17.a) Ont hes amed i


agramc ons
truc
t:-
i
) T r
iangl
ePQRs uchthatPQ=9c m, PR=7c ma ndtr
ian
gleRPQ=600 (2mark
s)
i
i)T helocuso fap oi
ntMs u c
hthatMi se q ui
dist
antfr
omPa n
dQ. (1ma
rk)
i
ii
)T helocuso fap oi
ntNs uchthatRN≤3. 5cm. (
1ma r
k)
b) Ont hed ia
g rami npart(
a)
i
) Sh adether egionB, c
o n
tai
ningallt
h ep oint
se ncl
ose
db ythel
ocusonMandt
helocuso
fNsuchtha
tPM≥QM.
(2ma
rks)
i
i)F i
ndthea reao ftheregi
ons ha
dedinp art(b)(i
)abov
e. (
4ma r
ks)
18.Ane x
aminati
o nin vol
vesawr i
tt
entestandap r
acti
calt
est
.T hepro
babi
li
ty
Thatacandidatep ass
esthewr i
tt
entestis6/11.If t
hecandi
datepas
sest
hewri
tt
ent
est
,th
enthep
rob
a b
il
it
yofp
ass
ingt
hepra
cti
calt
est
s3/5,
i ot
herwisei twou l
dbe2/7.
a
) Il
l
us t
rat
ethisinfo r
ma t
ioninatreedia
gram. (2mark
s)
b
) Deter
mineth epr obabi
li
tythatacandi
date
i
) Pa ss
esb otht es
ts (2mark
s)
i
i)p as
sesthewr i
tt
ent es
ton l
y (
2ma r
k)
i
ii
)p as
seso nl
yo netest (
2marks)
v) fail
sb o
tht est (2mark
s)

19.MrRaoi sawat
ersuppl
ie
rinacert
ainmar
ket
.Heh asatankwh i
chhol
d s 20,
000li
tre
s.
Thetankisbe
ingf
il
le
dwi t
hwa t
erf
romtwopipesPandQ.waterf
lowsatthera
teof150L/minu
tethr
oughpip
ePand100l/
minute
t
hroughp i
peQ.
a)Ifthetankise
mp t
yandthetwopip
esareope
nedatthesameti
me , c al
cul
atet
heti
metakentofi
ll
theta
nk. (3marks)
c) Onac e
rtai
ndayMrRa ost
art
edwithanempt
ytank,ope
nedp i
pesPandQf or30minut
es,af
te
rwh i
chheopenedp
ipeRtosup
plyhi
s
cu
stomers.Rsuppl
ie
swa t
eratara
teof20li
tr
esperminut
e.Cal
cul
atet
h et
imeitt
ooktofi
l
l t
hetan
k.(7ma r
ks)

20.Co
mple
tethet
abl
eb el
owb
yfil
li
ngint
h eb
lanks
pac
es. (
2mar
ks)
0
x 00 300 600 900 1200 1500 1800 2100 240 2700 300 330 3600
0
Cosx 1.00 0.
50 -
0.87 -
0.87
2Cos½x 2. 00 1.93 0.
00

a) Ontheg r
idp rovi
dedu singas cal
eo f 1cmtor epre
sent300o nthehori
zonta
l axi
sand4cmtorepr
ese
nt1uni
tonthever
ti
cal
0 0
axi
sdrawt heg r
apho fy=c osxa n dy=2c os½x (
4ma r
ks)
b) Stat
ethea mpli
tudea ndp eri
odo fy=2c os½x (2mar
ks)
c) Useyourg rap
ht oso l
veth eequ a
ti
o n (2mar
ks)
2cos½x–c osx=0
22
21.a)T a
k i
ngt heradi
u softh eeart
hR=6371k ma n
dπ= , calcu
lat
eco r
rec
tto2d .pt
hedis
tanc
ebet
weenth
etwociti
es,
A(600N,
7
290W)a ndB( 600N, 310E)a l
ongth ep ar
all
elo fl
at
it
ude. (3mark
s)
b)I fiti
s1200h r
sa tcityA, whatist hel
oca lt
imeatcit
yB? (
3marks)
c) Ana eroplaneflewd ueSo uthfromap ointP(600N,450E)top oi
ntQ.
Thed i
stancecove r
edb yth eaerop l
anewa s800km.De t
erminetheposi
ti
ono fQ. (4mar
ks)
22.Thema r
kedp ri
ceo fate l
evisi
ons etis25,600.Onc as
hp ayme n
tac ust
omerisgive
nad i
sco
untof5%onthemar
kedpri
ce.TheT.V
canals
ob eb oughto nh i
rep ur
c ha
s etermsb ypayi
ngad ep o
sitofsh12,640and16e qua
lMo n
thl
yin
sta
ll
ment
sofsh.1450eac
h.
a) Calc
ulat
e
i
) Ca shpric
eo fthema c h
ine (
2marks)
i
i)T hehir
ep urchasevalue (3mark
s)
i
ii
) Calc
ulat
et herat
eo fc ompo undinterestchar
g edperMo nthforhir
epurchas
eterms. (4mar
ks)
Iv
)F i
ndd i
ffe
renceb et
we enc ashp ri
cev al
uea ndh i
repurcha
s eval
ue. (1mar
k)
23.Thetabl
eb el
ows ho wsma sse
so f100f orm4s t
udents
.

Mas
sKg 30-
34 35-
39 40-
44 45-
49 50-
54 55-
59 60-
64 65-
69
Noofs
tud
ent
s 4 6 10 14 22 24 14 6
F
ind:
-

F
ORMARKI
NGSCHEMESI
NBOX0724351706 Pa
ge|48
Ma
the
mat
ic
s121/
1,2
a) Me a
nma ss (4ma
rks
)
b) Vari
ance (5ma
rks
)
c) Standa
rddevi
at
ion (1ma
rk)
24.
I
nthefi
gur
ebelo
wABCi
sat
ang
entt
oth
eci
rc
lea
tB.Gi
vent
hat<ABG=400,
<BGD=450a
nd<DBE=250

Fi
ndthes
iz
eoft
hef
ol
lo
win
gan
gle
sgi
vi
ngr
eas
onsi
nea
chc
ase
a
) <BDG (
2ma
rks
)
b
) <DGE (
2ma
rks
)
c
) <EFG (
2ma
rks
)
d
) <CBD (
2ma
rks
)
e
) <BCD (
2ma
rks
)

SUKEL
E

121/1
MOCKEXAMI NATI
ON
.MOCKSKCSEPREDI CT IONS.
TheKenyaCert
if
ica
teo fSecondar
yEducat
ion
MAT HEMATICS
Paper1
2½ho u
rs
SECTIONI(50MARKS)
Answerall
thequest
ionsinthisse
cti
on.
1. Wi t
houtusi
ngma themati
cal
tabl
esorca
lcul
at
or,
eva
lua
te

(
4ma
rks
)

2. Twoci
rcl
esof
equa
l r
adi
usha
vet
hei
rce
ntr
esa
tO(
0,0)a
ndA(
5,0)r
esp
ect
iv
elya
sshownb
elo
w.Gi
vent
hata
ngePAQ=1200,
l
Cal
cul
at
ethear
eaoft
hesha
dedr
egi
on. (
4marks
)

F
ORMARKI
NGSCHEMESI
NBOX0724351706 Pa
ge|49
Ma
the
mat
ic
s121/
1,2
3. Si
mbavi
ll
ageis74kmNo r
thWes
tofNy
ati
vil
l
age.Ki
f
aruv
il
la
gei
s42kmwesto
fNy
ati
vil
l
age
.Byu
sin
gap
pro
pri
at
esc
aled
rawi
ng,
fi
ndt
hebear
ingofKi
far
uvil
la
gef
romSi
mbavil
la
ge. (3mar
ks)
4. Si
mpl
if
ytheexpr
essi
on

(
3ma
rks
)

5. Mutuasol
dac artoKyal
oatalossof 20%. Ky
alosol
dthecart
oMu emaataprof
itof50%.Muemasoldthecart
oMusyok
aata
l
ossof25%.Ca l
culat
etheper
cent
a geprofi
tKyal
owo ul
dhavemadehadhesoldthecardi
rec
tl
ytoMu sy
oka.
(
3mark
s)
6. Oneofthein
teri
orangle
so f
ap ol
ygo nis840.I
f e
achoft
heo t
herangl
e s1470,
si howma nysi
desdoest
hepolygo
nhave
?
(3marks
)
7. TwobrandsofsugarXandYc os
tings h.45andsh.50res
pect
ivel
y.I
nwh a
trat
iomu stt
heybemixe
dino r
dertomak
ea25%
pro
fi
tbysell
i
n gthemixt
ureatsh
. 60? (3marks)
8. Si
mp l
if
ycomp l
ete
ly

(
3ma
rks
)

9. Ac
yli
ndr
ic
ali
ronp
ipei
s2.
1ml
onga
nd12c
misi
tse
xte
rna
ldi
ame
ter
.Th
eme
tal
is1c
mth
icka
ndh
asad
ens
it
yof7.
8g/
cm3.

T
aki
ng ,
fi
ndi
tsma
sso
fth
ema
ter
ia
lus
edt
oma
ket
hei
ronp
ipei
nki
l
ogr
ams
.

3marks
)
10.F
indai a+1)i
f( +4jispar
all
elt
o-2i+6j
. (3mar
ks)
11.F
romap oi
nt20mawayona l
eve
lgrou
nd,t
heangl
eofe
lev
ati
ontot
h el
owerwi
ndo
wli
n s270a
ei ndt
hea
ngleo
fel
eva
ti
ont
oth
e
t
opli
neofthewi
ndowis320.Ca
lcu
lat
eth
eh e
igh
toft
hewindow.(3mar
ks)
12.Th
ec u
rve i
nt
ers
ect
swi
tht
hel
i
ney=2x-
5att
hep
oin
ts A(
-1,-
7)a
ndB(
2.5,
0).
Det
ermi
net
hev
alu
eso
f
aandb. (3mar
ks)

13.Ev
alu
ate (
3ma
rks
)

14.Apart
iclemo vinginastra
ightlinemo vesfrompointPtopointQ,ad i
st
a nc
eo f168me t
res
.Itst
art
sfr
omr estandaccel
era
tes
uni
for
ml yfor5s econds.I
tthe ntra
velsatac on
stantspe
edfor10s eco
n dsandfina
ll
yd ec
eler
atesuni
f
ormlyfor3s e
condsto
cometor est
.Ca l
cula
tethema ximums p
eeda t
tai
nedinkm/ h.
(3marks)
15.Twosimilarcon t
ai
nershavema s
ses256k gand108k gres
p e
cti
vel
y.Ift
h esurf
aceareaofthesmall
ercont
aineri
s810cm2,whati
s
th
es u
rfaceareao fth
elargerc ont
ainer? (3ma r
ks)
16.Si
xwe eksa f
terplant
ing,t
heh eghto f
ma iz
ep l
antswereme a
suredtothen e
arestce
ntimet
re.Thefr
equencydist
ri
but
ionisgi
veni
n
th
etableb el
ow.
Heig
h t
(x) 0≤x<5 5≤x<15 15≤x<20 20≤x<35 35≤x<55
Fre
qu e
n cy 3 8 19 15 30
Drawah isto
gra mtorepre
sen ttheaboveinfor
ma t
ion. (4marks
)
SECTIONI I(50MARKS)
Answera nyfi
v equest
ionsfromt hissect
ion

17.Usi
ngar u
lerandap ai
ro fcomp ass
esonly
(
a)Co nstr
uctap ar
all
e l
ogramABCDs uchtha
t,AB=8c m, AC=12cma ndangleBAC=22. 50.Dropaper
pen
dicu
larf
romCto
ABp rodu
c e
d . (4ma rks)
(
b)F in
dt hes i
zeofan gl
eBAD. (1mar
k)
(
c)I nthediagr
ami n( a
)a b
ov eshowtheloc
iofp oi
ntPsuchtha
t
(i
) AP>PB (
1mark)
(i
i) AngeAPB≥900.
l (
2marks
)
(i
ii
) BP>2c m (
1mark)
(i
v) Shadethelocuso fP. (1mar
k)
18.(
a)Fi
ndt h
ee quat
iono ftheperpendi
cul
arbis
ect
oro ft
heli
n ej
oi
ningA(
3,9)andB( 7,5)int
hefo r
ma x+b
y+c=0.
(
3mar
k s)
(
b)T heperpendic
ularb i
sect
orin(a)i
nter
sect
swi t
htheli
nejoi
ningth
ep oi
nts( 2,4)and(
-3,1)atC.Fi
nd
(i
) Thecoordinat
eso fC. (3mar
ks)
(i
i) ThelengthofCf romli
neAB (2mar
ks)
(
c)T heli
neth r
ough( 2,4)and( -
3,1)makesana ngl
eof wi ththepo
sit
iv
ex -a x
is.
Findt
hev al
ueof .
(
2mar
k s)
19.Dr
awt hequadri
lat
eralA(-6,-1);B(
-6,-
4);C(3,-7)andD(3,2)
.(1mar
k)
Onthesameg ri
ddrawt heima ge

F
ORMARKI
NGSCHEMESI
NBOX0724351706 Pa
ge|50
Ma
the
mat
ic
s121/
1,2

(
a)A1B1C1D1t
hei
mag
eofABCDu
nde
ren
lar
geme
ntc
ent
re(
0,-
1)s
cal
efa
cto
r . (
2ma
rks
)

(
b)A2B2C2D2t
hei
mag
eofA1B1C1D1unde
rarot
ati
oncent
re(1,0)t
hro
ugha
nan
gl f+900.(
eo 2ma
rks
)
(
c)A3B3C3D3t
hei
mag
eofA2B2C2D2 u
nderar
efl
ect
io
nintheli
ney=x . (
2ma
rks
)

(
d)A4B4C4D4t
hei
mag
eofA3B3C3D3 u
nde
rat
ran
sla
ti
on a
ndwr
it
edo
wnt
hec
oor
din
ate
soft
hef
i
nal
ima
ge.

(
3ma
rks
)
2
20.Apart
icl
etr
avell
i
n ginas tra
ightli
n ep a
ssesthroughaf i
xedp oi
ntOwi t
hav el
oci
tyof0.5m/ s
.Theac c
ela
rati
onam/ s,tse
cond
s
af
te
rp a
ssi
ngO,i sgivenb ya=1. 4–0. 6t.Fi
nd
(
a)T heti
mewh enthep art
icl
eisin s
tantaneousl
ya trest
. (4mark
s)
(
b)T hema xi
mu mv el
ocityofthep art
icl
e. (3mark
s)
(
c)T hetota
ldist
ancet r
avelle
db yth epart
icl
eb et
we ent=0a ndt=10s econds
. (
3ma r
ks)
21.Arect
angul
artankwh oseinternaldimen s
ionsa r
e2.04mb y1.68mb y2.64misseve
n–e i
gthful
lofmilk.
(
a)I ft
hetankisma deo fme t
alo fthi
ckness30mm. Calcul
atetheext
ernalvo
lumeofthetankwh enclos
ed.
(
3ma rks)
(
b)Ca lcul
at
ethev ol
u meo fmilkinth etankinm3l eavi
ngy ouranswerto3s i
gni
fi
cantfi
gures.
(
2ma rks)
(
c)T hemilkistobep ackedins ma l
lpackets
. Eachpacketisintheshapeofari
ghtpyramidonane quil
at
eralt
ri
angul
arb
aseof
sid
e19.2cm. Theh ei
g htofeachp acketi
s13. 6cm. F
ullpacket
so b
tain
edaresol
da tsh.
35p erpacket
.Cal
cula
te
(
i)T hevolumeo fmilki
nc u bi
cc enti
me t
resco nt
ainedineachp ack
etto4s i
gni
fi
cantf
igures.
He ncefi
ndthenumberoff
ull
packet
s. (
3ma r
ks)
(
ii
)T heexactamo u
n tth
a twillbereali
sedfromt hesaleofallt
hep ac
ketsofmil
k. (
2ma r
ks)

22.Ast
adi
umc anful
l
yma xi
mizeonpr
ofi
tsi
f72000peo
pleareac
commo d
ated
.Eachrowac
commod
atest
hesa
menumberofp
eop
le.
Onacer
tai
nd a
y25r owswerede
ff
ect
ivean
ditwast
here
foredec
ide
dthateac
ho ft
her
emai
ni
ngrowswoul
dha
vetos
q u
eez
e40
mor
epeopl
eforthest
adi
umt oha
vethemaxi
mu mnumberofpeo
ple
.Calc
ulat
e
(
a)Ho wma nypeop
leeachrowofse
atshol
ds. (
6mar
ks)
(
b)T h
en umberofr
owsinthest
adi
um. (
1mar
k)

(
c)F
romt
het
ota
lti
cke
tss
ale
s;i
f i
sus
edo
nre
pai
rso
fth
ede
ff
ect
iv
ero
wsa
nd o
fth
ere
mai
nde
rone
xpe
nse
s,s
h.

4,
320,
000i
sle
ft
.Ca
lcu
lat
eth
eva
lueo
fea
cht
ic
ket
. (
3ma
rks
)
23.Copyandc
omp
let
eth
eta
blef
ort
hef
unc
ti
onof (
2ma
rks
)
x -3 -
2 -
1 0 1 2 3 4
2
-2x -18 -
2 0 -
2 -
32
5+3x -4 2 5 11 14
y -22 5 3 -
15
(
a)Us
eth
ev a
lue
sfr
omthet
abl
etop
lotag
rap
hof . (
3ma
rks
)
(
b)Us
eyourgr
apht
oso
lve
(
i) (
1ma
rk)
(
ii
) (
2ma
rks
)
(
c)Deter
min
etherangeofv
alu
esofxwh i
chsat
is
fyt
h ei
neq
uali
ty (2mar
ks)
24.I
nthef
i
g u
rebe
low,Oisth
ecentr
eofth
ec i
rcl
e.A,B,
Ca ndDarepoi
nt
sont
hec
ir
cumfe
ren
ceoft
heci
rcl
e.A,
O,Xa
ndCa
re
p
oin
tsonast
rai
ghtl
i
ne.DEisatang
enttotheci
rc
leatD.AngeBOC=480a
l ndang
leCAD=360.

(
a)Gi v
ingr
eas
onsi
nea
chc
ase
,fi
ndt
hev
alu
eof
thef
ol
lo
win
gan
gle
s
(
i) CBA (2ma
rks
)
(
ai
)BDE (
2ma
rks
)
(
bi
)CED (
2ma
rks
)
F
ORMARKI
NGSCHEMESI
NBOX0724351706 Pa
ge|51
Ma
the
mat
ic
s121/
1,2
(
b)I
tisa
lsogi
vent
hatAX=12c
m,XC=4c
man
dDB=14c
man
dDE=15c
m.Ca
lcu
lat
eth
ele
ngt
hof
(
i) DX (
2ma
rks
)
(
ii
) AE (
2ma
rks
)

SUKEL
EJOI
NTEXAMI
NAT
ION

121/
2
MATHEMAT
ICSPAPER2
Ho
urs
SECTI
ONI (50marks
)
Answe
ral
lt
heque
sti
ons

1. Fi
ndmatri
xXgiv
entha
tAX X= wh e
reA= (3marks)
2. Th
ec os
tofpr
ovi
din
gac ommodi
tyc
onsi
st
so ft
rans
port
,la
bou
ran
drawma
ter
ia
linth
erat
io8:4:
12res
pect
iv
ely
.Ift
hetra
nsp
ortc
ost
i
ncre
asesby12%labourco
st18%a n
dr a
wma t
eri
al
sby40%,fi
ndt
hep
erc
enta
geinc
reas
eo fpr
odu
cin
gthen ewcommodi
ty(3
mks)

3. F
indt
hen
umb
ero
fte
rmsi
nth
ese
ri
es (
3ma
rks
)

4. Us
ingb
ino
mia
lex
pan
sio
nsi
mpl
i
fy l
eav
ingy
oura
nswe
rint
hef
orm wh
erea
,ba
ndca
rer
ati
ona
lnu
mbe
rs

(
3mar
ks)
5. T
hefi
gurebe
lowsho
wsext
ernal
int
ers
ect
io
no ft
woch
ord
sSMa ndNKwh
icha
rep
rod
uce
dtome
eta
tP.
Give
nth
atSOKi
sth
e
d
iame
teroft
heci
rc
lean
dthatSM=15cm,MP=18cma
ndPK=22c
m

M 18 P
S 15

22
O

N x

a
).Ca
lcu
lat
eth
eval
ueofxrep
rese
nti
ngl
eng
tho
fch
ordNK. (1ma
rks
)
b
).Ca
lcu
lat
eth
era
diusoft
hecir
cl
e (
2ma
rks
)

6. De
ter
min
eth
eeq
uat
io
nofac
ir
cl
eif
thec
oor
din
ate
sof
thee
ndp
oin
tso
fit
sdi
ame
tera
re l
eav
ingy
our
a
nswe
rint
hef
ormo
f wh
erea
,b,
c,da
ndea
rei
nt
ege
rs.
(3ma
rks
)

2
2 r 1+c
7. Make‘
c’t
hesub
jec
toft
hefor
mul
aifv = + 2 (3mar
ks)
3 r
2
8. Fi
ndth
eareae
nclo
sedbyt
hecur
ve xa
y=81- n
dt h
ex -
axi
sus
ingmi
d-o
rdi
nat
eru
lewi
th9s
tr
ips
. (4mk
s)

9. Wi
th
outu
sin
gac
alc
ula
toro
rta
ble
s,f
i
ndt
hev
alu
eof
xin

(
3ma
rks
)
2
10.Sol
veforϑinthee qu
atin6c
o os ϑ-sinϑ- 4=0inther
ange
0 0
-
180 ≤ϑ≤180 (3mks)
.
11.Wamb uadepo
sit
ed6,400inani nve
s t
me ntacco
u n
twhic
hpays30%inte
res
trat
ep e
rannumcompound
edsemi–ann
uall
yfo
r3y e
ars
.
1
Muindein
vest
edo n
ea ndah a
lfti
me sthatofWa mbuaat12 % pe
rannums i
mpleint
ere
stf
or6y
e a
rs.F
indwh
o s
einv
est
mentear
ned
2
moreint
ere
standbyh owmu chgivey o
ura nswert
othenea
res
tshi
ll
in
g. (3mks)
12.For
mi neq
uali
ti
estha
tsati
sfyt
h egiv
e nregionR. (
4ma rk
s)

F
ORMARKI
NGSCHEMESI
NBOX0724351706 Pa
ge|52
Ma
the
mat
ic
s121/
1,2

13.Us
elo
gar
it
hmst
oev
alu
ate (
3mk
s)

14.Gi
vent
hat a
ndt
haty
=1wh
enx
=1,
fi
ndt
her
el
at
io
nsh
ipb
etwe
enxa
ndy
. (
3ma
rks
)

15.Abia
sedc
oini
swe
ight
edsuc
htha
tit
sta
ili
stwi
cea
sli
kel
ytoa
ppe
ara
sth
ehe
ad.
Fin
dth
epr
oba
bil
i
tyt
hatt
hatata
ilandah
ead
ap
pearwh
enth
ecoi
nisto
sse
dtwi
ce. (3mark
s)
16.T
hep
oin
tU i
sma
ppe
don
toU’ u
nde
ranant
ic
loc
kwi
ser
ota
ti
ona
bou
tth
eor
igi
n.F
indt
hea
ngl
eofr
ota
ti
on.
(
3marks
)

SECTIONII(50ma rk
s)
Answeronlyf
iv
eq ues
tionsfr
omt hi
ssect
ion
17.ABCDi saquadr
il
ater
alwithcoordi
nate
sA( 2,1),
B( 3,2),C(3,4)andD( 0,3)
.ABCDi sma p
p e
do nt
oA’B’C’
D’unde
rtrans
for
mati
on
Tgivenbyashearwit
hx–a xi
sinva
ria
nts uc
hthatA’(4,1).
a) Deter
minethe2x2t ransf
ormati
onma t
rixrepr
esent
ingTa ndhencedeter
minethecoor
dinat
eso fB’
,C’a
n dD’.(
2ma r
ks)
b)A’B’C’D’i
stra
nsfor
me dtoA”B”C”D”u nderatr
ansf
orma t
ionHsu chthatA” an dD”
Deter
minethe2x2ma tr
ixrepr
esent
ingHa ndhenced e
terminet
hec oor
d i
nat
esofB”andC” (3marks
)
c) A”B”C”D”mappedontoA” ’
B”’
C”’D”’underatr
ansf
orma t
ionVrepresen
tingaref
lec
ti
onintheline .Det
ermi
nethe2x2matr
ix
re
p r
esent
in
gVa ndh encedet
erminethecoordi
nat
eso fA”’
B”’C”
’D”’ (3ma r
k s
)
18.
Thefoll
owingta
bleshowsh ei
ghtsof200s ee
dli
ngsof t
hesamesp e
c i
esme as
ure
dtothen ea
restcm.
Height
(cm) f r
equency
70-79 7
80-84 30
85-89 66
90-94 57
95-99 27
100-109 13

a
) Cal
culatethe:
-
i
) Quart
il
ed evi
ati
on 3ma r
ks
b
) Sta
nda r
dd evi
ati
onusingaworkingme a
no f102 3ma r
ks
c
) Drawa no gi
vecurv
erepres
enti
ngthea bov
ein f
ormati
ona n
du seittodet
ermineth ep er
centagen umb erofseedli
ngwh oseh ei
g h
t
ran
gesb etween85≤H≤93. 4marks
19.Inordertoensureopt
imalheal
thalabtechni
ciann ee
dst ofeedtherabbi
tsonad a i
lyd i
etco ntaini
ngami nimumo f24g ramso ffa
t,
36gramso fcar
bohydra
tesand4gramso fpr
otei
n. Rath
e rthanorderrab
bitf
o odtha tiscustomb l
endeditischeapertoorderfoodX
andfoodYa ndb l
endthemforano pt
imaluse.Onep acketoffoodXc ont
ains6g r
amso ffat,12g ramso fcar
boh yd
rates,2g r
amso f
prot
ei
n sandc os
tsSh50.Wh i
leonepacketoffoodYc on t
ains12gramsoffat,
12g ra mso fcarbo h
ydrates,
1g r
amo fproteinsandit
cost
sSh60
a)
.F orma l
ltheinequa
li
ti
estorepre
sentthei
n f
ormati
o nabo ve
. (4mk s).
b)
.Gr aphalltheine
quali
ti
es (4ma r
k s
)
c)
.De ter
mi nethenumberofpacket
so f
typeXa ndYf eedthatshouldbeu s
edforo pt
ima l h
ea l
tha tminimumc ost.(2ma r
ks)
20.MrKibet’sPAYEp e
rmo nt
hisk s
h29,522. T
h erat
ioo fhisbasi
csa l
aryt
otaxableallowanc esis13:9. Heisho u
s e
db yh i
semp l
o yer
andpaysan orminalr
entofks
h9000. Histaxabl
ep ayish i
sg r
osspayplus15%o fh i
sg r
o sspa yle
ssthen orminalr
en t
.Hei senti
tl
ed
F
ORMARKI
NGSCHEMESI
NBOX0724351706 Pa
ge|53
Ma
the
mat
ic
s121/
1,2
t
oap
ers
ona
lre
li
efo
fks
h1056p
ermo
nth
.Th
efo
ll
owi
ngt
axr
ate
sar
eus
edi
nth
eta
xco
mpu
tat
io
n.

Incomep erannum(k£p e
r Rates(KshperK£)
annum)
1-5808 2
5809-11280 3
11281-16756 4
16753-22224 5
22225a ndo ver 6
Cal
culat
e:
a) Hi
sb asi
cs a
larypermo nt
h. 7ma r
ks
b)T h
ep ercent
agec hangei
nh i
sPAYEifhista
xablepayincr
easesby10% 3ma r
ks
21.Agroupo fst
ud e
ntswishtobui
ldupap i
leoftoybric
kssoa stohave2bri
cksi
nth
etopr
ow,
4bri
cksi
nth
ese
c ondrow,a
nd6b
ric
ks
i
nthet hi
rdrowa ndsoo n.I
fthe
yh a
ve3000b rick
s
a) Howma nyrowscanth e
ycomp l
et
eandh owma nyb r
ick
sareth e
yleftwi
th
? (4marks
)
b) Cal
culat
et h
ele a
stnumb e
rofbri
cksthe
ys hou
lda ddtohaveallbr
ic
k sus
edwit
hnore
mai
nde
r? (2marks
)

c) Coun t
ingfromt heto p,te3rdr
h o w,9thr ow,27thr owa ndsoo nformag eome t
ri
cprogr
essi
on.Ifrowsformingthisprogre
ssi
onare
tobesumme du p,calcula
tet heto t
alnu mberofb r
icksformingthisarr
angeme ntaf
te
rthenumbero fbri
cksin(b)aboveh av
ebeen
inc
luded. (4marks)
22.Pa ndQa r
et wop i
ntso nla t
it
ude60°S. Thei
rlongitu
d esare30° Eand150°Wr espe
cti
vel
y.Fi
ndtoo nedecimal pl
ace:(Taket
he
22
radi
uso fthee ar
th=6370k ma ndπ= )
7
a)T hed i
st
a nceink mb etwee nPa n dQa longthep aral
lelofl
atitud
es. (2marks)
b)T heshor t
estd i
sta
n cealongt hee art
h’ssurfac
eb etweenPa n dQ. (3ma r
ks)
c) Awe atherforecaste
rr e
p ortsthatthec ent
erofac ycl
o neat(30°S120°W)i smo vi
ngduesout
ha t24k nots
.Ho wlongwi ll
itt
aket
o
reac
hap oint(45°S, 120°W) . (2marks)
d) Ap l
anel ea
v esPa t1400h rstoQa tas peedof900k notsal
o ngtheshort
estroute
.Deter
minetheti
mea tQwh entheplanearr
iv
ed.
(3ma rks
)
3 2
23.Ac urv
ei sreprese
n t
edb yt hefollowingfun c
ti
on ;y=x -4x +5x- 2.Usethec urv
etoanswert
hef ol
lowingqu e
s t
io
ns;
d y
a)
.F ind (1mk)
d x
b)
.(i
).De t
ermin ethevalueso fthestationerypoint
so fthecu r
ve. (4mks)
(i
i
).De t
ermi nethen atureo feachs t
ati
o ner
yp oi
n tonthec urve (2mks)
3 2
c)
.I nthesp acep r
ovi
d ed,sketchthec u
rveo fy=x -4x +5x- 2 (3mk s)
24.T
h r
eeq u
a nti
tiesA,Ba ndCa resuc hthatAv ar
iesdirect
lyasth esquar
erooto fBandinve
rsel
yasthes qua
reo fC.
a).Gi vent hatA=4wh enB=64a ndc=5, fi
n d
i)
.T hel a
wc onnecti
ngA, Ba ndC (4mks)
ii
).Awh enB=16a ndC=10 (2mk s)
b)
.I fBisincreasedb y44%a n dCd e cr
easedb y20%, fi
ndt hepe r
centa
gechang einA. (4mk s)

KIENIEAST
121/1
MAT HEMAT ICS
PAPER1
.MOCKSKCSEPREDI CTI
ONS/..
SECTION1: (
50MKS)
Answe ra l
lt
h equesti
onsinthi
ssect
ioni
nthespacepro
vide
d.
1. Withou tusi
n gac al
cul
ator
,eval
uat
e;
-8+ (5)x(
- 8)
- -(
-6)
(3mk
s)
-3+(-8)÷2x4
2. Fact
orizea ndsimpli
fytheexp
ress
ion.
X2-
9
2 (
3mk s
)
5x -13x -
6
3. Usecu ber ootandreci
p r
oca
ltabl
estoeva
lua
tecor
rectt
o2d eci
malp
lac
es.(
3mks)
1 3
- 0. 512
0.489
4. I nt hefi
g ur
eb el
owABCDi sacycli
cquadr
il
at
e r
al
.L i
neTBCisp
aral
l
elt
oli
neADa
ndangeACB=400.
l

F
ORMARKI
NGSCHEMESI
NBOX0724351706 Pa
ge|54
Ma
the
mat
ic
s121/
1,2

400

Fi
ndthesizeo
f;
a)Angl
eCAD (
1mk
)
b)AngleTBD (
2mks)
5. Giv
enthatm =i +2j–3ka ndn=4i –j–kfi
nd|n- m|co
rrec
ttoonedec
imal
plac
e. (
3mks)
̂ ̂ ̂̂
6.Gi v
enthatX=- 2,f
in
dth ev
a l
ueofyandzfo
rthes
imul
tan
eou
se q
uati
on. (
3mks)
x+y–z=- 1
x–2y+z=- 7
7. Theequat
ionofli
neL1is2y–5x–8=0a ndL2pa
sse
sthr
oughpoi
nts(
-5,
0)andi
sp e
rpe
ndi
cul
art
oL1.F
indt
hee
quat
iono
fL2
l
eavi
ngitindoubl
einte
rceptf
orm. (
3mks)
8. Thedi
agrambelowshowsat r
iang
ula
rpri
smABCDEF.AB=6c mAE=BE=7c ma ndBC=20cm.

E
D

C
7c
m

A 20c
m
6c
m
B
F
indt
hes
urf
acea
reao
fth
epr
is
m. (
3mk
s)

9. AF
ore
xBu
rea
uinKe
nyab
uysa
nds
ell
sfo
rei
gnc
urr
enc
iesa
ssh
ownb
elo
w.
Buyi
ng Sel
li
ng
(Ks
h) (Ks
h)

1Eu
ro 137.
65 138.
20

1USd
oll
ar 94.
33 94.
84

F
ORMARKI
NGSCHEMESI
NBOX0724351706 Pa
ge|55
Ma
the
mat
ic
s121/
1,2
Abusi
nessmanfromAme ri
cac on
ve rt
ed20,730Eu r
osintoKenyashil
li
ngs.
a) Calcul
at
ethea mountofmo ne
y ,i
nKe nyashi
l
lingsthath
erec e
ive
d. (1mk)
b)T heb u
sin
e s
sma nsp
entKs h2,410,300wh i
l
ei nKenya.Heth enconv
ert
edther
ema
ini
ngKe
nyas
hil
l
ing
stoUSdol
l
ars
.
Calcul
at
ethea mountofmo ne
y ,tothenea
restUSd ol
lar
,thatherece
ive
d. (
2mks
)
10.Twoma t
ri
cesPa ndQa r
esucht ha
t

Giv
e
P=
nthatt
h
( )
ed
k 4
3 2
et
ermi
a
n
n
a
n
d
to f
PQ=4f i
ndthev
( )
Q =
al
ueo f
1 2
3 4
k. (3mks
)
11.Const
ruc
talineXY9. 2cml ong.Byc o
nstr
uct
iond i
vid
eXYi nt
o5e q
ualpar
ts. (
3mks
)
12.Thefo
ll
owingfi
gurerepr
esentsane twit
hap at
hma r
kedonit.

E F

E D C F

A B

E
F
a) Wh ats
oli
dd oe
sthenetrepre
sent
. (1mk)
b) Drawthesoli
dandma rkthepat
ho nit. (3mks)
13.TwotownsAa ndBare3000n aut
ica
lmilesapar
t.Botht
ownsaresi
tua
tedonth
eequa
tors
uchth
atBist
oth
ee a
stofA.Calc
ulat
e;
a)T helongi
tudedi
ff
erencebet
weenAa ndB. (1mk)
b)T heloca
lti
mea tAifthel
ocalti
mea tBis2:
15a .
m. (2mks)
14.Sol
vetheequat
ion.
2Co s2θ=1 f
o r 00≤θ≤3600 (4mks)
15.Apoli
cepat
rolcart
rav
ell
ingat120km/ hischas
ingasusp
ectcarwh
ichis5k
ma wa
yandtr
ave
li
ngat80km/
h.h owfarawa
ymu st
th
ep o
li
cetra
velinorde
rtoc a
tchupwi t
hthesuspec
t. (3mks)
16.Writ
edowntheinequa
li
tie
sd e
fi
n i
ngtheu n
shadedregi
onRb e
low.

SECTIONII
:(50MKS)
Answeronl
yfiv
eques
ti
onsi
nthi
sse
cti
oni
nth
espac
ep r
ovi
ded
.
17.Thele
n g
thof45peapod
sweremea
sur
edt
othen
eare
stmil
l
imete
rsa
ndt
abu
lat
eda
ssh
ownb
elo
w.

L
ength(
mm) 10-
19 20-
29 30-
39 40-
49 50-
59 60-
69 70-
79
f
req
uenc
y 2 7 11 13 6 4 2

Cal
cul
at
e;
i
)Themeanl
engt
h. (
2mk
s)
F
ORMARKINGSCHEMESINBOX0724351706 Pa
ge|56
Ma
the
mat
ic
s121/
1,2
i
i)Themedi
a nle
ngt
h. (
3mk s)
i
i
i)Giv
enthatpeapo
ndsoflen
gthabov
e54.5mmwe r
econs
ider
edfo
rexp
ort
,whatper
cen
tag
eoft
hepeapondsqual
if
i
ed
f
orexp
ort. (
2mks)
i
v)Onthegri
dp r
ovi
ded,d
rawah i
st
ogr
amt or
epr
ese
ntth
edata
. (
3mks)
18.T
hedia
gramb el
owrepr
ese
ntsasol
idf
rust
rumwit
hbaser
adi
us21cmandt
oprad
ius35c
m.thehe
igh
tofth
efrust
rumis30c
m.

35c
m

30c
m

21c
m

Usingπ=3. 142, Calc


ulateto1d eci
ma lpl
ace,
i
) T hev olumeo fme t
al i
nthefrustr
um. (4mk s)
i
i)T hef r
u s
trumi sme l
tedandr ec
astedi
ntos o
lidsphere.I
nt heproc
ess20%o ft
heme tal
casti
slost
.Calcul
at
et hera
d i
us
ofthes ph e
re. (
3mk s)
i
ii
)T het otalsurf
aceareao fthesol
id. (
3mk s)
19.ThreetradersMa ry,AketchandWa mb uadeci
d e
dt obuyat ra
iler
.T h
ema rke
tpri
ceofthetra
il
erwasKsh4,200,000.Beforethe
y
couldp a
yt hed eposi
t,t
h efi
gurewa sdecr
easedint h
eratio5:6.Thedeal
eragre
edthatthet
hreecoul
dp a
yad eposi
tof40%o fthe
mo neyan dth eres
ttob ep a
idwithi
no neyear.Th erat
ioofco nt
ri
but
ionofMa r
ytoAk e
tchwas2:3,where
asther a
ti
oofcon t
ri
buti
on
ofAketcht oWa mb uawa s4:5.T hebalan
cewa stob epaidtothedeal
erfro
mt hepr
oc e
edsofthetr
ail
eri
nthesa merat
ioasthe
deposit
s.T het hr
ees har
e dtheproceedsf
romt hetrai
le
rinthes amerat
ioasthedep
os i
ts
.Du ri
ngtheyear
,thet
r ai
l
erreal
iz
edSh
1,400,000.
i
) Ho wmu chd i
dthet r
a i
l
era c
tual
lycost
? (
2mk s)
i
i) Ho wmu chd epos
itdidAk et
chc o
ntri
bute? (
2mk s)
i
ii
)Ho wmu cho ftherema i
ningamo untdi
dWa mb uapa yattheendoftheyear
?
i
v) Af terpay i
ngtherema ini
nga mountattheendo ftheyear,howmu chmo ne
ywas
Ma ryleftwith?
20.a) So l
vethee quati
on.
x+y=17
xy–5x=32

b)T heSt .T homa sMa thema t


icsteacherp l
annedtos hareo ut240s weetstoacla
ss.
Ho weve rsixmo res t
uden t
sjoinedth eclass
.Asar esult
,ea choftheincr
ease
dn umb e
ro fst
udent
sg ottwentysweet
sles
s.
i
) F indth eo r
iginalnu mbero fst
ud entsinthec l
ass. (
4mk s)
i
i)Inth esec ondwe ekt hePrincip
a lagreedtoincreasethen umb erofsweetsbya662/3%o fth
ep r
evi
ou swe e
k.
Calculatethet ot
al numb erofswe etssharedo u
tinth eseco ndweek.ifonl
ytheo r
igi
nal nu
mb e
rofs tud
e nt
swe r
epres
ent.
(
2mk s)
fA1B1C1a eA1( 5)B1( dC1( . 11 1
21.Theve
rticeso fatriangleABCa r
eA( 6,1)B( 6,3)an dC( 8,1) .Thevert
iceso r 4, 2,5)an 4,
7) ABC i s
arot
at
iono fABC.
a) Ont heg ridp rov
idedp lotthetri
a ngl
esABCa ndA1B1C1. (
2mk s)
b)F indb yc onstruct
iont hecentrea nda ngl
eo frot
ati
on. (
3mk s)
c) Determi nethev erti
ceso ftr
iangleA11B11C11wh ereA11B11C11istheimageoftri
angleA1B1C1underar e
flec
ti
o ni
nthe
li
ney =-x. (
3mk s)
d) DrawA111B111C111t heima eA11B11C11u
g nderre
flect
iona longX=0. (
1mk )
c)F romt het ri
an gl
esd rawn ,s
tat
eo nep ai
rthatis
i) Directl
yc on gruent. (
1mk )
i
i) Op o s
iti
v el
yc ongruent. (
1mk )
sS=1/ 3 2
22.Apart
icl
emo vesa lon
gas tr
aightli
n es u
c hthati
tsdispla
ce men tSme t
resfromag i
venp oi
nti 3t–3 t +9t+5wh eretisth
e
t
imeinse c
o nd s
.F ind.
a)t hed i
splaceme nto fthep a
rti
clea tt
=5c o r
rectt
o2d ecimal pl
aces. (
2mk s)
b)t hev e
loc i
tyo fthep ar
tic
lewh ent=6. (
3mk s)
c)t hev a
lueo ftwh e nthep art
icl
ei smo me nt
aril
yatr e
st. (
3mk s)
d)t hea c
c eler
a t
iono fthepa r
ti
clewh e
nt =2. (
2mk s)
23.Thebounda r
ieso faran cha r
es t
rai
g htli
n es,AB,BC, CDa ndAD.Di s32k monab eari
ngo f2330fr
omA, Cisdirec
tl
yNo r
thofDand
FORMARKINGSCHEMESI NBOX0724351706 Pa g
e|57
Ma
the
mat
ic
s121/
1,2
0
duewestofA;Bi s25k mo nabea
ringof318 fr
omC.
i
) u si
n gasu i
tabl
escal
e, r
epr
ese
nttheinf
ormati
onab o
veinthes
cal
edr
awi
ng. (
3mks)
i
i)F romthes cal
ed r
awingin(i
)abovefi
nd
a)t hedist
ancea ndbear
ingofBfro
mD. (
2mks)
b)t hedist
ancea ndbear
ingofAfro
mB. (
2mks)
i
ii
)F i
ndthea r
eao fth
eran c
hinsqua
rek i
l
ome t
ers
. (
3mks)
⃗ ⃗
24.Atri
angl
eOP Qi ssuchthatOP=pandOQ=p.Ap o
intRd i
vi
desOP i
nther
ati
o1:
2an
dap
oin
tSd
ivi
desPQi
nth
erat
io5:
2.OS
an
dRQme etsatT.De te
rmine.
i
) OS (
1mk)
i
i) QR (
1mk)
i
ii
)GiventhatQT=n QRa ndOT=mOSf i
ndthescal
arsma ndn. (
6mks)
i
v) Showt hatpointO,Ta ndSar
cco l
li
near
. (
2mks)

KIENIEAST
121/2
MATHEMATICS
PAPER2
.MOCKSKCSEPREDI
CTI
ONS/
..

Sect
ionIc o
nsis
tsof16quest
ion
s.
1. Uselogar
ithmtabl
estoeva
lua
te; (3mks
)
341.34x0.02357
2.149
2. Jumac a
np ai
ntah a
lli
n9d ay
san dKeyac
anpai
ntth
ehalli
n18day
s.Howlo
ngwoul
ditt
aketop
ain
tth
esa
meh a
lli
fth
eywor
k
to
gether
? (2mks
)
3. Aq u
a n
ti
tyAv ar
iespar
tl
yasBa ndpart
lya
sthesq
uarer
ootofB.WhenB=4,A=22andwhenB=9mA=42.F indAwhenB=25.
(5mks
)
4. Thefi
gurebelowrepr
esen
tsacubo i
dPQRSTUVW.
PQ=45c m, QR=9c ma ndRW=10c m.

T W

U 10c
m
V
R
S
9c
m
P
45c
m Q
Cal
cul
atetheangl
eb et
weenli
nePWa ndpla
n ePQRSc o
rrec
tto2decimal
pla
ces. (3mks)
5. Makeqthes ub
jectofthef
o r
mulain;
m d
= (3mks)
n q -3
5
6. a) Expand(2–x ) (2mks)
b) Useexpansi
oni n(a)abov
eu pt
othete
rmi nX3t
oa ppr
oxima
te1.995t
o4s .
f. (2mks)
7. Wit
houtusi
n gmathe mat
ic
altab
lesnorca
lcul
ator
,si
mp l
if
y;
3 3
- i
nthefor
mab (3,
mks)
5-13 5+ 13
8. Int
hefi
g ur
eb e
low, GDFisatangentt
ocir
cleABCDa tD.AECFa n
dBEDa rest
rai
ghtl
i
nes
.AE=4c
m,BE=2c
m,ED=10c
ma nd
DF=10c m.

B
A

4c
m 2c
m
E

10c
m

F
F
ORMARKI
NGSCHEMESI
NBOX0724351706 Pa
ge|58

D
G
Ma
the
mat
ic
s121/
1,2

10c
m

F
indt
helen
gthofAF. (
3mk
s)
9. F
indXif
; l o
g(x-1)+2=lo
g(3x+2)+l
og25 (
3mk
s)
10.T
hetab
lebel
owshowssomeval
uesoft
hef
unc
ti
on y=X2+3.
X 0 ½ 1 1½ 2 2½ 3 3½ 4 4½ 5 5½ 6
Y 3 4 5¼ 7 12 15¼ 19 28 39

a)Comp l
etethet
able. (2mk s)
2
b)Usethemi dordi
nateRu l
ewithsi
xordi
natestoest
imat
et h
ea r
eaboundedbyy=x +3,theyax i
s,t
hexaxisa
ndt heli
ne
x=6. (
2mk s)
11.Fi
ndtheradiusandthecoo r
dina
tesoft
hecentreofacir
cl
ewh oseequa
ti
onis
2 2
x +y +7x–2y+7=0 (
3mk s)
12.9pupil
sinac er
tai
nsch oolst
ate
dthenumbero fst
orybooksthe
yh a
das11, 8,512, 15,13,14,19,11.Fi
ndt
h est
andarddevi
at
io
n
oft
hed a
ta. (3mk s)
2
13.Sol
vetheeq ua
ti
on 4Sin θ+4Co sθ=5f or00 ≤θ≤3600.Gi veyouransweri
nd e
grees. (
3mk s)
14.Arect
anglehassi
deso fl
en g
th18c mand7.5cm.Wh ati
stheperce
nta
geerrori
nc a
lc ul
at
ingit
sa r
ea? (
3mk s)
15.Thefi
rs
t,t
h i
rdand9thtermsofanA.Pformthefir
stthr
eeconsecut
iv
eter
mso faG.P.Gi venthatt
hec ommondiff
eren
ceo ft
heAP,d
=6,fi
nd;
a)thefi
rs
tt e
rmo ft
heAP. (
3mk s)
b)thecommo nrati
oro ftheG.P. (
1mk )
16.Gibs
onb oughtamotorbikefors
h75,000.Itsdepre
c i
at
ionrat
eis12%p e
ra nnu
m.Wh a
twillbetheval
ueoft
hemo t
orb
ikeatt
heend
ofsi
xt
hy ear,
tonear
est5c ents. (
2mk s
)

SECTI
ONB: (50MKS)
17.Acomp anyempl
oyeeear
nsamo nt
hl
ybas
icsal
aryo
fsh25,000.Heishou
sedb
yth
eempl
oye
randt
hus15%o fh
isb
asi
csa
lar
yis
adde
df o
rthepurp
osesofta
xati
on.Her
ecei
vesmedi
cal
all
owanceofsh4,
800a
ndhou
sea
ll
owan
ceofsh2760.
a)findtheemploy
ee’
staxab
leinc
ome. (
2mks)
Usethetabl
eoft
axat
ionbel
ow;

Month
lyt
axablei
nco
mei
nKs
h. Ra
tei
n%
1-4,550 10%
4,
550- 9,200 15%
9,
201–13, 800 20%
13,
801–18,620 25%
18,
621–23,940 30%
23,
941andab ov
e 35%

b
)I ft
hee mployeeisenti
tl
edtoap ers
on a
l rel
i
efofsh960p ermonth,de
terminet
hen e
ttax. (5mks)
c
)T heemp lo
y eewasg i
vena40%i ncreaseinhisbasi
csala
ry.Calcu
lat
ethep er
cent
ageincr
easeinhisi
n c
ometax.
(
3mk s
)
18.a
)F
b
)I
in
dt
nac
h
e
ei
rta
n
i
ve
nwe
rs
eo
e
ft
kab
h
ema
usin
t
e
s
ri
x[ ]
.
sma
1 4
3 2
nbou g
h t60p e
nsand240p a
ir
sofsocksforatot
alofKs h4,200.Inth
(
2mk
eseco
n
s
)
dwe e
k,he
bought90p ensan d60p ai
rsofsocksforatotal
ofKs h1,800.Usin
gma t
ri
xme t
hod,fi
ndthepric
eo fea
chpenandap a
iro
f
socks
. (
4mk s
)
c
) Anotherbu s
ines
sma nb ought100pe nsand300p ai
rsOf soc
ks.Hewa sall
owedaiscountof10%o neachpenanda
dis
counto f3%wa sa l
l
o wedforeac
hp airofsocks
.
Calc
ulatethepercent
aged i
scountal
lowe donthecostofall
goodsboughtbythesecondbusi
nessman. ( 4mks)
19.I
nth
efi
gureb elo
w( notdrawntos ca
le)
,BQ=4. 9cm,QF=7. 5cm,BF=6.2cm, BD=13. 02cman d<F BD=22.50.

B 13.
02c
m
0 D
22.
5
6.
2cm

4.
9cm F
7.
5cm

F Q
ORMARKI
NGSCHEMESI
NBOX0724351706 Pa
ge|59
Ma
the
mat
ic
s121/
1,2

Cal
culat
etot wodecimalpl
aces;
a)t h
elengthF D. (2mk
s)
b)t h
es i
zeofa ngl
eBDF (3mk
s)
c)t h
es i
zeofa ngl
eFBQ (3mk
s)
d)t h
ea r
eao ftri
angl
eBDF (2mk
s)
20.Aboxhas32c omp ac
td i
sks
.4o f t
hema r
emusi
cdis
ks,12havecome
dymovi
esa n
dtheres
tar
eact
io
nmo v
ies
.
a)F i
ndthep robabi
li
tyofran
doml ypi
cki
ng;
i) amu s
icco mpactdis
k. (1mk
)
i
i)a nact
ionmo vi
eorac o
me dymo v
iecompa
ctdi
sk. (2mk
s)
b) Agirl
p i
ckstwoc ompactdi
sksfor
mt heb
oxatra
nd o
m, on
eatati
me,wi
th
ou tre
plac
ement
.Fi
ndthep
robab
il
it
yth
at;
i)o necomp actdi
skisthatofanact
ionmovi
eandtheoth
erist
hato
facomedymo vi
e (
3mks)

i
i)b o
thcompa
ctdi
sksa
reoft
hesa
metyp
e. (
4mk
s)
21.Compl
et
ethet
abl
egi
venbel
owbyf
i
ll
ingi
nth
eb l
anks
pac
es.
X 00 150 300 450 600 75 900 1050 1200 1350 1500 1650 180
0 0

3Co
s2x 3.0 2. 6 1. 5 0 -1.50 -3.00 -2.
60 -
1.50 2.60
0 0 0
2Si
n 1.0 2.0 1. 7 0 -1.00 -1.
73 -1.73 -1 1.
00
(2x+30) 0 0 3
b) Ont heg r
idprovided
, ando nt hesames etofaxes,dra
wt hegrap
h sofy=3Co s2xandy=2Si n(
2x+300)for00≤ X≤1800.
0
Takethes cal
efor1cmf or15 o nt heXa xi
sandf or2u ni
tsontheYa x
is. (
4mk s
)
c)u sey o
u rgraphtoe s
tima t
eth erangeo fval
ueso fXforwh i
ch;
i
) 3Co s2x=- 2.2 (
2mk s
)
i
i) 3Co s2x≤2Si n( 2x+300)
Giveyo uranswertot hen eares
tde g
ree. (
2mk s
)
22.a)Us ear ul
era ndap airofco mp a
ssesonlytoc onst
ructatr
ian
gu l
ardi
a gr
amr epr
esen
tin
gafiel
dPQR, suc
ht ha
tQR=9c m,<PQR=
450a nd<PRQ=300. ( 4mk s)
b)Me asurelengths i )PQ (
1mk)
ii
)PR (
1mk )
c)Dr awth elocusofp oint
se quidi
stantf
rome ndp oin
tsPa ndR. (
1mk)
d)Dr awt helocusofap ointAt hatmo v
ess uchthat∠QRA=∠PRA (
1mk )
e)Ac o i
nislo s
tinsi
det ri
ang l
ePQRwi thi
nar egionwh i
chisnearert
oo requald
ist
anc
ePt ha
ntoR.Sh adetheregi
onthe
c
o i
nc ouldbef o
un d. (
2mk s)
23.Thetablebe l
ows howsv alueso fxands omev al
ueso fyfort
hecurve;
3 2
y=x +3x –4x–12 i nt herang e-4≤X≤2.
a)Co mp l
etethetableb yfi
ll
ingi nthemissingvaluesofy. (
2mk s)
X - 4.0 -3.5 -3.
0 -2.
5 -2.0 -
1.5 -1.0 -0.
5 0. 0.
5 1 1. 5 2.
0 0
Y -4.1 -1.
1 -
2.6 -9.
4 -
13.1 -
7.9

b
)Ont hegri
dp rov
ide
d, d
rawthegrap
ho f
;
3 2
y=x +3x –4x–12 f or-4≤X≤2.
Usescal
e;Ho ri
zont
alaxi
s2cmfor1u ni
tandve
rti
cal
2cmfor5uni
ts. (
3mk
s)
c
)Us i
ngthegraphin(b)abo
ve,so
lvetheequ
ati
ons
;
3 2
i)x +3x –4x–12=0 (
1mk
)
3 2
i
i)x+3x–5x–6=0 (
4mk
s)
24.I
nfi
gurebel
ow, RAPisatangen
ttothecir
cl
eABCD.ADi sp
ara
ll
elt
othest
rai
ghtl
i
neCBP
,∠ADC=1140a
ndAB=BP

F
ORMARKI
NGSCHEMESI
NBOX0724351706 Pa
ge|60
Ma
the
mat
ic
s121/
1,2

Cal
cul
at
egi
vi
ngy
ourr
eas
onsi
nfu
ll
,t
hes
iz
eof
;
a)∠ABC (
2mk
s)
b)∠BAP (
2mk
s)
c)∠DAC (
2mk
s)
d)∠ACB (
2mk
s)
e)∠RAD (
2mk
s)

KAKAMEGANORT
HSUBCOUNT
YJOI
NTEXAMI
NAT
IONS

KCSETr
ia
lEx
am
FORM4
MATHEMATI
CSPAPER1
.
TI
ME:2½HOURS

Ans
wera
llq
ues
ti
onsi
nthi
sse
cti
on.

1. Wi
th
outu
sin
gac
alc
ula
toro
rma
the
mat
ic
alt
abl
eev
alu
ate
: (
3ma
rks
)

1 2 3 1
2 + o f3 -4
5 3 4 6
1 2 1 3
1 -2 ÷1 +3
4 5 3 4

2. Si
mpl
i
fy: (
3ma
rks
)

[ ]
3 -
1
2
a -ab
4 4
a-b
3. Astr
aig h
tlinepass
esthr
ought
hepoi
nt(-
3,-
4)an
disperp
endi
cul
art
othel
i
newh
osee
qua
ti
oni
s3x+2y=11a
ndi
nt
ers
ect
sthe
x=a
xisa ndy -
axi
satpoi
nt
sAa n
dBr e
spec
ti
vel
y.Fi
ndth
eleng
thofAB. (
3ma r
ks)
4. Eva
luateu s
ingsquar
es,cu
besandr
eci
pro
calt
abl
es. (
4ma r
ks)

[ 1
3
27.
+
56 (0.
1
]
3 -2
071)
2

1
5. Gi
vent
hat2-5x≤ ( +7)≤6-xa
x ndt
hatxi
sani
nt
ege
r,f
i
ndt
hes
umo
fth
esma
ll
esta
ndt
hel
ar
ges
tva
lueo
fx.
3 3
(3ma
rks
)
6. Mak
auandKil
onz
oli
ve20k
mapart
.Maka
uleave
sh o
mea t10:
00amandwal
kst
ome e
tKi
l
onz
owhos
tar
tedwal
ki
ngat9:
30amto
mee
tMakau.
Thespe
edofMak
auandKi
lonz
oareinther
ati
oof3:
4.I
fthe
ymetat11:
30a
mfi
ndt
hei
rsp
eeds.
(
3ma r
ks)
7. I
nthef
i
gurebel
ow,l
ine
sABandXYar
eparal
l
el.
A

4c
m 12c
m

C
Y B

F
ORMARKI
NGSCHEMESI
NBOX0724351706 Pa
ge|61
Ma
the
mat
ic
s121/
1,2
2
Ift
hear
eaofth
esha
dedreg
ioni
s36c m,f
indtheare
aoftr
iangleCXY. (3ma
rks
)
2
b
8. Giv
entha
tloga=0.
30andlogb=0.48f
i
ndt h
ev al
ueofl
og . (2ma
rks
)
a
0
9. Inth
efi
gurebe
lowOist
hecent
reo
fthec
irc
lediamet
erAB.<AXP=90, AX=4c
man
dPX=10c
m.Ca
lcu
lat
eth
era
diu
soft
hesemi
-
ci
rcl
e. (3ma
rks
)

10c
m

A xO B
10.T
heg
rad
ien
tfu
nct
io
nofac
urv
eth
atp
ass
est
hro
ught
hep
oin
t(-
1,-
1)i
s2x+3)
.Fi
ndt
hee
qua
ti
ono
fth
ecu
rve
. (
3ma
rks
)
11.Ev
alu
ate
: (
3ma
rks
)

()
1
1
1 3
256)2x36
x(
27
-
1
729)3x722
(
12
12.Esti
matet heareab o unde
db ythecurvey= x +1, x=0, x=3andthex- ax
isusin
gthemi d
-o r
dinat
erule
.Us et
hre
es t
ri
ps.
2
(3ma r
ks)
13.ABCDi sar homb u s.Theme asur
eo fa
n gl
eABCi s1500.Thedi
agona
lsoft
h erhombusint
ers
ectatE.T heshor
terdi
agonal
measur
es
10cm. Ca l
cul
a t
eth elengtho ft
heside
so ft
her hombustot
h ene
ares
tint
egerhencecal
cul
atetheareaoftherhombus.
(3ma rks
)
14.Threepolicepostsa resuchthatQisonab e
aringof2100and12kmf r
omPwh il
eRisonab eari
ngo f1500and8k mfromP.
(a
)Us ingas uit
ablesca le
, d
rawad iagr
amt orepr
es e
nttheabo
vesit
uat
ion. (2ma r
ks)
(b
)F romth escaledrawi ngde t
ermine:
(i
)t h
eb e
a r
ingo fQf romR (1ma r
k)
(i
i)t h
ed i
sta
n ceo fRf r
omQ. (1ma r
k)
2 2 2
15.As t
uden texpands( x-y) incor
rect
lyasx +y .Fi
ndh i
sper
cent
ageerr
orifheusedthi
sinc
o r
rectexpansi
onforx=4a ndy=-5.Giv
e
youranswe rcorrectto2d .
p. (3ma r
ks)
16.Ap ul
leyisma deu po ftwowh e
elsofr
a d
ii6cma nd9cmrespec
ti
vel
yandt hedi
sta
nce
b
etweenth ei
rcen t
re sis18c m.

I
fab
eltp
ass
esr
oun
dth
etwop
ull
eys
,fi
ndi
tsl
eng
th. (
4ma
rks
)

SECT
IONI
I

An
sweranyfi
veques
tio
nsinth
isse
cti
on.
17.Aci
rcul
arl
awnissur
round
edb yapa
thofun
if
ormwi
dt
hof
7m.
Thea
reao
fth
epa
thi
s21%t
hato
fth
ela
wn.
(a
)Calcu
lat
etherad
iusoft
helawn. (
4ma
rks
)
F
ORMARKI
NGSCHEMESI
NBOX0724351706 Pa
ge|62
Ma
the
mat
ic
s121/
1,2
(b
)Gi venfurtherthatthepathsurroundi
ngt helawni sfence
do nbo t
hs i
desb yb ar
bedwi reonp o s
tsatinter
val
so f10me t
resand11
me t
resont heinnera ndo ut
ersi
desrespecti
vely.Calcul
atethetotalnumb ero fpostsrequ
iredf orthefence. ( 4ma r
ks)
(c
)Ca lcul
ateth etot
alcosto ft
hep ost
sifonep ostcostssh105. (2ma r
ks)
18.Afrustumwi t
har egul
arp ent
agonalbaseissu c
ht hatit
stopisofs ide12c ma ndb ott
omi so fsi
d e24cm. I
fitsperpe
ndicul
arhei
g hti
s
20cm. Ca l
culat
e:
(a
)T helengtho ftheslantedge. (5ma r
ks)
(b
)T hevo l
umeo fthefr
u st
um. (5ma r
ks)
19.FourtrucksA, B,Ca ndDt a
ke10d aystot r
ansport42,000b agso fma i
zet oad epot.Howe ver,tr
ucksAa ndBt oget
hertake30d ays
totra
n s
p ortthesamen umb erofbagswh il
etrucksCa ndDt ogethe rta
ke15d a
ys. Tr
uckAc arr
ies1½t i
mesthen umberofbagsB
carr
iesan dCc arri
es14/5timesasmu cha sD.
(a
)De t
erminet henumb erofb a
gso fmaizetra
nsportedb yeachtru
c kp erday. (5ma r
ks)
(b
)Al lt
het rucksA, BCa ndDwo rktogetherfo
r5d a ys,af
terwhichtru c
kCa ndDa rewithd
ra wn .Aan dBwo rktoget
herforanot
he r5
daysaft
e rwh i
cht r
uckAb r
eaksdown .Ho wlongd oestruckBtak etoc omp le
teth eres
tofth erema i
ningbags?
(5ma r
ks)
20.Eunicebo ughtsomeo r
an g
eswo rt
hKs h45, whil
eSh a r
ons p
entth esamea mo untofmo neyb utboug htt
heo ra
n g
esatad i
scou
n tof
75c ent
sp ero r
an g
e.
(a
)I fEuniceb oughtano rangeatShx ,writ
ed ownas imp l
if
iedexpr
e ssi
onfo rthetotalnumb ero forangesboughtbyEu ni
ceandSh aron
.
(3ma r
ks)
(b
)I fSharonb ought2mo reorangesthanEu ni
ce.Findh owmu che achs pento nano range. (5ma r
ks)
(c
)F indtheto t
alnumb erofo ra
ngesb o
u ghtbyEu ni
cea ndSh ar
on. (2ma r
ks)

21.(
a)T
hefi
gur
eshowsav
elo
cit
yti
megra
phofa
no b
jec
twhic
ha c
cel
er
ate
sfr
omr
estt
oav
elo
cit
yVm/
sth
end
ece
ler
ate
sto
r
esti
nat
ota
lti
meof54seco
nds
.Ift
hewho
lej
ourne
yis810m,

m/
s

T
ime(
sec
) 54

(i
)F i
ndt heval
u eofV. (
2ma r
ks)
s12/ 2
(i
i
)F i
ndt hedeceler
ati
ong i
venth eini
ti
alacce
ler
ati
o ni 3m/ s . (
2ma r
ks)
(b
)Ab u slef
ttownxa t10:45a ma ndtrav
e l
l
edtowa r
d stownYa tana v
eragespe
edof60km/hr.Acarle
ftto
wnXa t11:15amonthe
sa
med ayandt r
avel
ledalo
n gthes a
mer oadatana veragespeedo f100km/ h
r.Thedi
st
anc
eb e
tweentownXandtownYi s500km.
(i
) Determinethetimeofdaywh enthecarover
tookth eb u
s. (
3ma r
ks)
(i
i
)Bo thv e
hicl
esc ont
inuedtowa r
dstownYa tthei
ro ri
g i
nalspeeds.Findhowlongtheca
rhadtowai
tintownYbef
o r
eth ebusar
ri
ved
.
(
3ma rks)
2
22.Theveloci
tyo fapart
icl
etsec ondsaf
terpass
ingaf i
x edpointO, i
sg i
venbyV=a t +b
tm/s,wher
eaa ndbar
eco ns
tants
.Giv
enthat
i
tsveloci
tyis2m/ swh ent=1s ecanditret
urnsto0wh e
nt=4. 5secs,c
alc
ula
te;
(a
)T hevaluesofaa ndb. (
4ma r
ks)
(b
)He ncefind;
(i
)T hevaluesoftwh entheparti
c l
eisins
tant
aneouslya tres
t. (
2ma rks)
(i
i
)T hetotaldi
stancetr
avell
edb ythepart
icl
ed u
ringthef i
rst4s ec
o n
d s. (
2ma rks)
(i
i
i)Thema xi
mu mv elo
cit
ya t
tai
ne dbythep a
rti
cl
e. (
2ma r
ks)
2 3
23.(
a)Co
mpl
et
eth
eta
bleb
elo
wfo
rth
efu
nct
io
ny=-
4–6x+3x+2x. (
3ma
rks
)

x -
4 -
3 -
2 -
1 0 1 2
y

2 3
(
b)Drawth
eg r
aphofy=-4–6x+3x+2xfo
rva
lue
sfoxf
rom-
4to2. (
3ma
rks
)
(
c)Useyou
rg r
apht
osol
ve.
3 2
(
i) 2x+3x –4x–2=0 (
2ma
rks
)
F
ORMARKI
NGSCHEMESI
NBOX0724351706 Pa
ge|63
Ma
the
mat
ic
s121/
1,2
3 2
(
ii
)4x+6x-12x–8=0 (
2ma
rks
)

24.Ap
ara
ll
el
ogr
amOACBi
ssu
cht
hatOA=a
,OB=b
.Di
sth
emi
d-
poi
nto
fBCOE=h
OCa
ndAE=k
AD.

(
a)Expre
sst
hef
ol
lo
win
gint
ermso
fa,
b,ha
ndk
.
(
i) OC (
1ma
rk)
(
ii
)OE (
1ma
rk)

(
ii
i
)AD (
1ma
rk)
(
iv
)AE (
1ma
rk)

(
b)F i
ndtheval
ueso
fhandk
. (
4ma
rks
)
(
c)De t
ermin
ethera
ti
os:
(
i) AE:ED (
1ma
rk)
(
ii
)OE:OC (
1ma
rk)

KAKAMEGANORTHSUBCOUNTYJ
OINTEXAMI
NAT
IONS
FORM4
MATHEMATI
CSPAPER2
.
TI
ME:2½HOURS
1. Eval
uat
ewi
th
outu
sin
gtabl
eso
raca
lcu
lat
or (
4ma
rks
)

1
3
3
(
0.000125) x 0.
0049 x 3.
9x10

0.
325 x 0.
0036
2. F
indt
hev
alu
eof
thet
ermi
nde
pen
den
tofxi
nth
eex
pan
sio
nof (
3ma
rks
)
6
2 1
3x +
3x

3. Si
mpl
i
fyt
hef
ol
lo
win
gg i
vi
ngy
oura
nswe
rinthes
imp
les
tfo
rmp
oss
ibl
e.
2 - 5
(
3ma
rks
)
6+ 3 7- 5

4. Wi
th
outu
sin
gta
ble
sorac
alc
ula
tore
val
uat
e (
3ma
rks
)

Ta
n225 -c
os330
Si
n210 +cos840

2 2
5. Gi
vent
hat a= b+ b+c ma
kect
hes
ubj
ecto
fth
efo
rmu
lar
. (
3ma
rks
)

6. T
woma
tri
cesAa
ndBa
res
ucht
hat

A= K 4 B= 1 2
3 2 3 4

Give
nt hatthed e
ter
mi nantofAB=4f indtheimageoftr
iang
leABCwh er
eA=( 2,0)
,B( 3,2)andC(3,4)u n
derst
ret
ch,
st
ret
c hfactorK, par
a ll
elt
otheXa x
is. (
3mark
s)
7. a. F indthep os
iti
onv ec
torOCo fthecent
reo f
ac i
rcl
eC,wh os
ee quat
ioni
s
2 2
2x +2y +4x-6y-26=0 (
2mark
s)
b.I ft
h ecirclepassesthroug
hP( 3,2),us
ev ec
tormethodtofi
n dt hedi
amete
rofthecirc
le. (
2mark
s)
8. Thesu mo fthed igi
tsinathreedi
g i
tnumberisnine.Thetensdi
g i
ti shal
fth
esumo fthesumo ft
heothertwoandthehund
redsd
igi
t
i
sh al
ftheu nitsdigi
t. Fi
ndthetot
alvalu
eo ft
hen umber. (3ma r
ks)
n
9. Give
nt hatyi sinve
rs e
lypropor
ti
onaltox a n
dβi st
heconstanto fprop
ort
iona
li
tyandthatx=2,wh eny=12, andx=4,wh en
y=3, f
i
n dthev aluesofna ndβ. (
4mark
s)
3
10.Fin
dt hee x
ac tareao ft
her e
gionboundedbythec ur
vey=9x-x andthexaxis
. (
4mark
s)
11.Inthefigureb elow,RPa ndRQa reta
n ge
ntstothecir
cl
ec e
ntreO, radiu
src m.OQp ro
d ucedmeet
sPRp roduc
edatT.QT=12cm
andQR=5c m

F
ORMARKI
NGSCHEMESI
NBOX0724351706 Pa
ge|64
Ma
the
mat
ic
s121/
1,2
Ca
lcu
lat
eth
era
diu
soft
hec
ir
cl
e. (
3ma
rks
)

R
5c
m

T 12c
m Q O

12.Th
efig
urebe
lowre
pre
sen
tsari
ghtpy
ramidwit
haver
texVa
ndar
ect
ang
ula
rba
se,
ABCD.VA=VB=VC=VD=40c
m.
AB=30cmandBC=22cm.Xi sth
emi d
-poi
ntof
BC.
Cal
cul
at
eth
es i
zeo
fth
eangl
ebet
weenplane
sVBCa n
dABCD (3mk
s)

C
D
22c
m

A 30c
m B

13.Gi
vent
hat a = 3i-2j+3k a
nd
b = 2i-4j-3k

F
ind 2a-3b (
3mk
s)
2
14.I
f25x +k+9 i
saper
fe
ctsqu
arefi
ndx (
2mks
)
15.T
hef
ig
ureb
elo
wsh
owsaci
rcl
ecent
reOto
uch
ingt
hev
ert
ic
esA,
B,Co
ftr
ia
ngl
eAB=8.
8cm,
BC=5.
4cma
ndAC=9.
2cm.

8.
8cm
9.
2cm

B C
5.
4cm

Cal
cul
atet
her
adi
uso
ftheci
rcl
etot
hene
are
stwhol
enumb
er. (
3mk
s)
16.XAYisata
nge
ntt
othec
irc
leABCD.ADispa
ral
l
elt
oth
estr
ai
ghtl
i
neCBY.An
gleADC=114°,
andAB=BY

C
F
ORMARKI
NGSCHEMESI
NBOX0724351706 Pa
ge|65
Ma
the
mat
ic
s121/
1,2

114° B
D

T
A
Cal
cul
at
ean
gle
s
i
. ABC (
1mk
)
i
i
. BCA (
1mk
)

17.Thefoll
owi ngt ablesho wsi nd i
vi
d ualra
teso fincomet ax
IncomeK£PA Rate(sh.PerK£)
1–4512 2
4513- 9024 3
9025–13536 4
13537–a bove 5
Mr.Ka r
iukilivesinac omp a nyho useforwh ichh epaysan ominalrentofKsh.610permo nth.Fortaxat
ionpur
p os
e,h i
sb as
icsal
aryis
i
n cr
easedb y15%.Hei si nsureda ndpa yssh.1200a spremiumsp ermo nthandc l
ai
msi nsur
ancerel
iefofK£36p erannu m.Hea ls
o
cl
aimsaf ami l
yr eli
efo fsh .
660p ermo nth.I nadd i
ti
on,heisame mb ero faco-oper
ati
vesociet
y,t
owh ichheremitsKsh.1500p er
mo nt
h ,asshares .IfMr .Kariuki
’sP.A.Y.Ei sksh.2400p e
rmo nt
h,cal
c u
latehisnetsal
aryinshil
l
ingspermo nt
h .
(
10ma r
ks)
18.a.u si
n gar ulera ndap airo fc
o mp as
seso nlyco ns
truc
t
i
. T ri
an gl
eABC, sucht hatAB=9c m, AC=7c ma nd<CAB=60° (2mks)
i
i.T helo cuso fP, su cht hatAP≤BP (2mks)
i
ii.T helo cuso fQs uc ht hatCQ≤3. 5c m
i
v.L ocuso fRs uchth ata ngleACR≤a ngleBCR (2mks)
b.F indthea reao fth er e
g ionsa t
is
fi
edb yb othPa ndQ (
2mk s
)
19.Point
sD( 0°, 24° E),E( O°, 21°W),F (60°S,120°W), G(60°S,110°E)a remarke
di nag l
oberepre
senti
ngthee ar
thwithradi
u s
22
=0. 7m.( Takingπa s )
7
a.F indthel eng t
ho fth ea rcDE. (
3mk s
)
b.I fAist hec entreo fthel at
it
u de60°S, andBi sthecentr
eo ft
h el
a t
it
u deO°f i
nd
i
. thelengt
hAB (3mks)
i
i
. theareao ft
h ema j
ors ectorAF G (4mks)
20.Inag roupo f40p eople, 10a reh eal
thyan dev e
ryp er
sono ft
h ere
ma i
n i
ng30h asei
therhighbloo
dp res
sure,
ah ighlev
el of
chol
e st
erolorb o t
h .15h a
v ehighb l
o odp r
e s
surea nd25h avehighlevelofcholes
ter
ol.Ifap e
rsonissel
ect
eda tran
d omf r
omt hi
s
group,wh atisth eprob ab i
l
itytha the/s
h e
a. Ha sh ighb loodp res sureo nl
y (
4mk s
)
b. Ha sh ighl evelofc ho l
esterolonly (
2mk s
)
c. Ha sh ighb loodp res sureandh i
ghlev elofcholes
ter
ol (
2mk s
)
d. Ha se it
h erhighb loo dp ress
u reorh i
g hlevelofchol
ester
o l (
2mk s
)
21.Threecon secuti
vet ermsi naG. Pa re 32x+1, 9xa nd81r especti
vely
.
a. Ca lculat
et hev alueo fx (
2mk s
)
b.F indthec ommo nr a ti
oo fthes er
ies. (2mks)
c. Ca lculat
et hes umo fth efir
st10t ermso f t
hes e
ries
. (
3mk s
)
d. Gi venth atthe5tha n d7tht e
r mso ft
h eG.Pi n(a)aboveformthe1sttwoc on s
ecut
iveter
mso fanA. PCa l
culat
ethesu m
ofthe1st20t ermso ftheA. P. (3mk s)
22.Twov ari
ab l
esya ndxa reb eli
evedtob ere l
atedbyt heequati
ony=x+a xb.T hetabl
ebelows ho
wst hecorr
espondingvaluesofxand
y.

X 1 1.
5 2 2.
5 3 3.
5 4
y 7.
54 9.
33 11.
00 12.
59 14.
12 19.
90 27.
23

a
. Bydra
wingasui
ta
blel
ineg
raph,e
sti
mat
etheval
uesofaandb. (
9mk
s)
b
. Wri
tedownthee
quati
onconn
ecti
ngyandx. (
1mk
)
23.T
hemar
ksobt
ain
edbyfi
ft
ycand
idat
eswerer
eco
rdedinth
eta
blebe
low.
F
ORMARKI
NGSCHEMESI
NBOX0724351706 Pa
ge|66
Ma
the
mat
ic
s121/
1,2

Mar
ks 0-9 10-19 20-29 30-39 40-
49 50-59 60-69
No
.ofca
ndi
dat
es 6 8 12 9 7 5 3
a. Drawac umulat
ivef
requencygraphandu seittoesti
ma t
e. (3mks)
i
. Me di
an (1mk)
i
i. Qu ar
ti
led ev
iat
ion (
2mks)
i
i
i. Theperc
entagen u
mb erofcandi
date
sfail
ingifthep as
sma rkwa s25ma rks. (
2mks)
i
v.T herang eofma r
ksscoredbythemiddle30%o fthecandidat
es. (
2mks)
24.Atheat
rehasas eat
ingcapaci
tyof250p eopl
e.T hec hangesaresh.100foranordin
aryse
atandsh.
160foraspec
ial
seat.I
tcost
s
sh
.16,000tos tag
ea
Showandt hetheat
remu stmakeaprofi
t.T herearen e
vermo rethan200o rdi
narysea
tsandf
orashowtotakepl
ace,a
tlea
st50
or
d i
nar
yseatsmu stbeoccupi
e d
.T henumb erofsp e
cia
l s
eatsisalwaysles
sthantwic
ethenumberofo
rdi
n a
rysea
ts.
a.T a
k i
ngxtob ethen umberofordi
naryseat
san dyt henumb erofspecial
seat
s,wri
tedownallt
heine
qual
it
ies
r
epre
sentingtheinf
ormati
ona bove. (
4mks)
b. Onag r
ap hpaper,s
ho wthere
gionrepresent
e dbythea bo
v ei
n equal
it
ie
s. (
4mks)
c. De t
erminethen umberofseat
so f
e a
chtypeth atshoul
db eb o
o kedinorde
rtoma x
imi
zeprof
i
t. (
2mks)

L
ANG’
ATA

121/1
MATHEMAT
ICS
Pape
r1
PREMOCK.
Ti
me:2½Hour
s

SECT
IONA
An
swe
ral
lqu
est
io
nsi
nth
iss
ect
in
o

1. Gi
vent
hat =Mm F
indt
hev
alu
eofM (
3ma
rks
)

2. F
indt
her
ang
eofv
alu
est
hats
ati
sf
yth
ein
equ
ali
ty
. (
2ma
rks
)

3. De
ter
min
eth
eva
lu fxf
eo orwh
icht
hema
tri
xbe
lowi
sas
ing
ula
r. (
3ma
rks
)

4. So
lvet
hes
imu
lt
ane
ouse
qua
ti
on. (
3ma
rks
)

5. T
hef
i
gur
ebe
low(
notd
rawnt
osc
ale
)sh
owst
hec
ros
s-s
ect
io
nofame
tal
baro
fle
ngt
h3me
tre
s.T
heya
ree
qua
lse
mi-
ci
rc
les
.

De
ter
min
eth
ema
sso
fth
eme
tal
bar
sink
il
ogr
amsi
ft
hed
ens
it
yoft
heme
tal
is8.
87g
/cm3. (
3ma
rks
)

6. T
hee
qua
ti
ono
fal
i
nei
s. F
ind
;

a
)T h
eg r
adi
entoft
heli
ne. (1mar
k)
b
)T h
ee q
uati
onofali
nepassi
ngthr
oughthep
oint(
1,2)a
ndp e
rpe
ndic
ulartot
h egi
venli
ne. (
2ma
rks
)
7. T
hefi
gurebel
owshowas o
li
dc oni
cal
frus
tumwi
thtopandbot
tompar
all
elci
rcul
arpl
anesof9pcm2a n
d16p
cm2r
espec
ti
vel
y.I
fth
e
v
olu
meo ft
heconefr
omwh ic
hthefrus
tumwascutof
fi
s320c 3
m, cal
cul
ateth
evo l
umeo ft
hefr
ust
rum.
(
4mark
s)
F
ORMARKI
NGSCHEMESI
NBOX0724351706 Pa
ge|67
Ma
the
mat
ic
s121/
1,2

8. T
hep
osi
ti
onv
ect
orso
fpo
int
sx,
yan
dza
re, ,
and r
esp
ect
iv
ely
.Ift
he

Po
int
sx,
yan
dza
rec
oll
i
nea
r,f
i
ndt
hev
alu
eof
t. (
3ma
rks
)

9. I
fT= ,
mak
ePt
hes
ubj
ecto
fth
efo
rmu
la. (
3ma
rks
)

10.Twofl
agpost
s4. 7mand6.5mi nhei
ghtst
andadi
st
anceof8.
1mapa
rt,
andast
rai
ghtro
pejo
insth
eirt
ops
. Find:
a)T hele
ngtho ft
herope. (3marks)
b)T heangl
eo fdepr
essi
onof t
herope. (
1mar
k)
11.Ki
pyeg
ob oug
h taTele
vis
ionsetandmarke
dita
tSh.32,
500.Hel
at
ersol
dth
eTVsetat10%dis
counta
ndsti
l
lmad
eaprof
itof30%.
Cal
cul
atet
hep ri
ceatwhic
hKip y
egosol
dtheT.
Vset. (
3mar
ks
12.Fi
ndtheva
luesofwh ic
hs a
ti
sfythet
ri
gonomet
ri
cequat
io
n.
2Si2
n-5Co s+1=0f or (3marks)
13.TheL.C.Mofthr
een umbersis7920a ndthei
rG.C.Dis12.Twoofthenumbersare48a nd264. Usingf
actornota
ti
o nfi
ndthet
hir
d
numberifoneofi
tsfact
orsis9. (
3ma r
k)
14.Twosimil
arcanshavedif
fer
en theigh
ts8cma ndtheot
herone10cm. i
fth
esurfa
cea r
eaofthelargercanis480cm2, fi
ndth
es u
rfa
ce
ar
eaofthesma l
l
erc a
n. (3marks)
15.Twotypesofcof
feeareavai
lableinashop.GradeAcostSh.150perki
logr
ama ndg r
adeBc ostSh .
160p erki
logr
am. IfGr
adeAa nd
Baremi x
edintherat
io2:x,findxs ucht
hatthesel
li
ngpri
cewil
lgi
v ea25%p r
ofiti
fthemixt
urei ssol
datSh .
1950f or10kg.
(
3ma rks)
16.Atankissuppl
ie
db ytwop i
pes. T
h el
arge
rp i
pecanfi
llt
heta
n ki
n10mi nute
slessthanth
es mall
erp i
pe.Runni
n gthetwopip
es
t
ogetherfi
l
ltheta
n ki
n12mi nutes.Fi
ndtheti
metakenbyeachpi
pes e
parat
el
ytof i
l
lthetan
k . (4marks)

SECTIONB
AnswerANYfi
vequ
est
io
ns
17.Inthefi
gur
ebel
owABCDisac
ycl
i
cqu
adr
il
at
era
lan
dth
ata
ngeABD=420,
l BAC=580a
ndDBC=360.

Gi
vin
gr e
asonsf
i
ndthev
alu
esof
;
a) AngleDAC. (
2ma
rks
)
b) AngleADB. (
2ma
rks
)
c) AngleACB. (
2ma
rks
)
d) AngleCDB. (
2ma
rks
)
e) AngleCEB. (
2ma
rks
)
18.Usi
ngthesameaxe
sahor
iz
ont
als
cal
eof
1cmt
ore
pre
set300a
n ndv
ert
ic
als
cal
eof4c
mre
pre
sen
t1u
nit
.
a)F il
li
ntheta
ble. (
2ma
rks
)

Drawthegr
aphofy=Sin(x-15)an
dy=2Cosxo nt
hegr
aphpap
erpr
ovi
ded
, (
5mar
ks)
b) Useyourgr
aphs
,sol
vet
hee qu
ati
ons
:
i
) Si
n(x-15)=0.5 (
1mar
k)
i
i) Si
n(x-15)=2Cosx. (
2mar
ks)
19.Api
rat
eb o
atsai
l
edfor
mp or
tAo nabear
ingo
f 0
050 atas
pee
dof112km/
h,fo
r2½ho
urst
opo
rtB.
Fro
mBi
tch
ang
esit
scou
rse
F
ORMARKI
NGSCHEMESI
NBOX0724351706 Pa
ge|68
Ma
the
mat
ic
s121/
1,2

andt
ravel
edo nb e
ari
ngo f1700ataspeedof75km/hforhour
stowar
dspor
tC. Fr
omCittr
avel
edt
oportD.Disonab ea
ri
ngof
0
130 and160k mf r
omA.
a) Usi
ngas ca
leo f1cmtorep r
ese
nt40km,drawad i
agra
ms howi
ngthepos
it
io
n soft
hep
ortsA,
B,CandD. ( 5mark
s)
b) Useyourdi
agramt ofi
nd;
i
) T hed i
sta
n ceofCfromD. (1mar
k)
i
i)T heb e
aringofCf r
omD. (1mar
k)
c) Amarin
ep oli
cep os
tlea
ve spor
tAtoint
erce
ptthepir
ateboata
tMa si
tmovesfr
omBtoCi nth
eshor
tes
tti
mep os
si
ble.
i
) Ho wf arfr
o mp o
rtAwi l
lthet
wob oat
sme e
tatM? (2mar
ks)
i
i) Ift
h eboatsmeetaft
er2h our
s,whati
sthespeedofth
ema ri
nepo
li
ceboat? (1mark
)
20.Thet
ablebelowr e
pres
entsma r
ksscor
edinma t
hs.

Usethea bovefrequencytab letoc alculat


e :
i
) Me a n (
3marks
)
i
i) Stan dar
dd eviat
ion. (
4marks
)
i
) Up perq uart
il
e . (
3marks
)
21.Thecashp ri
ceo fatractorisKs h.1. 8milli
on.T hetract
o rcana l
sob eboughtthroughab ankloan.T
hec u
stomeri
ssupposedt
orais
ea
downp aymen tof20%o fthec ashp ri
ce.T hebalanceisp ai
dt hroughtheloana t10%c ompoundint
erest
.Simbab o
ughtthet
ract
or
t
hrought heloan.
a)i)Ca lculat
et hea mounto fmo n eyh eb orrowed . (
3marks
)
i
i)Simb apaidt heloanin4y ears.Ca lculat
eth etotalamo unthep ai
dforthetractor
. (
3marks
)
b)F i
ndh owl ongh ewo uldtaket ore payt heloani fhepaidat otalofSh.2,276,640. (
4marks
)
22.a) Af actor
yma nufact
ures100, 000s we e
tsinthef i
rsth ou
r. Therat
eo fpr
o ducti
ond e
creas
esby20%e veryhour
.
i
) Ca l
c ul
atethen umb ero fswe etsp roducedinth e4thh our. (
3marks
)
i
i) Ca l
c ul
atethet ot
alnu mb ero fswe etsp r
odu c
edi nth efi
rst5h our
s. (
3marks
)
b)T h ee l
eventhtermo fa nAr it
hme t
icp rogr
e s
sionisfo urti
me sthesecondterm. Thesumo ft
hefir
stsev
en
t
ermso fthesa mep rogr
es si
o nis175. Find;
i
) T h efir
stterm. (
3marks
)
i
i)T h ec o
mmo nd i
ffer
en ce. (
1mark)
23.Theprobabil
itythatap upilgo estos ch oolbyab oda -
bod ais 2/ 3 a ndb yama tat
uis1/ 4ifheusesaboda-b
odatheproba
bil
it
y
t
hathei slat
ei s2/ 5andi fh eu s
e sma t
atuthep robabil
ityofb e
inglat
eis 3/ 10. I
fheu s
esothermeansoftra
nspor
ttheprob
abil
i
tyof
bei
nglateis3/ 20.
a) Dr awat reed i
agramt or epresen tthisinf
orma t
ion. (
3marks
)
b)F indt hep r
ob abi
li
tytha thewi llb elateforschool. (
3marks
)
c)F indt hep r
ob abi
li
tytha thewi llb elateforschool i
f hedo e
sn otuseama tatu. (
2marks
)
d) Wh ati
sth ep r
obabili
tyt hathewi l
ln otbelatetos chool? (
2marks
)
24.Thet
ab l
eb elows howst hev alueso fxa nds omev alueso fyforthecurvey=x 3+3x 2–4x–12i nthera
nge
a) Co mp l
etethet abl
eb yfilli
ngi nt hemi ssi
ngv al
u e
so f y. (
2marks
)

b) Ont
hegr
idprovi
deddr
awtheg
rapho
fy=x3+3x2–4x–12int
hera
nge. (
4ma
rks
)
c
) Bydr
awi
ngas
u i
ta
blest
rai
ghtl
i
neonth
esameg
rids
olv
eth
eequa
ti
o 3 2
nx +3x –5x–6=0. (4ma
rks
)

L
ANG’ ATA
121/2
MATHEMAT I
CS
Pape
r2
PREMOCK.
Ti
me:2½Ho ur
s
SECTIONA(Answe
rALLque
sti
onsi
nthi
sse
cti
on)
1. Eval
uatewi
th
outus
ingma
the
mati
cal
tab
leso
rcal
cul
at
or
(
3ma
rks
)

2. Gi
vent
hatA=4+ a
nd a
ndt
hat whe
rec,
dandeareco
nst
ants
.Fi
ndthev
alu
esofc
,dande.
(
3mark
s)
3. Anunder
grou
ndwate
rtankisint
hes
hapeofah
emi
sph
e r
ic
albowl
.Gi
vent
hatt
hevo
lumeoft
hetan
kis19,
404 l
it
res
,fi
ndt
he
ra
diu
softhet
anki
nme tr
es. (
3mark
s)
4. Sol
vefo
rxinthet
ri
gonomet
riceq
uat
io
n.
FORMARKINGSCHEMESI NBOX0724351706 Page|69
Ma
the
mat
ic
s121/
1,2
Log2(3x+1)=2+L og2(
x-1) (
3mark
s)
5. T
hep
robabi
li
tyofaschool
winni
n gi
n 2/3fo
otbal
li
sandwinni
nginvo
ll
e y
bal
lis¼. F
indthep
ro b
abil
it
yo fwi
nni
ngatl
eas
tone
g
ame
. (
3mark
s)
6. T
hee
leve
nthter
mo fanA.Pisf
ourti
mest
h esec
ondter
m.If
thesumofthefi
rs
tseve
ntermsoft
h eA.Pis175 f
i
ndthe
i
) t
hefir
stter
m (
2mark
s)
i
i
)T hecommo ndif
fe
renc
e (
1mark
)
7. T
hed
atabelo
ws howstheageof10st
ude
n t
spick
edatra
ndominasec
ond a
ryscho
ol6,11,13,14,8,7,12,20,Pan
d9.
I
f;

i
) De t
ermin
etheval
ueofP (2ma
rks
)
i
i)F i
ndthest
anda
rddevi
at
iono
fth
edi
st
ri
but
io
n. (
2ma
rks
)
8. Sol
vet
hesimul
ta
neouseq
uati
ons (3ma
rks
)

9. T
heme
asu
reme
ntsa=6.
3,b=15.
8,c=14.
2ha
vema
ximu
mpo
ssi
bl
eer
ror
s1%,
2%a
nd3%r
esp
ect
iv
ely
.Fi
ndt
he

ma
ximu
mva
lueo
f (
2ma
rks
)

10.Thepri
ceo fan ewcarifSh.800,000.00.
Ifitdepre
ciat
esatacon
s t
antr
atetoSh.550,000wi
thi
n4yea
rs.
Findtheannual
rat
eof
depr
eci
atio
n . (3mark
s)
11.Atange
n tf
r oma next
ernalpoi
ntA( 7,
5)me e
tsac i
rc
lecent
reOwh oseequat
ionisa t p
o i
ntT,f
in
dt hel
engt
h
oft
hetange ntAT. (3mark
s)
12.Mwangia ndOt i
enoli
ve60k mapa r
t.Mwangileav
esh i
shomeat7.00a.
m,cycli
ngtowardsOt
ie
no’
sho
useat20km/ h
.Otien
oleav
es
hi
shomea t8. 00a.
mc ycl
ingtowardsMwa ngi
’shouseat8k
m/ h
.
a) Atwh attimed i
dtheyme et
? (2mark
s)
b) Ho wfa risth
eme eti
ngp o
intfromMwa ngi’
shouse? (1mark
)
13.Expa
nd upt
othefourt
ht er
m. (2mark
s)

Henc
eu s
etheab
oveex
pans
iont
oeval
uat
e(1. 6c
96) o
rrec
tto4d
.p. (
2ma
rks
)
14.Fi
ndthera
diu
sandce
ntr
eofaci
rcl
ewhoseeq
uat
ionis
(
3ma
rks
)

15.IfPvar
iesd
ire
ctl
yasRa
ndi
nve
rse
lya
sth
esquar
ero
otofQ.F
indt
hep
erc
ent
agec
han
gei
nPi
fRi
sin
cre
ase
d b
y40%andQ
dec
rea
sesby36%. (
4mar
ks)
16.Sol
vefo
rxin, wi
th
outus
ingt
abl
es. (
3mar
ks)

SECTI
ONB( AnswerANY5q ues
ti
onsinthissec
tion)
17.Aquadr
il
at
eral
ABCDh asvert
ic
esA(4,-4)
, B(2,-4),
C( 6,-
6)andD(4,-2)
.
a) Onthegri
dp r
ovi
ded,dr
awthequadri
late
ralABCD. (2ma
rks
)
b) ist
heimageofABCDu nderposi
tivequa
r t
ert
urnab
o u
ttheori
gi
n.Ont
hes
ameg
ridd
raw th
e
i
ma g
e andst
ateit
sco-
ordi
nate
s. (
3ma
rks
)
c) i
stheimageof unde
rthetr
ansf
ormati
ongiv
enbyT=

i
) De ter
minet heco-o
rdi
natesof. (3mark
s)
i
i) Onthes ameg ri
ddrawtheq u
adri
l
ater
al. (2mark
s)
d) Deter
mineas ingl
ema t
rixt
hatmapsABCDo nto. (2mark
s)
18.Thet
abl
eb e
lows howstheincometaxrat
esdur
ingac e
rta
inye
ar.
Mon t
hlyTax
able T a
xrateSh.
Per(K£)
income(K£)
0–506 2
507–1012 3
1013–1518 4
1519–2024 5
2025–2530 6
2531–3036 7
3037–Ab ove 7.50
Abankcl
erkea r
nsab asi
csalar
yofSh.55,
240andah ouseal
l
owanceofSh.30,
600permonth
.Shei
sal
sop
aidt
axa
bleall
owanc
e
amount
ingtoSh .15,
760p ermonth.
a) Cal
cul
ate
:
i
) He rmo nt
h l
ytaxabl
eincomeinK£. (2mark
s)
i
i)T hegrosstaxpayabl
e. (6mark
s)
b)T hecl
er
kise nti
tl
edtoap e
rsonalt
axrel
ie
fofKsh
. 1312permont
h.Cal
cula
teherne
tmo nt
hl
ytax. (
2mar
ks)

F
ORMARKI
NGSCHEMESI
NBOX0724351706 Pa
ge|70
Ma
the
mat
ic
s121/
1,2

19.T
hef
i
gur
ebe
lowi
sat
ri
ang
leXYZ
,ZY=13.
4cm,
XY=5c
man
dan
gl 70.
eXYZ=57.

Calcul
ate
a
)L engthXZ. (2ma
rks
)
b
) AngleXZY. (2ma
rks
)
c
)T hecircu
mradi
uso ft
ri
ang
leXYZ. (2ma
krs
)
d
)I faperpend
icu
larisdr
opp
edfro
mp oin
tXtocutZ
YatM.Fi
ndt
her
ati
oMY:ZM. (
2ma
rks
)
e
)F indtheare
aoftri
angl
eXYZ. (2ma
rks
)
20.I
nth
efigur
eb e
low,Eisth
emidpoi
ntofAB.OD:OB=2:5an
dFist
hepoi
nto
fin
ter
sec
ti
ono
fOEa
ndAD.

⃗ ⃗
Giv
enthatOA=aa ndOB=be x
pre
ssint
ermsofaandb.

a)i ) OE (1ma
rk)

ii
) AD (1ma
rk)
⃗ ⃗ ⃗ ⃗
b) Give
nf ur
the
rth
a tAF=t
ADandOF=h OEwh e
reta
ndha
res
cal
ar
s,f
i
ndt
hev
alu
eso
fth
esc
ala
rsta
ndh
. (5ma
rks
)
c) Showt h
atthep
o in
tsO,
FandEarec
oll
ine
ar. (
3ma
rks
)

21.Th
ed i
agr
ambel
owshowsacro
ss-s
ect
ionofabot
tl
e.T
h el
owerpar
tABCisah
emi
sph
ereofr
adi
us5.
2cmandt
heuppe
rpa
rti
sa
f
rus
tr
umo fac
one.
Thetopr
adiusoft
hefr
ust
rumisoneth
irdoft
hera
diu
soft
heh
emisp
here
.Thehe
mis
phe
repar
tisc
ompl
et
el
yfi
l
led
wi
thwate
rassh
owninthed
iagr
am.

Whent
hec on
tai
nerisi
nver
ted
, t
hewaterno
wc omple
tel
yfi
ll
sonl
ythefr
ust
rumpar
t.
a) Det
erminet
heh ei
ghto
fthefr
ustr
ump ar
t. (5ma
rks
)
b)F i
ndthesur
facear
eaofthefr
ust
rumpartoft
hebott
le
. (
5ma
rks
)
22. T
hefi
gur
eb el
owrepres
ent
sas qua
rebase
dp yr
amidwit
hequi
la
ter
altr
ia
ngl
es,AB=5cm.

Cal
cul
at
et h
e ;
a) Hei
ghtofthetr
ia
ngu
larf
ace
s. (
2ma
rks
)
b) Le
ngthofAC. (
2ma
rks
)
c) Angl
ebetweenVAandABCD. (
2ma
rks
)
d) Angl
ebetweenVADandABCD. (
2ma
rks
)
e
) Angl
ebetweenVABandVBC. (
3ma
rks
)

23.T
het
abl
ebe
lows
howma
sse
sof
200p
eop
lef
romKa
pso
yaVl
l
age
.

a
) Dr
awacu
mu l
at
iv
efr
eque
ncyc
urv
efo
rth
eab
oved
ata
. (
5ma
rks
)
b
) Us
eyo
urgra
phtoes
ti
mat
e;

F
ORMARKI
NGSCHEMESI
NBOX0724351706 Pa
ge|71
Ma
the
mat
ic
s121/
1,2
i
) Th
eme dianmass. (
1mar
k)
i
i
) Th
en umb e
rofpeoplewhosemassl
iesbet
ween70.5kgand75.5kg. (
2ma
rks
)
c
) Fi
ndthefoll
owingfr
o myourgra
ph.
i
) Th
elowe rquar
ti
le. (
1mark
)
i
i
) Th
eu pperquar
ti
le. (
1mar
k)
24.a
) Usi
ngar ul
erandcomp as
sesonl
y,const
ruc
ttr
ia
n g
leABCsucht h
a tAB=AC=4.3c
ma n
da n
gleABC=300.
(
3mar
ks)
b
) Measur
eBC. (
1ma
rk)
c
) Apoin
tPi sal
wayso nthesamesi
deo f
BCa sA.Drawthel
ocuso fPsuc
htha
tan
g l
eBACisal
wayst
wic
eangl
eBPC.
(
2mar
ks)
d
) Dro
pap e r
pendi
cul
arfromAt omeetBCatD.Me a
sureAD. (
2ma
rks
)
e
) Cal
cul
atetheare
ao ftr
ia
n g
leABC. (
2ma
rks
)

EMBUNORT
HSUB-
COUNT
Y

FORM4ENDOFT ERM2EXAM.
121/1
MAT HEMATI
CS
PAPER1
.MOCKSKCSEPREDICTIONS/..
2½HRS
SECTION1(50MARKS)
Answeral
lt
heques
ti
onsf
romthi
ssect
ion
1. Wi t
houtus
ingt
abl
esorc
alc
ula
torev
alu
ate (
3mk
s)

0.
38x0.
23x2.7
0.
114x0.0575
2. Exp
res
sth
ef o
ll
owin
gasas
ing
lef
rac
ti
on (
3mk
s)

2x-
3 x -
2 1-x
- -
3 2 4
3. User
eci
pro
cal
ands
qua
ret
abl
est
oev
alu
atet
o4s
ign
if
ic
antf
i
gur
es,
thee
xpr
ess
ion

1
3622
+4. (
3mk
s)
24.
56
4. AKen
yanban
kb u
y sa
nds
ell
sfo
rei
gnc
urr
enc
iesa
ssh
owni
nth
eta
bleb
elo
w

Buying(Kshs) Sel
li
ng(kshs)
1Euro 84.15 84.
26
1ste
rl
ingpo u
nd 118.35 121.
47
Atour
istcametoKen y
afromL o
ndonwith5000Eu r
oswh i
chh econv
ert
stoKenyashil
li
ngsatt
hebank.
Wh i
leinKeny
ah espe
nta
t
otalo
fs h
s289,000t henconve
rte
dtheb a
lan
ceint
os t
erl
i
n gpoundsatt
hesameb an
k .
Cal
cul
atetheamo u
n ti
ns t
erl
i
ngp oun
dth a
therece
ive
d. (3mks)
5. Aclos
edrectang
u l
arboxismadeofwo odofden
sit
y0.4g/cm3.Ift
hewo odi
s1.5cmt hi
ckandi
tsex
ter
nalmeasur
ementsar
e30cm
by240cmb y20cm. cal
cul
ateth
ema ssofwoodmakingthebox.(Giv
eyouranswerinKg). (4mark
s)
6. Theext
eri
orangleofaregula
rpoly
gonisequalt
oo ne
-thi
rdo f
theint
eri
oran
g l
e.Calcul
atet
henumberofsi
desoft
hepo l
ygo
n
(2mks)
2 2
2x-y+xy
7. Si
mpli
fy 2 2 (
3mk
s)
x-y
8. Det
ermi
nethei
neq
ual
i
ti
est
hats
ati
sf
yth
ere
gio
nRi
nth
efi
gur
ebe
low(
3mk
s)

9. F
indt
hee
qua
ti
ono
fap
erp
end
icu
larb
ise
cto
rof
li
nePQ,
int
hef
ormy=mx
+c.I
fth
eco
-or
din
ate
sofPa
ndQa
re(-
2,6)a
nd(4,
-2)
F
ORMARKI
NGSCHEMESI
NBOX0724351706 Pa
ge|72
Ma
the
mat
ic
s121/
1,2
r
esp
ect
iv
ely
. (
3mk
s)

10.Usi
ngaru
lerandapa
irofc
omp
ass
eso
nlyc
ons
tr
uctt
ri
ang
lePQRs
ucht
hatPQ=7c
m,QR=5c
mad<PQR=30o.Co
n nst
ruc
tth
elo
cusL
ofp
oin
tsequi
di
sta
ntf
romRPan
dRQ (
3mks)

11.Thr
eeyear
sa go,afa
therwa
st hr
eeti
mesaso
ldashi
sso
n.Inth
reeye
ars
’ti
me,
thes
umo
fth
eira
geswi
l
lbe76ye
ars
.De
ter
min
e
th
eirp
rese
nta ges
. (
3mks)
x 2y1 y
-3
12.Sol
ve8=4 a +
nd272x=9 g iv
ingyo
urans
wersa
sanex
actf
rac
ti
on. (
4mks)

13.Api
eceo
fwirei
sbentin
tot
hes
hap
eofani
sos
cel
estr
ia
ngle
.T h
eba
sea
ngl
esa
ree
ach48oa
ndp
erp
end
icu
larhe
igh
ttot
heb
asei
s
6c
m.Calcu
lat
ecor
rectt
o1dec
ima
lpl
acet
hel
engt
hofthewir
e. (
3mks)

14.T
hef
i
gur
ebe
lowi
sap
ris
mwh
osec
ros
s-s
ect
io
nisa
neq
uil
at
era
ltr
ia
ngl
eAB=3c
m,BE=5c
m

i
) Drawtheneto
fthepr
is
m (
2mk
s)
i
i) Ca
lcul
at
eth
esurf
acear
eaof
thep
ris
m (
2mk
s)
15.Gi
vent
hatP=5 –2 wh er 3
e = a
2 ()
nd =4 f
1
i
n ()
d|p
| (
3mk
s)
14702
16.Expr
essthenumber1470and7056eachasapr
oductof
pri
mef
act
ors.Hen
cee
val
uat
e l
eav
ingy
oura
n s
werinp
rimef
act
or
7056
fo
rm. (3mks)
SECTI
ONI I(50MARKS)
Answeranyfi
vequest
io
nsfro
mthisse
cti
on
17.Pati
entswhoatt
endedcli
ni
cinoneweekwer
egrou
pe db
yagea
sshownint
het
abl
ebel
ow
Ag ein(y
ear
s) No.o
fpat
ient
s
0-5 14
5-15 41
15-25 59
25-45 70
45-75 15

a) St
atethemo dalfr
equency (
1mk)
b) Est
ima t
etheme anage (
4mks
)
c) Onth egri
dp r
o v
ide
d ,d
rawah ist
ogramtoreprese
n tthedi
str
ibu
ti
on. (
5mks
)
18.Abu slef
tKis
u muat9.30a.mt oward
sNa ir
obiatana ver
agespee
dof81Km/ h
r.Amata
tule
ftNai
rob
iat10.
10a.
matanave
rag
e
sp
eedo f72km/h r.Thed i
sta
nceb et
we e
nKisumua ndNa ir
obii
s360km.
a) Determine
i
) T hetimetakenbefor
ethetwov ehi
clesmet. (
3mks)
i
i)T hed i
st
a n
ceb et
weenthetwov ehi
cle
s40mi nu
tesa f
te
rme et
ing
. (
2mks)
b) AcarleftKi
sumut oward
sNa ir
obiat9.50a.ma tanaveragesp
eedof90km/hr.
Determine
i
) T hetimewh enthecarcaughtupwiththebu s
. (
3mks)
i
i)T hed i
st
a n
ceo fNai
robif
romt heplac
ewh erethecarcaughtupwit
htheb u
s. (
2mks
)
19.Thed i
agrambe l
ows h
owsah e
mi s
phereontopo fafrust
rum.Thetopra
diusoft
hef
rus
tr
umi s3c
ma ndt
hebot
tomra
diusi
s8cm.
Theh ei
ghtofthefr
ustr
umi s4cm.

3CM

4CM

F
ORMARKI
NGSCHEMESI
NBOX0724351706 Pa
8CM g
e|73
Ma
the
mat
ic
s121/
1,2

Cal
cula
te
a)T hehei
g htofthec onefr
omwh ichthefrus
tr
u mwastaken (3mks
)
b)T hevol
u meo fthefrust
rum (
4mks)
c)T hetot
alvolumeo fthesoli
d (
3mks)
20.Thre
ep ort
sA, Ba ndCa resi
tua
tedins uc
hawa ythatpor
tAi s140kmo nacompassbeari
ngN65oEofB.Por
tCis200kmona
compassb e
aringS32oEo fA.As hi
pSi sdockedi
ns ea,
86k mo nabeari
ngof1900fro
mp ortB.
a) Usi
ngas caleof1c mtorepres
ent20km, dra
wad iag
ramt oshowtheposit
io
no fpo
rtsA,BCa nds
hipS (4mks
)
b) Usi
ngy oud i
agramf i
nd
i
) T hedis
tanceb et
we entheshi
pan dportA (1mk)
i
i)T hebeari
ngo fthes h
ipfromportC (
1mk)
i
ii
)T hedis
tancefromBt oC (
1mk)
i
v)F i
ndh owf a
rCi sSo uthofA (
2mks)
v) Compassb ear
ingo fSfromA (
1mk)
21.Aconstr
u ct
ionco mpanyistouse144t onnesofs
tonesi
ns i
teAa ndsi
teBKimaniandJamesaret
os u
ppl
yth
ec omp
anywit
hston
es
fr
omt h
e i
rqu ar
ry.T hecompanyp a
yss h24000totrans
port48tonnesofst
onesf
ore v
ery28km.
a) Ki
ma nit
ransported96t o
n n
estosit
eA, 49kma way
.
i
) F i
ndh owmu chhewa spaid (
4mks)

i
i)I fKima nispen dsk shs3000t ot r
an sp
o rteve r
y8t onnesofst
onestos i
teA.Ca l
cula
tehi
stota
l pr
ofi
t. (3mks)
b.J ame stranspo rt
st her ema i
ning48t onnest os i
teB,84kma way.Ifh ema de44%p rof
it
,fi
ndthemo neyh espentontr
anspor
tin
gt he
st
o nes (
3mk s)
22.Ac ert
a i
nn u mb ero fpeop l
ea greedtoc o ntr
ibuteequall
ytobuyb ookswo r
ths h.
12000foraschoo l
li
b r
ary.Fiv
ep eop
lepull
edou tso
thatoth e
rsa greedt oc on t
ributeane xt
ras h s100e ach.Thei
rco nt
ri
bu t
ionena bl
edthe
mt obuyb ooksworthshs2000mo rethatthey
ori
g i
na l
lyex pected.
a)I ftheo ri
ginal numb erofp eoplewa sx.Wr itedown
i
) Ane xpressi
o no fh owmu che achwa stoc o ntr
ibut
eo r
igi
nall
y (1mk)
i
i)T wod i
stinctex pr
e ssi
o nso fhowmu che ac hc ont
ri
bu t
edaft
erthefi
vep ul
ledo ut (
2mk s
)
b) Calculatetheo riginal numb ero fp
e ople (4mks)
c)T her ati
oo fth esu ppo s
edo r
iginalcontri
b uti
o ntothenewc ont
ribut
ion (3mks)
23.Theq ua dr
il
a t
e ralPQRSa n dP’Q’R’S’ha vec o-or
dinat
esP(-
2,5)Q(-1,
6)R( -
4,7)S(-4,
5)andP’(
4,2)Q’(
2,0)R’(
8,-
2)andS’(8,
2)
a) Ont heg ri
dp rovi
d edp lotPQRSa ndP’ Q’R’S’ (2mks)
b) Givent hatP’Q’R’S’ isima geo fPQRSa ft
era ne nla
rgement.Findtheco-ordi
na t
esofth
ec e
ntreandthes c
alefa
c t
orofenla
rgemen t
(2mks)
c) Ont hes amea xesp lotP2Q2R2S2t heima g eo fP’
Q’R’S’af
te
rar ot
ati
onc entr
e( 0,-
1)thr
ough–90o.Wr it
ed owntheco-o
rdina
teso f
2 222
PQRS (
3mk s)
d) P3Q3R3S3i
24.T hed i
s pl
a
st
ce
h
me
ei
n
ma g
tSme
eo fP2Q2R2S2a
t
resc over
e db
fterat
yamo v
r
a
i
n
n
sl
gp
a
t
a
i
r
o
nv
t
icl
()
e
c
eaf
t
o
t
r-
e
rt
4.
2
i
Pl
mets
otP3Q3R3S3o
ec
o ndsisgi
nt
v
e
h
nb
es
y
amea x
e s (3mks)

3 2
S=2t +4t –8t+3
Find
a)T hed ispl
ace me nto fthep art
icleatt=5s econds (2mks)
b)T hev eloci
tyo fth ep art
icleatt=5s eco nds (3mks)
c)T hev alueoftwh e nth ep art
icl
eismo me ntari
lyatrest (3mks)
d)T hea cceler
a ti
o no fthep arti
cleatt=2s econd s (2mks)

EMBUNORT HSUB-
COUNT Y
FORM4ENDOFT ERM2EXAM.
121/2
MATHEMAT I
CS
PAPER2
.MOCKSKCSEPREDI CT
IONS/..
2½HRS
1. Usetabl
esofl
oga
rit
hmstoev
alu
ate (
4mk
s)
2
58.
3 32X(0.9823)
693.5
3 1
2. Si
mpl
if
y + (3mk
s)
7-2 7+2

F
ORMARKI
NGSCHEMESI
NBOX0724351706 Pa
ge|74
Ma
the
mat
ic
s121/
1,2
3. Ma
keyt
hes
ubj
ecto
fth
efo
rmu
lag
ive
n
2 2
r-y
q=m 2 (
3mks)
y+3
4. Fi
ndth
ep er
centa
geerr
orint
heare
aofap a
rall
el
ogr
amwh os
ebasei
s23.
2ma n
dh e
ighti
s11.5c
m. (
3mks)
5. I
nwh a
trati
oshouldg
radePoft
eacost
ingsh450perkgbemix
edwit
hgrad
eQo ft
eac o
sti
ngsh350perkgsotha
taprof
i
tof10%is
madebysel
li
ngthemixt
urea
tsh451perkg? ` (3mks
)
6. I
nthefi
gurebel
owABCi sat
ange
nttotheci
rcl
eatpoi
ntB.Giv
entha
tBE=6.9cm,FE=7.8cmGE=4.1cm,DC=11.2c
ma ndED=
xc
m.De te
rminethel
engt
hBC,gi
veyouranswert
ofours
ign
if
ic
antf
ig
ure
s. (3mks)

7. F
indYwi
th
outu
sin
gta
ble
s.I
f2+L
og23+L
og2y=L
og25+1 (
3mk
s)

8. T
hreequan
ti
ti
esX,YandZaresucht
hatXvar
ie
sd i
re
ctl
yasthes
quar
eo fYa
ndinv
ers
elya
sthesq
ua r
ero
oto
fZ .Gi
vent
hatY
i
ncr
easesb
y5%a n
dZd ecr
easesb
y36%.F i
ndtheper
cent
agecha
ngeinX. (3mks
)
9. T
hefig
urebel
owshowstr
ia
nglePQRinsc
ri
bedi
nac i
rcl
ePQ=6cm,QR=3c man
da ng
lePQR=120o

Calc
ulat
eth elen gt
hPRa ndtheradiuso ft
hecir
cle (4mks)
10.Gi
11 a
v
ent
. Ex
h
p
a
tt
a
n
h
da
ema
nds
t(
r
i
i
x
mp
5 -
x2
3x 4
l
if
yt h
)
ef
irs
ha
tf
sn
ou
oi
rte
n
r
ver
s
ms (
e.Fi
ndx
2–½x) 6
(2mks)
(
2mks)
6
b. Us ethef i
rstthr
eet er
mso fthee xpansi
onabovetofi
n dtheapproximat
eval
ueof(1.
97) (
2mks)
12.Dete
rmineth eq uart
il
ed evi
ati
onf ort
h esetofdat
ag i
venb el
ow (
3mks)
20,40,30, 42, 10,18, 26,32
13.Thepoint
sA(- 4,2)andB( -8,
-4)a r
et heendpoint
sofad iamete
ro facirc
le.F
indth
ee q
uati
onoft
hecir
cl
ea ndle
aveyourans
weri
n
2 2
th
eforma x +b x +cx+d y+e=0wh er
ea,b,
c,
da ndea reconst
ants (
4mks)
14.Sol
veforθi nt hee q
uation
2
6Cos θ-s i
nθ-4=0i nt herange 0o ≤θ≤1800 (3mks)
15.Mrs.Kazidepo s
itsshs15,000i nab ankthatpayscompou ndint
erestat12%p.aIft
heint
ere
sti
scompoundedquart
erl
y,Ca
lcul
at
ethe
amountatth een dof2y ears(Ro undo ff
yourval
uesto4d e
cimalplac
e s) (
3mks)
9
16.Theprobabi
litythatastu d
entgetsg r
ad eAinMa th
ematicsis . Ifsheg et
sgr
adeAinma t
hemat
ic
sthenth
ep ro
babi
li
tytha
tsheget
s
10
4 3
Ainp h
y s
icsis .I fshed oesno tgetgradeAinma t
hema ti
csthenthep r
obab
il
it
ythats
hegetg
radeAinphysi
csis .Ca l
cula
tet
he
5 8
proba
bil
it
yt hats hegetsgradeAi nph ysi
csonly
. (
2mks)

SECTIONII(50MARKS)
Answeranyf
iv
equest
io
nsfr
o mthi
ssec
ti
on
17.Mr .
Njagi
,ac
ivi
lser
vantea
rnsabas
icsa
lar
yofsh38,
300ho
useal
l
owa
nceofs
h12,
000andmedi
cal
all
owanc
eofshs3600e
ver
y
mont
h.Hecla
imsafamil
yrel
ie
fofsh1172an
di n
sur
anc
erel
i
efo
f3%oft
hepre
miu
m’sp
aid.Us
ingt
axtab
lebel
ow

Tax
abl
eincome(
£)p
.a T
axKs
h/£
1-8800 2
8801-16800 3
16801-24800 5
24801-36800 7
F
ORMARKI
NGSCHEMESI
NBOX0724351706 Pa
ge|75
Ma
the
mat
ic
s121/
1,2
36801-48800 9
Over48800 10

a
) Ca
lcu
lat
eMr
.Nj
agi

san
nua
lta
xab
lei
nco
mei
nKe
nyap
oun
dsp
era
nnu
m (
2mk
s)

b
)Ta
xdu
eev
erymo
nthf
romMr
.Nj
agi
to2d
eci
mal
pla
ces (
5mk
s)

c
)If
fur
the
rde
duc
ti
onsa
rema
dee
ver
ymo
nthf
romh
iss
ala
ry

- WCPSo
f2%o
fba
sics
ala
ry

- L
if
ein
sur
anc
epr
emi
umo
fsh
s.4600

- Sa
ccoloa
nrepay
men
tsh
s.14200
Cal
cul
at
e
i
) To
tald
educt
io
ns (
1mk
)

i
i) Hisn etp aype rmo nth (
2mk s)
18.Comp letethet ablebelo
wf o rthetrigo
n ometr
icfunct
iony=2Co sθa n
dy=s i
n½θ (2mks)
θ 0 3 6 90 12 15 180 21 24 27 300 33 36
0 0 0 0 0 0 0 0 0
Y=Si n½θ 0 0.7
1
Y=2Co sθ 2 -
2 1.7
3
b. Usingt hetab l
ea bovedrawt heg rapho fy=Si n½θa ndy=2c osθo nth esa
mea xe
s (5mks)
(Useth es cal
e:1c mt orepresent30oo nt hexa xi
sand2cmt orepresent1u ni
tontheyaxi
s)
c. Usey ou rcurve stofi
nds oluti
ont otheequ at
ion Sin½θ-2Co sθ=0 (
1mk )
d. Stat
et he
i
) Amp li
tud eoft hec ur
vey=2Co sθ (
1mk )
i
i) Peri
o do fthec u r
vey=Si n½θ (
1mk )
19.Ap ointP(50oN, 10oW)iso nth ee a
rth’
ss ur
face.Aplanefli
esfr
o mPd u
ee as
tonap ar
all
elofl
ati
tu
defor6hou rsa
t300k n
o t
sto
portQ.
a) Determin eth ep osi
ti
ono fQt othen earestdegree
. (
4mk s)
b)I ft
h etimea tQwh enthep l
a n
el andsis11.20a.mwh atti
mei si
ta tP. (
2mk s)
c)T hep l
a nelea vesQa tthesa mes peeda n
df l
iesduenorthfor7h oursal
o ngalongi
tu
detoairp
o r
tR.De t
ermi
neth epos
it
ionofR.
(
4mk s)
20).Thes umo f21sta nd65thtermso f a
na ri
thme t
icseq
uen c
eis368.Gi vent ha
tthesev
enthte
rmo ft
hes e
quenc
ei s40.Fi
nd
a)T hec ommo nd iff
erence (3mks)
b)T hefirstterm (
2mk s)
c)T hes umo fthef i
rst16termso f t
h eA.P. (2mks)
d) Givenf urtherth a
tthe1st,5thand13tht e
rmso ftheari
thmeti
cs e
qu enc
ef ormthefi
rs
tthr
eecon s
ecut
ivet
ermsofag eo
me t
ri
c
progression,

Fi
nd
i
) T h
ecommonrat
io. (
1mk
)
i
i)T h
esumofth
efir
st20t
ermso
fth
eG.
P. (
2mk
s)
21.I
nthef
ig
urebel
owOA= OB= a ndOC=3OA.Po
intMd
ivi
desOBi
nth
era
ti
o3:
2

F
ORMARKI
NGSCHEMESI
NBOX0724351706 Pa
ge|76
Ma
the
mat
ic
s121/
1,2
a
) Ex
pre
ssi
nte
rmso
f a
nd
i
) AB (
1mk
)
i
i
) MC (
1mk
)

i
i
i) MA (
1mk
)

b
. Gi
vent
hatMN=k MCa
ndAN=h
ABwh
ereka
ndha
res
cal
ar
s,e
xpr
essMNi
ntwowa
ysh
enc
efi
nd
i
) Th
eval
ueofkandh (
5mk
s)
i
i
)T h
erat
ioofAN:
NB (
1mk
)
i
i
i) Ex
pre
ssMNi
nte
rmso
f a
nd o
nly (
1mk)
3 2
22a
.Co
mp let
eth
efo
ll
o wi
ngt
abl
eforth
eeq
uat
iony=2x+3x–6x–4
x -4 -3 -2 -
1 0 1 2
3
2x -128 -16 0 2 16
2
3x 48 27 12 3 0 12
-6x 24 0 -12
-4 -4 -4 -4 -
4 -
4 -
4 -4
y -60 4 -
4 12

3 2
b. Ont h
eg ri
dpr
ovi
deddra
wthegrap
ho f
y=2x +3x -6x-
4 (
3mks
)
c) Byd r
awingasui
ta
bles
tr
aig
htli
neuseyourg
rapht
os ol
vet
heeq
uat
io
ns
3 2
i
) 2x +3x -
4x-
2=0 (
3mks
)
3 2
i
i) 2x+3x–6x=4=0 (
2mks
)
23.Thefi
gurebel
owshowsarec
tan
gula
rtankofl
engt
h8m, wi
dt
h6mandhei
ght4m.
SBi
sad
iag
ona
loft
het
anka
ndTi
sthemi
dpo
into
f
SR.

Cal
cul
ate
a)T hel
engt
hSB (
3mk
s)
b)T heang
lebet
wee
nSBa n
dth
ep l
aneCDSR (
2mk
s)
c)T hel
engt
hATto1d.p (
3mk
s)
d) Angl
eATD (
2mk
s)
2
24 a)Compl
etet
het
abl
ebel
owfo
rthefun
cti
ony=x+3 (
2mk
s)

x 1 1.
5 2 2.
5 3 3.
5 4 4.
5 5 5.
5 6
y 4 7 15.
25 19 28 39

b
) Us
ethemid-
ord
ina
terul
ewithfi
vest
ri
pstoe
sti
matet
hear
eabo
undedbyt
hec
urv
e,t
hel
i
nex=1a
ndt
hel
i
nex=6
(2mk
s)
c
) Us
eint
egra
ti
ontof
indtheexac
tare
ain(b)ab
ove (3mk
s)
d
) Ca
lcu
lat
ethepe
rce
ntag
ee r
rorar
isi
ngfr
omtheuseof
themi
d-
ordi
nat
erul
e (
3mk
s)

L
UGARISUBCOUNT
Y

121/1
MATHEMATICSALT
.A
Pape
r1
June
/.MOCKSKCSEPREDI
CTI
ONS,
.
Ti
me:2½Ho ur
s

F
ORMARKI
NGSCHEMESI
NBOX0724351706 Pa
ge|77
Ma
the
mat
ic
s121/
1,2
1. Wi
th
outu
sin
gca
lcu
lat
orse
val
uat
e: (
3mk
s)

2. Th
eint
eri
oran
g l
eofar
egul
arpol
ygonis4ti
mesth
eext
eri
orang
le.
Howmanysid
esd
oest
hepol
ygonha
ve?(3mk
s)
3. Mr.Ma
kasembohasat
ri
angul
arpl
otth
a tme
asu
re170m,190mand210m.
Fin
dthea
reao
fth
ispl
otinh
ect
are
s
(
3mk
s)
4. Us
eta
ble
sofr
eci
pro
cal
son
lyt
ofi
ndt
hev
alu
eof (
4mks)

5. Anel
ect
ri
cpol
eissu
pport
edt
ost
andv
ert
ic
all
yonal
eve
lgr
oun
dbyat
ig
htwi
re
.Th
ewi
rei
s p
egg
eda
tad
ist
anc
eof6me
tre
s
f
romthef
ooto
fthepol
eassh
own.

Th
ea n
glewh i
chthewi
rema
keswi
tht
heg
rou
ndi
sth
reet
ime
sth
ean
glei
tma
keswi
tht
hep
ole
.Ca
lcu
lat
eth
elengt
hofth
ewi
ret
oth
e
ne
ares
tc e
nti
me t
er. (
3mk s)
6. Dra
wt henetoft
hesol
i
dbel
ow. (2mk)

7. At
ri
ang
leABCwh
osea
reai m2i
s4c sma
ppe
don
tot
ri
ang
leA1B1C1wh
osea
reai m2u
s64c nde
rat
ran
sfo
rma
ti
onma
tri
x

a
) Cal
cul
ateth
ep o
ssib
lev
alu
esofn (
2ma
rks
)
b
)F i
ndtheimageofA(3,
4)und
ert
hea
bov
ema
tri
xtr
ans
for
mat
io
nwh
eren
<0 (
2ma
rks
)
2
8. Ex
pre
ss i
nth
efo
rmo
fax+b
x+c
=0.
Whe
rea
,ba
ndca
rec
ons
tan
tsh
enc
eso
lvef
orx (
4mk
s)

9. Twos i
milarsoli
dsh avema sseso f80k ga n
d270k grespect
ivel
y.Fi
n dth
es u r
facea r
eao fthela
rgersol
idifthesma l
lers
o l
idhasa
surf
aceareao f48cm2 (
2mk s)
10.AKe nyanba nkbuysa nds el
lsfore
ig nc urrenci
e susin
gt heratesshownb el
o w;
Buying sell
in g
(ks
h) (k
sh )
Euro86.25 86.97
100J apanesey e
n66. 51 67. 26
AJ apane
set r
avell
ingfromF ra
n c
ea rrivesinKe nyawi t
h5000Eu ros,whi
c hh ec onve
rtstoKe nya
ns hi
l
li
n gsattheban k.
wh il
einKenya
hesp e
ntat ot
a lo
f ksh289, 850a ndt henc o nv
e rt
edt her e
ma in
ingKenyans hil
lingstoJap a
n e
seYenattheb ank.Calcul
atetheamount
ofJapanese Ye nt hather ecei
ved (3mks)
11.Threeli
ter
so fwater(den si
ty1g /cm3)i sa ddedt otwelveli
tersofal
cohol(den s
ity0.8g /cm3) wh atisthedensit
yo ft
hemixtur
e
(
3mk s)
12.Thea ngl
eo fele
vati
o noft h
et opof ac li
fffr
o mp o i
ntPis45°. F
romap ointQwh ichisl0mf romPt o
wa r
d sthefootofthecli
ff
,the
angleofel
e va
tionis48°Ca lcula
teth eh eightof t
hec l
if
f. (
3mk s)
2 2
13.Ac i
rcl
ec e
n tr
e0h a sthee quat
ionx +y =4. T heareao ft
hec i
rcl
einthefirstquad r
antisdivi
dedint
o5v erti
calst
ripsea
c hofwidt
h
0.4cm.
(a
)Us ethee q
u at
iono fthec i
rcletoco mp let
eth etablebelowf orval
uesofyc orrectto2d.p. (
1mk )
x 0 0.4 0.8 1.2 1.6 2.
0
y 2.00 1.60 0
(b
)Us ethetrapezoidalrul
et oesti
ma t
et h eareao fthec i
rcl
e. (3ma rks
)
14.At r
adersel
lsab ago fbeansf orksh2, 100a ndth a
tof ma i
zeatksh.
1,200.Hemi xedma izeandbeansintheratioof3:2.Findhow
mu chthetradershoulds el
lab ago fthemi x t
uretor e
alizethesamep rof
it (
3mk s)
F
ORMARKI
NGSCHEMESI
NBOX0724351706 Pa
ge|78
Ma
the
mat
ic
s121/
1,2
15.Sol
veforx (
3mk
s)
x 3x
-1
27+3 =108
16.Findt
hea r
eao
fth
eci
rc
lei
nth
edi
agr
amb
elo
w (
3mk
s)

SECTI
ON11(
50MARKS)
Answe
ran
yfi
veq
ues
ti
onsi
nth
iss
ect
io
n

17.Agroupofpeopl
epla
nnedtocont
ri
buteequal
lytowar
dsaloca
lAboli
ti
onproj
ectwhic
hn eed
edKsh200000t ocompl
et
e.Howe
ver
,
40memb er
softhegro
uplef
tth
ep r
o j
ect
.Asar e
sult
,eac
ho f
theremai
nin
gme mberswe r
etoco
n t
rib
uteKsh2500.
a)F i
ndtheori
gin
alnumberofmembersinth
eg r
o up
. (5mks
)
b)F or
tyf
iveper
centoft
heval
ueoftheproj
ectwasfundedbyasani
ta
ryNGO, Ca
lcul
at
et heamountofcont
ri
buti
ontha
twoul
dbemade
byeac
ho ft
heremain
ingmembersofthegro
up . (
3mks)

c) Member
’sc o
ntri
bu t
io
nswe r
einter
msof l
abourpr
ovi
dedandmo n
eycont
ri
but
ed.
Ift
hera
ti
oofth
ev al
ueofl
abo
urt
oth
emo
ney
co
ntr
ibu
tedwa s6:9;cal
cul
at
ethetot
alamountof mo n
eycontr
ibut
edbyt
hemember
s. (2mks
)
18.I
nthefi
gurebelo
wPQRi st
angentt
othecir
cl
ea tQandthean
glePQS=28°an
dangl
eUTQ=54°andUT=TQ

Sta
ti
n greas
ons,dete
rminetheval
ueso fang
lesnamedbel
o w
a) <ST Q 2mk s
b) <T QU 2mk s
c) <T QS 2mk s
d) Re fl
exangl
eUOQ 2mk s
e) <T QR 2mk s
19.Onth egri
dprovi
d e
db el
ow:
(a
)Dr awtri
angleABCwh os
ecoordi
natesar
eA( 8,
6),B(
6,1O)andC(1O,12)andi
tsimage A’B’
C’af
terunder
goin
ga
ref
le
cti
onintheliney=x.Wri
tetheco-or
dinat
esofA’B’C’ (
4Ma rk
s)
(b
)T ri
angl
eA’B‘C’un de
rgoesanenl
a r
gementcent
re(0,0)sca
lef
act
o r½tofo
rmtria
n g
leA’
’B’

C”.Drawtr
ian
g l
e
A’’
B’
’C’
’. (3Ma r
ks)
(c
)T ri
angl
eABCi sstre
tchedwit
hy-a x
isinv
aria
ntandst
retchf
act
orof½ t oobt
ai
nt ri
angl
eA’
’’
B’
’’
C’
’’.Dr
awtri
angle
A’’
B’
’C’
’’
. (3Ma r
ks)
20.a)
.Us i
ngar ul
erandac omp a
ssonl
y, c
o ns
tr
u c
tatri
angl
eABCs ucht
h atAB=6.8cm, BC=5.6cman dangeABC=37½0
l
(3mks)
(b
)L ocat
ethe:
(i
)L ocusPsuchthatangleAPB=ang l
eACB (3mks)
(i
i
)L ocusQsucht ha
tQi sequi
dis
tanttopoi
ntsAandB (2mks)
(i
i
i)L ocusRsuchthatRi sequi
di
st
anttoline
sABa ndAC (2mks)
21.a)T hre
ep oi
nt
sA( 0,4)B(2,
3)andC( -
2,-
1)arever
ti
cesofatri
ang
le.
F i
n d
;
i
) T hegradi
entofAC 1mk

i
i
) Th
eg r
adi
entoft
heperp
endic
ularbi
sect
orofl
ineAC 1mk
i
i
i) Th
ecoordi
nat
esofth
emi d
-poi
ntofli
neAC 1mk
b
).i
) Th
eg r
adi
entofAB 1mk
i
i
) Th
eg r
adi
entoft
heperp
endic
ularbi
sect
orofl
inesAB 1mk
i
i
i) Th
ecoordi
nat
esofth
emi d
-poi
nt:ofAB 1mk
c
)i) f
indt
heequat
io
no ft
heperpe
ndicul
arbis
ect
orof AC 1mk
i
i
) Th
eequati
onofper
pendi
cul
arbise
ctorofAB 1mk
i
i
i) Henc
efin
dthecoor
dinat
esofth
ec i
rcumcen
treof t
het
ri
ang
le 2mk
s.

F
ORMARKI
NGSCHEMESI
NBOX0724351706 Pa
ge|79
Ma
the
mat
ic
s121/
1,2
2
22.Thedi
spl
ace
mento fapar t
ic
leSme t
res
,tsecondsaf
terpas
si
n gafi
xedpo
int0i
sg i
venbyS=3+2t-
5t
Cal
cul
at
e:
(a
)T hedi
spl
aceme ntofthepart
ic
le2secondslat
er (
2mk
s)
(b
)T heti
metakenforth epar
ti
cl
etoretu
rntoO (2mk
s)
(c
)T hemaxi
mu md i
s p
lacementofth
ep a
rti
cl
e (
3mk
s)
(d
)T hei
nit
ial
veloci
tyo fth
ep a
rti
cl
e (
2mk
s)
(e
)T heacc
ele
rat
iono fthepart
icl
eaft
ertsec
o n
d s (l
mk)
23.Thedi
agr
amb el
owr epr
e sen
tsaconic
alves
selwh i
chsta
ndsvert
ic
all
y.
Theves
selconta
inswa te
rtoad e
pthof30c m.th
er a
diusoft
hewate
rsurf
acei
nt h
evess
elis21c
m(t
ake )

a) Calc
u l
atethevolumeo fthewa t
erintheves
selincm3 (
2mk s)
b) Wh ename talsphereiscompletel
ysubmergedinthewate
r,t
h el
evelofthewateri
nthe ve
sselr
ise
sb y6cm.cal
cul
ate
i
) ther adi
uso fthen e
wwa tersurf
aceinthevesse
l (2mks)
i
i)thev olu
meo ftheme t
alsphereincm3 (3mks)
i
ii
)ther adi
uso fthesphere (2mks)
24.Thet
abl
eb el
o ws howsp at
ien
tswh oatte
ndac l
ini
cino neweekandwe r
eg r
o upedbyageas sh
o wninthetab
lebel
ow.
Agexy ears 0 x <5 5 x <15 15 x <25 25 x <45 45 x <75
Numb erofp at
ient
s 14 41 59 70 15
(
a)Es t
ima t
etheme ana ge (4mks)
(
b)Ont h egri
dp rovi
d e
dd ra
wah i
stogra
mt orepr
esentth
ed i
st
ri
b ut
io
n . (3mks)
Usethes c
ales:1cmt orepres
ent5u ni
tsontheh or
izo
ntala
xis2cmt or e
prese
nt5u ni
tsont
hev
erti
cal
axi
s.
(
c)( i
) St
a t
et hegroupinwh i
chtheme di
anma rkli
es (
1mk )
(i
i
) Av er
tic
a l
li
ned rawnthroughtheme d
ianma r
kdivi
desthetot
a la
reaofth
eh ist
ogr
ami
ntotwoequal.
Usin
gt h
is
i
nforma t
ionesti
ma t
etheme di
anma rk.(
2mk s
)

F
ORMARKI
NGSCHEMESI
NBOX0724351706 Pa
ge|80
Ma
the
mat
ic
s121/
1,2
LUGARISUBCOUNTY
121/2
MATHEMATICSALT
.A
Pape
r2
June
/.MOCKSKCSEPREDI
CTI
ONS,
.
Ti
me:2½Ho ur
s

SECT
IONI:(
50MARKS)
1. Usel
oga
ri
thmta
ble
stoe
val
uat
e: (
4ma
rks
)

2. Ev
alu
ateb
yra
ti
ona
li
zi
ngt
hed
eno
min
ato
ran
dle
avi
ngy
oura
nswe
rins
urdf
orm. (
3ma
rks
)

3. Ma
kent
hes
ubj
ecto
fth
efo
rmu
la. (
3ma
rks
)

4. T
hefir
st
,th
irda
ndthesev
ent
hte
rmsofani
ncr
eas
inga
ri
th
met
icp
rog
res
si
ona
ret
hre
eco
nse
cut
iv
ete
rmso
faG.P.
Ift
hefi
rs
tte
rmo f
t
heAPis10.Fi
ndthec
ommondif
fe
ren
ceoft
heAP (3ma
rks
)
5. F
act
oris
ecompl
ete
ly (
2ma r
ks)
2
45-5x

2
6. a
)Fi
ndt
hee
xpa
nsi
oni
nas
cen
din
gpo
wer
sof
Xof u
ptot
het
ermi
nx . (
2ma
rks
)
7
b
) Henceev
alu
ate(
0.99) t
ofoursi
gni
fi
cantf
igu
res
. (
2ma r
ks)
7. T
hef
igu
rebel
owshowstwoconc
en t
ri
cci
rcl
eswit
hd i
ame
ter
sAB=10c
man
dPQ=5c
m.F
indt
hel
engt
hofCDwh
ichi
sata
ngentt
o
t
hei
nne
rcir
cl
eandal
soac ho
rdofthel
ar
gercir
cl
e. (3mar
ks)

2
8. Ab odyi smo vingi nas tra
ightlinesuc hthatit
sveloci
tyVm/saft
ertsec
ondsisg i
venbyv=5t -½t+3.Fin
dthedist
ancetr
ave
lle
d
duringt hethi
rds e cond. (4marks
)
9. Calculatethes t
and ardd evi
ati
o no ft
h edata:
34, 61, 49,57, 53, 37, 59 (3marks
)
10.Ifx=44a ndy=20. 1c a l
cul
a t
et hegreate
s tpossi
bleper
cent
ageerr
orinx-y .
Gi v
ey ou
ranswe rt
o1deci
ma l
plac
e.
(3marks
)
2 2
11.F in dtherad i
u sandc entreo facircl
ewh o
see qua
tio
nisx +y +3x+2=0. (3marks
)
2
12.Fi
llinth etabl
eb el
o wa ndu semi d-o
rd i
nateruletofi
ndtheare
ab oun
dedb ythecir
cley=x -1
fr
omx- 7t ox=- 1u si
n g6s tr
ips. (3marks
)
X - 7 -6.
5 -6 -5.
5 -5 -
4.5 -4 -3.5 -3 -2.5 -
2 -1.
5 -1
y 48 35 24 15 8 3 0
13.Solveth eequa t
ion2c os(3t+600)- 0.5f
or00 t< 1800 (3marks
)
14.Fi
n dwi thoutusingl ogta bl
eso rcalculat
orsthevalueofxwhic
hs at
is
fi
estheequati
on
2
l
og 3(x -9)=2l og 23+1 (3marks
)
15.Aq uantit
yVi sp artl
yac onst
an tandp ar
tl
yv ari
esasu .I
fu=1wh env=12a ndu =3wh env=22.
Fi
n dthev alueofvwh e nu=5. (3marks
)
16.Ama nd eposi
tss h.50,000i nani nvestmenta cco
u n
twh i
chpays12%inter
estp.acompoundeds e
mi-an
nuall
yFindt
hea mounti
nth
e
accoun taft
er3y ears. (3marks)

F
ORMARKI
NGSCHEMESI
NBOX0724351706 Pa
ge|81
Ma
the
mat
ic
s121/
1,2

SECTIONB( 50ma rks)


17.Ab u
st rav
ell
ingfromNa kurutoMa nder
aa ver
a g
esataspeedof70km/hrando ntheret
urnjourn
ey,thebusaver
ages20k m/hrs
lowe
r
andtakes4h ourslongerthanonthejourneyfrom Na ku
rutoMa nde
ra.
a)F indthedis
ta n
ceb et
weenNa kuruandMa nd
era. (4marks
)
b) Di esel
c ons
u mpti
onis0.32l i
tr
esperkilo
me tr
eonthejour
neyfro
mNa kur
ut oMandera.However
, t
hi
srateinc
reas
e sby
25%o nt h
er etu
rnjourney.Cal
culat
ethea mountofdi
ese
lthebusco n
sume sfo
rthetr
ip. (3marks
)
c)I fdi
eselcostssh.
65p erli
tr
ea ndthebusma kes3roundtr
ipsi
nawe e
k,deter
minethetota
lcostofdi
eselr
equir
edtor un
thebusfor5mo nt
hs. (3marks
)
18.Kimutaiear
nsK£12, 000p.aandisho u
sedbyt hecompanyatanominalr
a t
eofKs h.
2000per mo nt
h.15%o fhi
sb asi
csal
aryi
s
addedtoh i
sincomef orthepurpose
so ft
axati
o n
.Heg et
safamil
yrel
iefofK£1320p.aandisenti
tl
edtoareli
efof10%o fhisi
nsur
ance
ofK£800p .
a.
IncomeK£p .a RateperK£
1-2100 10%
2101-4200 15%
4201-6300 25%
6301-8400 35%
Over8400 45%
a) Ca lcul
atethetaxabl
eincome . (2marks
)
b) Ca lcul
atehisPAYE (5marks
)
c) Ki mutai
’sotherdeducti
onsincl
udesW. C.P.
Ss h.
600p.
m,NHI Fsh.
500p .
m. Cal
cul
atehis netmonthl
ys a
lar
y.
(3marks
)

19.I
nth
etr
ape
ziu
mbe
low, =3 .
Tdi
vi
desSRi
nth
era
ti
o4:1a
ndUi
sth
emi
dpo
into
fQT
.PUa
ndQRi
nt
ers
ecta
tX. =h

a
nd =k .

Gi
vent
hatPQ= a
ndPS=

a
) Ex
pre
ss i
nte
rmso
f a
nd (
1ma
rk)

b
) Ex
pre
ss i
nte
rmso
f , a
ndh
. (
2ma
rks
)

c
) Expr
ess i
ntermsof , a ndk. (
3marks
)
d
) Hen
ceobta
inth
ev a
lue
sofhandk. (
3marks
)
e
) Det
ermi
nether
ati
oinwhi
chXd i
vi
desQR. (
1mark)
20.T
hep
rob
abil
i
tyofacan
dida
tep
ass
ingherse
cond
arye
xami
nat
io
nis .
Ifs
hep
ass
esh
ere
xami
nat
io
nth
epr
oba
bil
i
tyo
fherj
oi
ni
ng

u
niv
ers
it
yis I
fsh
efa
il
she
rex
ami
nat
io
nth
epr
oba
bil
i
tyo
fhe
rjo
ini
ngt
heu
niv
ers
it
yis .
Ifs
hej
oi
nst
heu
niv
ers
it
yth
e

p
rob
abi
l
it
yofh
erg
ett
in
gaj
obi
s a
ndi
fsh
edo
esn
’tj
oi
nth
eun
ive
rsi
tyt
hep
rob
abi
l
it
yofh
erg
ett
in
gaj
obi
s .
Usi
ngat
ree

di
agr
am, f ind:
a)thep robabi
li
tythatshefail
sh erexami
na t
io
n . (
3marks
)
b)findthep r
ob abi
li
tythatsheg otajobaf
terfai
li
nghersecond
aryexaminat
io
n. (
2marks
)
c)thep robabi
li
tythatshejoinstheu ni
ver
sit
y. (
2marks
)
d)thep robabi
li
tythatshed i
dn otgetajob. (
3marks
)
3
21.Apar
ti
clemo vi
ngi nastra
ightli
n eissuchthati
tsdis
pla
c e
mentsme t
resfromagiv
enpoi
nti
ss=t -6t+2t+3wh e
reti
sti
mein
s
econds
, Fi
nd:
a)T hed i
spl
aceme ntoft
h eparti
cleatt3. (
2marks
)
b)T heveloci
tyofthep a
rtic
lewh eret=4. (
2marks
)
c)T hevalueoftwh eretheparti
cleismome nt
a r
il
yatres
t. (
3marks
)
d)T hea c
cele
rati
o noftheparti
clewh ent=4 (
3marks
)
22.Th
ema nagerofac i
n e
mawi shestod iv
idetheseat
savai
labl
eint
otwoclassesAandB.Hehast
hef
o l
l
owingcons
tr
ai
nts
.
i
) T herearenotmo rethan120s ea
tsavai
labl
e
i
i)T heremu s
tb eatleas
ttwicea sma nyBc l
asssea
tsasther
eareAclasssea
ts
i
ii
) Cla
ssAs eatsarepric
eda tsh.30e a
chan dcla
ssBa tsh.
20eachanda tl
eas
tsh.
2000sh
o u
ldb
ec ol
l
ecte
dateac
hshowto
F
ORMARKI
NGSCHEMESI
NBOX0724351706 Pa
ge|82
Ma
the
mat
ic
s121/
1,2
meettheexpe
nses.
a
) Wri
tein
e q
uali
ti
esfr
omthecon
str
ain
tsli
st
edab
ove. (
3ma r
ks)
b
) Ont
h egri
dp r
ovid
edplo
tth
eineq
uali
ti
es. (
4ma r
ks)
c
) Fi
ndthen umberofs
eat
sofea
chkindwhic
hwil
lgi
vet
hema
ximu
mpr
ofi
tan
dca
lcu
lat
e t
hi
smaxi
mumprof
i
t.
(
3ma r
ks)
23.Th
eposi
ti
onsofa
ir
portPandQar
e(60°N,45°W)an
d(60°N,
K°E)r
esp
ect
iv
ely
.Itt
ake
sap
lan
e5h
ourst
otr
ave
ldu
eEastf
romPto
Qata
naverag
espee
do f600k
not
s.Ta
kingR=6370kmand

a
) Cal
cul
atet
hevalu
eo fk. (
3ma
rks
)
b
)T hel
ocalt
imeatPis10:45a.
m. Whati
sthelo
calt
imeatQwhe
nthepla
nereac
hesth
ere
. (
4ma
rks
)
c
)F i
ndthedi
sta
ncePQme as
uredal
ongacir
cleofl
at
it
udetot
hene
are
stkm.(3mark
s)
24.T
hef
ig
urebel
owisasquarebasedpyr
amidABCDVwithAD=DC=6cma ndhei
ghtV=10cm.
a
) Stat
ethepro
jec
ti
ono fVAo nth
eb a
seABCD. (
1ma
rk)

b
)Find:
i
) t
hele
ngthofVA (
3ma
rks
)
i
i
)theangl
ebetwe
enVAandABCD (
2ma
rks
)
i
i
i)t
heangl
ebetwe
enthepl
ane
sVDCa
ndABCD (
2ma
rks
)
i
v)v
olu
meo ft
hepyra
mid (
2ma
rks
)

T
ETU

121/1   
  
 
  
   
 
Mathemati
csPa pe
r1
2½Ho ur
s
Ter
m2, .
1 13
o f18- 3+2 x
2 2- 5
1. Eval
u a
te: (3ma r
ks)
1 3 3
+3 ÷
2 4 4
2. Theh e
ight
softwos i
mi l
arpai
lsar
e12cma nd8cm.T h
elarg
erpai
lca
nh o
ld2li
tr
es.
Wh ati
stheca p
aci
tyo ft
h es
ma l
l
erpai
l? (3ma r
ks)
5
3. Giv
ent ha
tCo sA= a ndangl
eAisacut
e,fi
ndtheva
lueof (3marks
)
13
4. Atou r
is
tarr
ivedinKe nyawit
hster
li
ngpound(£)4680all
ofwhic
hh eex
cha
ngedint
oKen
yanmo n
ey.Hes
pentKs
h.52,352whil
ein
Kenyaandc onver
tedt h
er e
stofthemoneyi ntoU.Sdoll
ar
s.Cal
cul
at
etheamounther
ece
ive
dinU.Sdoll
ar
s.Theexcha
ngerat
es
wereasfol
lows. (3mar
ks)

Curr
ency Bu y
ing Sell
i
n g
.
US$ 65.
20 69.
10
St
erl
i
ngPo u
nd(£) 123.40 131.80

4 2
5. Twob
oysandag i
rls
har
edso
memone
y.T
heelde
rbo
ygot o fi
t,
they
oun
gerb
oyg
ot oft
her
ema
ind
erandt
hegi
rlg
otther
est
.
9 5
Fi
ndt
heperc
en t
ages
har
eoft
hey
oun
gerboyt
othegi
rl

ssh
are
. (
4mark
s)
2
9x -1
6. Si
mpl
if
y: 2 (
3Ma r
ks)
3x+2x-1

7. Atwodi
gitnumberi
sfo
rmedfr
omt hefi
rs
tfo
urpr
imenu
mber
s.
(a
)Dra wt
hetabl
etoshowthepos
si
b l
eoutc
omes. (
1Ma
rk)
(
b)Ca l
cula
teth
ep r
obab
il
it
ythatanumberc
hose
nfr
omthet
wodi
gi
tnu
mbe
rsi
sane
venn
umb
er. (
1Ma
rk
F
ORMARKI
NGSCHEMESI
NBOX0724351706 Pa
ge|83
Ma
the
mat
ic
s121/
1,2
8. Ac ar
pen t
erconstr
uc t
e dac l
o se
dwo odenb oxwithint
ern
alme as
urement
s1.5metre
slong,0.8metr
eswideand0.4metreshi
gh.The
wo odusedi nconstructin
gt heb oxwa s1.0cmt hi
ckandh a
sad e
nsi
tyof0.6g/
cm³.Det
e r
mineth
ema ss,i
nk i
log
rams,ofthewood
usedinc onst
ruct
ingth eb ox.(Le
aveyou ranswerto1deci
ma lpl
ace)
. (3ma r
ks)
9. Solvethefoll
owingin equal
ityandsho wy oursol
uti
ononan umberli
ne. (3ma r
ks)
1
4x–3≤ ( x+8)<x+5
2
10.Solveforxg i
venthat
Log(x–4)+2=l og5+l og( 2x+10) (3marks)
11.Fromav iewingtower30me t
resaboveth eground,t
heangl
eo fdep
ress
ionofanobj
ectonthegrou
n s300a
di n
dtheangleof
el
evati
ono fana i
rcr
a f
tv er
tical
lyabovetheo bj
ec s420.
ti Cal
culat
ethehei
g h
tofth
ea i
rcr
aftabov
ethegr
o u
n d
. ( 3marks)
12.Thep osi
ti
onv ec
torso fAa ndBa regivena sa=2i –3j+4ka ndb=- 2i–j+2k
respect
ivel
y.
Fi
n dto2d ecimalpl
a ces,thelengt
ho fvectorAB. (3Ma r
ks)
13.Ar egu
larpo l
ygonha sinter
n alangl
eo f1500ands i
deofle
ngth10cm.
(a
)F i
n dthenu mberofside softhepolygon. (2Ma r
ks)
(b
)F i
n dtheperimete
ro fthep olygon. (2Ma r
ks)
14.Inthefigurebel
owPQRSi sar ect
angleinwh ic
hPS=10Kc ma n
dPQ=6Kc mMa ndNa remidpoi
ntsofQRa ndRSres
pecti
vel
y.Fi
nd
theareaoftheshadedp ar
t. (4ma r
ks)
P S

Q M R

15.Us i
ngap ai
rofco
mp a
ssesa ndarul
eronl
y,c
o n
str
uctt
rap
ezi
umABCDi nwhi
chAB=8cm,BC=5cmD=4.
1cman
da n
gleABC=60o
andABi spara
ll
elt
oDC. Dete
rmin
ethrou
ghc o
nst
ruct
io
n,theh
e i
ghtofth
etrap
ezi
um. (
4mks
)
16 Twon umb e
rsar
eintheratio5:7.When15isadde
dtoeachnumber,
therat
iocha
nge
sto5:6.
Fi
n dthetwonumbers
. (
3mks
)
17.(
a )(i
)Fil
lthet
abl
ebel
o wf ort
hefunc
ti
on.
2
y=2x +5x–12fo r-8x4 (
2ma r
ks)
x -8 -
7 -
6 -
5 -
4 -
3 -
2 -
1 0 1 2 3 4
y 76 -
9 -
5 40
2
(i
i
)Us ingt hetabl
e,drawth egrapho ft
hefunct
iony=2x +5x–12. Us
ethesc
ale1cmto1uni
tont
hex-ax
isand1cmfor
10u nitsfo
rthey–a x
is (
4ma r
ks)
(b
)Us et heg ra
phd ra
wna bo v
et osolv
ethefoll
owin
ge quati
ons
.
2
(
i)2x +5x–12=0 (
2ma r
ks)
2
(
ii
)3–7x–3x =0 (
2ma r
ks)
18.Thefol
lo winga r
ema ssesof25s tudent
sinform4c l
ass
.
49,51, 50, 60,55,45,56,51, 58,59,40,54,44
44,42, 59, 50,62,46,43, 57,56, 52,43,41,
(a
)Pr e
pareaf re que
ncyd i
stri
buti
ont ab
lewithau ni
fo
rmc l
asssiz
estar
ti
ngwit
hthecl
ass40–43. (
4ma r
ks)
(b
)Es t
imatet heme dia
nma ss (
3ma r
ks)
(c
)Dr awah is
to gr
amf ort
hed ata. (
3ma rk
s)
19.Asolut
io nwh o
sevo l
ume80l i
tr
esisma deupof40%o fwaterand60%ofal
cohol
.Wh nxl
e i
tr
esofwa
teri
sadde
dt h
ep e
rce
ntag
eo f
al
cohol dropsto40%.
a)F i
ndth ev al
u eofx (
4marks
)

b
)Th
irt
yli
tr
eso
fwa
teri
sad
dedt
oth
ene
wso
lut
io
n.Ca
lcu
lat
eth
epe
rce
nta
geo
fal
coh
oli
nth
ere
sul
ti
ngs
olu
ti
on. (
2ma
rks
)

c)I f5lit
ersoft
hesol
uti
onin(b)a
boveisa
ddedt
o2l
it
ersoft
heori
gi
nals
olut
io
n.Cal
cul
at
eint
hes
imp
les
tfo
rm,th
erati
ono
fwa
tert
o
thatofal
coho
linth
eresul
tin
gsol
uti
on. (4mark
s)
20.
Th ediag
rambelo
ws ho
wst woci
rc
les
,cen
treAa
ndBwh i
chi
nte
rse
ctatp
oin
tsPandQ.
An gePAQ=700,
l a
nglePBQ=400a n
dPA=AQ=8cm.

F
ORMARKI
NGSCHEMESI
NBOX0724351706 Pa
ge|84
Ma
the
mat
ic
s121/
1,2

0
A 70 400 B

Uset h
ed i
agramtoc al
c u
lat
e
(a
)PQt oc orr
ectto2d ecimalpla
ce s (2Ma rks
)
(b
)PBt oc orr
ectto2d ecimalpl
aces (2Ma rks
)
(c
)Ar eao fthemin o
rs egmentofthecirc
lewhosecent
reisA (
2Ma r
ks)
(d
)Ar eao fshadedregion (
4Ma r
ks)
21Ast
raightlinepassesthroughthepoint
s(8,-
2)and(4,-
4).
(a
)Wr iteitsequati
oni ntheforma x+by+c=0, wherea,bandca rein
teger
s. (
3Ma r
ks)
(b
)I ft
hel inein(a)abovec ut
sthex -
axi
satpoi
ntP,det
erminethecoordi
nat
esofP. (2Ma rks
)
(c
)An othe rli
ne,wh i
chisp erpe
ndicul
artotheli
nein(a)abovepass
esthrou
ghp o
intPandcutstheyaxi
satthepoi
n tQ.Det
ermi
neth
e
coordi
n ate
so fpointQ. (3Ma r
ks)
(d
)F i
ndt hel e
n g
tho fQP (
2Ma r
ks)
22.a
) T rainAl eav
esas tat
ion45mi nute
sbefor
etra
inB. Bot
htrai
nstrav
eli
nthesamedir
ecti
ona n
dthei
rspe
edsare36km/hand
48km/ hr esp
ecti
vely.
i
) Ho wl o ngwilli
ttaketrai
nBt oc at
chupwithtr
ai
nA? (
3marks)
i
i) Howf arfromt hestar
twe rethetwotrai
nswhentheyme t
. (2ma rks
)
b) Aca rac c
eler
atedfromr es
ttoav el
ocit
yof10m/sin10s ec
o nd
s.Itt
rave
ll
edatthi
sve
loci
tyfor20se
condsandth
e ncametoasto
pin
5sec onds.Find;
i
) T heinitia
laccele
rati
on . (
2marks)
i
i)T hed i
s t
ancetravel
led. (2ma rks
)

i
ii
)T hea
ver
agev
elo
cit
y. (
1ma
rk)
23.T
hefi
gur
ebel
owshowsamo
del
ofas
oli
dint
hes
hap
eof
afr
ust
umo
fac
onewi
thah
emi
sph
eri
cal
top
.

70c
m

28c
m 60c
m

Thediameterofthehemisph e
ri
caltopis70cma ndiseq ualt
othediamete
rofthet
opo ft
hefrus
tum.
Thefr
ustumh asab as
ed i
a mete
ro f28cma ndas l
antheightof60cm.
(a
)Ca lcu
lat
eth eare
ao ftheh emis
ph e
rica
lsurf
ace. (1mar
k)
(b
)Ca lcu
lat
eth esl
anth e
ighto ft
hec onefr
omwh ichthefrustumwa scu
t. (4mar
ks)
(c
)Ca lcu
lat
eth etot
alsurf
acea re
ao fthemo de
l (5mar
ks)
24.(a)( i
)Drawt hequ a
dril
atera
lA(- 6,-
1),B(-
6,-4),C(-
3.-7)andD(-3,-2)Onthesamegriddrawtheimage
(i
i)A'B'C'
D' ofABCDu nderane n
larg
eme ntcent
re(-1,
-1)andscal
efact
orY. (2mk
s)
(b
)A' '
B''
C''D'
' ofi
ma geA'B'C'D'underarotat
ionc e
ntre(0,
0)through90o (2mk
s)
(c
)A' '
'B'
''
C'''
D'''theimageo fA' '
B''C''
D''underarefl
ecti
onintheli
ney=x–2 (2mks)
(d
)A'
T
ET
'
'
U
'
B''
''
C''
''D''
''theimag eofA'''
B''
'C''
'D'
''underatransl
a()
ti
on2a
8
n dwri
tedowntheco-
o r
dia
ntesofth
efi
nal i
mage
.(2mks)

121/2 
  
  
  
  
  
 
Mathemati
csPape
r2
2½Ho u
rs
Ter
m2, .
1. Usema t
hemati
cal
tab
lest
oev
alu
ate
: (
4ma
rks
)

2. Ma k
edthes
ubje
ctoft
hefor
mul
a: (
3ma
rks
)
2 1+d b
a = -
b 3
3. Solv
efo
rxinth
ee q
uat
ion
: (
3ma
rks
)

F
ORMARKI
NGSCHEMESI
NBOX0724351706 Pa
ge|85
Ma
the
mat
ic
s121/
1,2
0 0
Si
n(4x–10)–cos(
x+60)=0
4. Si
mp l
if
yle
avi
ngy
ouran
swe
rth
efo
rma+b cwh
erea
,ba
ndca
rer
ati
ona
lnu
mbe
rs. (
3ma
rks
)
3 1
+
7-2 7
2 2
5. Th
ee q
u at
io
no fac
ir
cl
ec e
ntr
e(M,
N)i
s2x +2y –8x+5y+10=0
Fi
ndtheval
uesofMandN (
3ma
rks
)
6. Sol
veforxgiv
enth
atth
efoll
owi
ngma
tri
xiss
ing
u l
ar (
2ma
rks
)

7. Aq uant
if
yAi sp art
lyconst
antan dpa rt
lyvar
iesin
vers
elyasaquantif
yB.
GiventhatA=- 10wh enB=2. 5, A=10wh enB=1. 25
Findthevalueo fAwh enBis1.5 (4mark
s)
8. Findthequa r
ti
led ev
iat
ionforthes e
to fdat
ab el
ow: (2mark
s)
16,18,10,8,5,11,4a nd7
9. Pa ndQa retwop oint
ss uc
ht hatOP=i +2j +3K,0Q=4i +5j–3K.Mi sa p
p o
inttha
tdiv
ide
sPQe xt
erna
ll
yint
herat
ion3:2.Fin
d
thecoordi
n at
eso fM( 3ma r
ks)
10.Withoutusingma themati
caltab
leso rcalcul
ator
,eval
uat
e: (
3ma r
ks)
2log5-½ l og16+2l og40
11.Ac of
feedea l
ermixestwob r
an dsofco f
feexan dytoobta
in40kgo ft
hemixtureworthKsh
.2600.Ifbra
ndxisval
uedKsh.
70p e
rkg
andb r
andyi sv al
uedatKsh55p erkg ,
c a
lcul
ateth
erati
oinit
ssimp l
estf
orminwh ichbr
andsxandyaremixe
d
(
4ma r
ks)
12.Thec o
o r
dinatesoftwoa i
rport
sMa ndNa r
e(600N 350W)a nd(600N 150E)res
p e
cti
vel
y.Cal
cula
te:
a)L ongit
udinaldiff
erence (1mark
)
b)T heshortestti
mea na er
opla
n ewh osesp e
edis250k not
swil
ltaketoflyf
romMt oNa l
ongacir
cl
eo fl
at
it
ude.
(2mark
s)
13.Findint
egralvaluesofxforwh ich; (
3ma r
ks)
5 and 3x14<- 2
4 3
14.Ex pan
d( 1–2x ) uptoterminx .Us etheexpansi
ontoeval
uat
e:
6
(1.
02)t o4d eci
ma lpl
aces
. (
3ma r
ks)
15.Thefigur
eb elowisac uboi
dABCDEF GH.AB=12c m,BC=5c m, CF=6.5cm.

E F

A B

a) St
ateth
ep r
o j
ect
io
no fAFonthepla
neABCD (1ma
rk)
b) Cal
cul
atetheangl
ebetweenAFandthepl
aneABCDcorre
ctto2d
eci
malpl
ace
s (
3marks
)
16.Usi
ngar u
lerandapairofco
mp a
ssonl
yconst
ruc
tatri
angl
ePQRinwhi
chQR=6.6c
m,PR=3.8cma
ndPQ=5.6c
m.
Lo
catepoi
nt(X)ins
idetr
iang
lePQRwh i
chi
sequi
dis
tantf
romPandRsuc
hthata
ngePXR=900
l (
3ma r
ks)

SECTIONB:( 50MARKS)
ATTEMPTF IVEQUEST IONSONL Y
17.Ayouthgro updecide
dt orai
seKsh.480,000tobuyap i
eceoflan
dc ost
ingKsh.80,
000p erhect
are.Bef
oret
heact
ualpay
men
twa
s
made,fouroftheme mb er
sp u
ll
edoutan deachofthoseremain
ingha dt
op ayanaddit
io
nal Ks
h.20,000:
a)I ft
heo r
igi
n aln
u mbero fgr
oupme mberswa sx,
writ
ed own ;
i
) Ane xpres
s i
ono fhowth eremain
ingme mbersweretoc o
ntri
buteaf
terthefourpul
le
do ut (
1ma r
k )
i
i) Anexpres
s i
ono fhowth eremain
ingme mbersweretoc o
ntri
buteaf
terthefourpul
le
d (
1ma r
k )
i
ii
) Deter
minet heme mberswh oact
ual
lycon t
ri
bute
stowa r
dsthepu r
chaseofla
n d (
3ma r
k s
)
b) Cal
culat
eth erat
ioofthes upp
osedori
ginalcont
ri
buti
o ntothenewc ont
ribu
tion (2marks)
c)I ft
helandwa ssub -
di
videdequal
ly,f
indthesiz
eo f
lande a
chme mb e
rg ot
(3mk s
)
18.a)Completethetabl
eb elo
w:
X 0 150 300 450 600 750 900
4Co
sX 4.
00 2.
00 0 -
4.00
0
2Si
n(2X-
30) -
1.00 1 1.
732 1.
732 1

F
ORMARKI
NGSCHEMESI
NBOX0724351706 Pa
ge|86
Ma
the
mat
ic
s121/
1,2
0 0 0 0 0 0
X 105 120 135 150 165 180
4Co
sX -2.
00 4.
00
2Si
n( 300)
2X- -
1 -
1.732 -
1

0 0
b) Ontheg r
idprovi
dedd r
awthegra
pho fy=4co
s2xan
dy=2s i
n(2x–30) f
oro ≤x≤1800.Tak
ethes
cal
e1cmf
or150o
nthex-
axi
sand2c mfor1u ni
tonthey-a
xis. (5ma
rks
)
c)i) St atetheampli
tu
d eofy=4c os2x (
1ma r
k)
i
i)F i
ndtheperi
odo fy=2s i
n(2x–300) (1ma
rk)
d) Useyourgraphtosol
v e
:
4cos2x–2s i
n(2x–300)=0 (
1ma r
k)
19.Inthef
igurebel
owMa ndNa r
epointsonOBa
ndBAres
pec
ti
vel
ysu
chtha
tOM:MB=2:3a ndBN:NA=2:1.ONandAM
i
n t
ers
ectatX
A

N
X

O B
M
a) Giv
enthatOA=a OB=b
Expr
ess: (4marks
)
i
) ON
i
i) AM
i
ii
) ABintermsofa a
n db
b) Bytaki
ngOX=hON a ndAX=KAMwh erehandKares
cal
ar
s,f
i
n dt
woexp
res
si
onsf
orOXinter
msofaandbh e
ncedet
ermine
val
uesofhandKandthera
ti
oinwhi
chxd i
vi
desAM (6mark
s)
20.Inth
efigur
ebel
owP,Q, RandSar
epoi
ntsonthec
irc
lec
ent
reO.PRTan
dSTVares
tr
ai
ghtl
i
n e
s.Li
neUVi
satan
gentt
othecir
cl
ea t
S.AngeRST=500a
l ndang
leRTV=1500

P
Q

U V
S T

a
) Givingreas
ons
,ca
lcul
at
e:
i
) a n
g l
eORS (
2ma
rks
)
i
i
) An gl
eUSP (
1ma
rk)
i
i
i) Angl
ePQR (
2ma
rks
)
b
) GiventhatRT=7cm,ST=9c
m,c
alc
ula
tet
o3s
ign
if
ic
antf
i
gur
es:
i
) L e
n g
thofli
nePR (
2ma
rks
)
i
i
)T herad
iusoft
heci
rc
le (
3ma
rks
)

21.T
het
abl
ebe
lows
howst
hed
ist
ri
but
io
nof
mar
kss
cor
edb
yFo
rm4s
tud
ent
sinma
the
mat
ic
sex
ami
nat
io
n:

Mar
ks 30-
34 35-
39 40-
44 45-
49 50-
54 55-
59 60-
64 65-
69 70-
74 75-
79 80-
84
No
.ofst
ude
nts 1 5 12 14 21 24 25 8 5 3 2

a
) St
at
ethemodalc
lass
..
..
..
..
..
..
..
..
..
..
..
..
..
..
..
..
..
..
..
..
..
..
.. (
1ma
rk)
b
) Ca
lcul
at
ethemedi
anma r
k (
3ma
rks
)
c
) Us
ing57asth
eassumedme a
nma rk,
Ca
lcul
at
e ( i
)theact
ualmeanforth
eg r
oupsdata (
3ma
rks
)
F
ORMARKI
NGSCHEMESI
NBOX0724351706 Pa
ge|87
Ma
the
mat
ic
s121/
1,2
(
ii)thes tan dar
dd eviat
ion (
3mark
s)
22.Thep robabil
ityt hat3c an di
date:Jane,Ma rt
inandEd it
hwi l
lpa s
sa nex
aminat
ionare¾,2/3a
nd4/5r
espe
ctf
ul
ly.F
indth
epr
obab
il
it
y
that
:-
a) Allt
h eth r
eec an didateswi llpass (
2marks
)
b) Allt
h eth r
eec an didateswi llnotpa s
s (
2mark
s)
c) Onlyo neo fth emwi l
lp ass (
2mark
s)
d) Onlytwoo fth emwi l
lp ass (
2mark
s)
e) Atmo sttwoo fthe mwi ll pass (
2mark
s)
23.Atriangleh asv erti
cesA( 1,2),B( 4,4),C(6,2)
a) Drawt riangl
eABCo nt heCa rtesi
anp lan
e (
1mark)
b) A1B1C1 isthei ma g
eo ftriangl
eABCu nde
rar otat
iono f900c lockwis
eaboutori
gi
n.
Stat
et hec oord i
n at
eso fA1B1C1 (
2marks
)
11 11 11
c) ABC i sth ei mag eo fA1B1C1u nderre
flect
ioninthel i
n ey=x ,
Determin ethec o ordinateso fA11B11C11 (
3mark
s)
111 111 111
d) Drawt riangl
eA B C t h
ei ma g
eo fA11B11C11 underar efl
ecti
oninthel
iney=0 (
2mks
)
e) Describeas ing l
et r
a nsforma t
iont hatmapst r
iangleA111B111C111 o
ntothetr
ia
n gl
eABC (
2marks
)
Ana ri
thme ti
cp rogressive( AP)h a
sth efi
rstter
m( a)andt hec ommo ndif
fe
rence(d)
a) Writed ownt h eth i
rd,n inthandt wen t
yfif
thtermso ftheAPi ntermsofaandd (
1mk)
b)T heAPa bovei si ncreasinga ndt h
et hir
d,ninthandt wentyf i
ft
ht er
msformthefi
rstth
reec
ons
ecu
ti
vet
ermsofaGeomet
ri
c
progression(G. P).T hes i
xthtermso ftheAPi s78.
Calculat
e :
i
) T hefirstterma n dc ommo ndif
ferenceo ft
h eA.P (
5mark
s)
i
i) Sumo fthefirstn in
et er mso ftheAP (
2mark
s)
Thedi
ff
erenc ebetwe ent hef ou rt
ha nds event
hte r
mso f a
ni ncreasi
n gA.P (
2mks)

KI
GUMO

121/1
MAT HEMATICSALTA
Paper1
.MOCKSKCSEPREDI CT
IONS/
..
2½h o
urs
SECTIONI(50ma r
ks)
Answeral
lt
heq u
esti
onsint
hi
ssec
ti
oni
nthes
pac
esp
rov
ide
d.
1. Eval
uate (
3ma
rks
)

-
36÷(6)×4–(-3)
-
6-8÷2+17
2. User
eci
pro
cal
tabl
est
oev
alu
ate (
4ma
rks
)

3. Sol
vef
orxi
nt hef
ol
lowi
ngequ
ati
on.
2si
n(2x+300)-1=0 f
or 00≤x≤2700 (
3ma
rks
)
4. Th
efig
ureb
elowshowsaso
li
dregul
art
et
rap
acko
fsi
des6c
m.

6c
m
6c
m

6c
m
(
a)Sk etc
ht henetofth
es ol
i
d (
1ma
rk)
(
b)F i
n dthesurf
aceareaoft
hesol
id (
2ma
rks
)
5. Fi
ndtherangeofval
u eofxwhic
hsat
isf
ythei
nequ
ali
tybel
ow:
1 1
(2x-1)< ( x+3)<3( x+4) (
3ma
rks
)
4 4
6. Usi
ngthetr
iangleshownonthegri
d,enl
ar
getheobj
ectb
yascal
efa
cto
rof2,
cen
treA. (
2ma
rks
)

F
ORMARKI
NGSCHEMESI
NBOX0724351706 Pa
ge|88
Ma
the
mat
ic
s121/
1,2

7. F
indt
hev
alu
eofti
nth
eeq
uat
io
n: (
3ma
rks
)

()
t
1 10
512)
×( 9 =4096
64

F
ORMARKI
NGSCHEMESI
NBOX0724351706 Pa
ge|89
Ma
the
mat
ic
s121/
1,2
8. So
lvet
hef
ol
lo
win
gsi
mul
ta
neo
use
qua
ti
ons (
4ma
rks
)

2 2
x+y=16

y=2x+1
9. AKen yanbankbuysandsell
sfo
reigncurr
enci
esatt
heex
changera
tesshownbelow.
Buyi
ng(Kshs) Sel
li
ng(Kshs)
1Euro 148.56 149.00
1U.SDo ll
ar 94.22 94.50
AnAme ric
ana r
riv
edinKenyawith20,000Euro
s.Heco
nvert
edal
ltheEurosint
oKenya
nShi
ll
i
ngsattheba
nk.Hespen
tKshs.
2,
510,200wh il
einKenyaandconver
tedthere
ma i
ni
ngKeny
ashil
li
ngsint
oU. SDol
lar
satt
heb
ank.F
indtheamounti
ndoll
ar
sthathe
re
ceived. (4mark
s)
10.Sol
vef o
ryi ntheequ
ation
. (
3marks)

11.Anumberi
sfor
medbyfi
ndi
ngth
ed i
f
fer
encebe
twee
nthepr
odu
ctso
fpri
men
umb
ersb
etwe
en20a
nd30a
ndt
hato
fpri
men
umb
ers
be
twee
n1a nd15.
Fin
dthenumberf
ormed
. Wr
it
ethen
u mb
eri
nwords
. (
3ma r
ks)

12.F
indOBa
ndt
hec
oor
din
ate
sofBg
ive
nth
at a
nd (
3ma
rks
)

13.Workoutan
dgiv
ey oura
nsweri
nasimp
li
fi
edf
orm.
of1
r
( )
2 3 6 21 1 1

ee
× -3 ÷2
7 411 12 4 2
14.Thr
eet sM,Na n
dOa resu
chtha
tNisth
esout
hMan
dtot
heEa
sto
fO.
Howf
ari
sMf
romOg
ive
nth
atf
ormMt
(
3ma
oNi
r
s1.
k
7m
s
)

an
dfromNt oOis4.6m. (3ma
rks
)

2 2
15.Fa
cto
ris
eh –k ,h
enceeva
luae32822-32722
t (3ma
rks
)
2
16.Acur
vewhosegradi
entf
unct
io
nis3x -3hasit
stwos
tat
io
nar
ypo
int
s,
onea
tpo
int(
-1,
8)a
ndt
heo
the
ratp
oin
t(1,
b).
Findi
ts
eq
uati
onan
dt h
ev a
lueofb. (3mar
ks)

SECTIONII(50marks
)
AnsweranyFiv
e(5)qu
esti
onsonl
yint
hisse
c t
io
n.
17.Aho u
seistobesol
deit
heronacashb
asisorthr
oug
hal
oan
.Th
eca
shp
ric
eiss
h1,
750,
000.
Thel
oanc
ond
it
io
nsa
rea
sfo
ll
ows
:th
ere

F
ORMARKI
NGSCHEMESI
NBOX0724351706 Pa
ge|90
Ma
the
mat
ic
s121/
1,2
istobead ownp ay
me ntof10%o ft
hecashpri
ceandt h
eresto ft
hemo neyist
obepai
dthrou
ghalo
ana t10%pera
nnumco
mp o
und
int
eres
t.
Acustomerdeci
dedtob uyth
eh ous
ethroughaloan.
(a
)( i
)Ca l
cula
tetheamou ntofmoneyloa
nedtothecust
o mer. (2mar
ks)
(i
i)Thecustomerpai
dt heloa
nin3y ea
rs.Cal
cula
tethetot
alamo untp
a i
dforth
eh o
use. (4mar
ks)
(b)Findhowl ongthecust
o merwouldhav
et a
kentoful
lypayforthehouseifsh
ep a
idato
talo
fsh1,
891,750. (4mar
ks)
2
18.a)Dr awth egr
apho f2x +3x–6f ortheval
uesofxfro
m- 3t o3. (5mar
ks)
b) Us etheg r
aphtoso l
vetheequa
ti
on:
2
i) 2x +3x–6=0 (1mar
k)
2
ii
) 2x +3x–3=0 (1mar
k)
2
a) Solvetheequat
ion2x +x–7=0u si
ngt h
eg r
aph. (2mar
ks)
19.I
nt hef
igurebel
ow,vectorOA= a ndOB= .OP=2 a n dOQ=OA=3:2.

Q
Q

A
A

K K
a

OO BB P
P
(a
)Ex pre
ssthef ol
lowingv
ectorsintermso f a nd .
(i
) QB (
2ma r
ks)
(i
i
)AP (
1ma r
k)
(b
)T heli
neQBa ndAPi nt
ers
ec tatK.GiventhatQK=mQBa ndAK=nAPwh erema ndna r
escal
ar
s,byexpre
ssi
ngOKi ntwodif
fe
ren
t
ways,f
indth erat
ioAK:KP. (
7ma rks
)
20.T
hreevil
lagesR,Sa ndTares uchthatSis3k mo nabeari
ngof N300Efr
omRa ndTis4kmo nabear
ingof1200fr
omS.
(a
)( i
) Us in
gas ca
leofcmtor eprese
n t0.
5km, dra
wad iagr
amt oshowtherelat
ivepos
it
ionofvi
l
lage
sR, SandT
(
3ma rks
)
(i
i)F i
ndth edist
anceandb eari
ngo fvi
ll
ageRf r
omT (
2ma r
ks)
(b
)As t
rai
ghtma inroadrun
sfro mv i
ll
ageRt oT.F i
ndthelengt
ho fthesho
rte
s tpa
thfro
mv il
l
ageStothema i
nroad.
(
2ma rks
)
(c
)Af t
erwa l
kingfor2.2kmfromt hejuncti
onon RTalon
gthep a
tht o
wa r
dsvil
l
a geS,ast
udentnot
ic
esthatt
hean g
leofel
evat
iont
othe
topofatreein thev i
l
lageis220.Es t
imatet
heh e
ightofthetr
eeinme tr
es. (
3ma r
ks)
21.T
woe qualcir
c l
eswithce
ntresPa ndQa ndradi
us7cmi n
ter
sectatpoint
sAa ndBa sshownint
hefig
urebelow.Giventha
tthe
di
stancebetweenPa ndQi s10c m.L in
eABme etsl
in
ePQa tX.

P X Q

Ca
lcu
lat
e:

(a
)T hele
ngthofth
echordABc or
rec
tto2d.
p (
2ma
rks
)
(b
)T heangl
eAPQh enc
ea n
gleAPB (
2ma
rks
)
(c
)T heare
ao ft
hesha
dedregi
o n
. (
6ma
rks
)
22.
Thefi
gurebe
lowisthes
peed–timegrap
ho f
ajou
rne
y

F
ORMARKI
NGSCHEMESI
NBOX0724351706 Pa
ge|91
Ma
the
mat
ic
s121/
1,2

Ift
hetot
aldi
sta
ncetra
vell
edinthe80sec
ondsi
s920m, c
alcu
lat
e:
a)T heval
ueofV. (
4marks)
b)T heacc
eler
ati
onduri
ngthefi
rst15sec
onds (
3marks)
c)T hedi
sta
ncetrav
ell
edinthef
inal40se
conds
. (
3marks)
23.T
hed i
agr
amb el
owac irc
le,c
en t
reO.PQisata
ngenttotheci
rcl
ea tQandPTORisast
rai
ghtl
ine
.QRSTi
sac
ycl
i
cquad
ril
at
era
lin
whic
ha ng
leRTS=350a ndRTa ndQSa r
edi
amete
rs.Gi
vingre
asonsfory
oura
n s
wer,
fi
ndthesi
zeof
:

a) Acut
eangl
eROS. (
2ma
rks
)
b) Angl
eRQS. (
2ma
rks
)
c) Angl
ePQR. (
2ma
rks
)
d) Angl
eQPT. (
2ma
rks
)
e) Angl
ePQT. (
2ma
rks
)
24.
Thetabl
ebel
owshowsaf
iel
db o
okwithmea
sur
eme
ntso
far
ic
efi
el
d.(
AG=250m)
a) Makeaske
tchdr
awingo
ftheri
cefi
el
d. (
3ma
rks
)
G
200 F70
E60 130
100 D80
C40 60
40 B50
A
b)F i
ndthear
eaoft
heri
cefi
el
dinh e
cta
res
. (
7ma
rks
)

F
ORMARKI
NGSCHEMESI
NBOX0724351706 Pa
ge|92
Ma
the
mat
ic
s121/
1,2
KIGUMOCL UST ER
MAT HEMAT ICSAL TA
Paper2
.MOCKSKCSEPREDI CTI
ONS/..
2½h our
s
SECTIONI( 50ma r
ks)
Answerallt
heq u
esti
onsinthi
ssec
ti
on.
1. Uselogar
it
hmt abl
estoeval
uat
ethefo
ll
owingt
ofo
u rsi
gni
fi
can
tfi
gur
es. (4ma
rks
)
4.5622×0. 038
6.
82×0. 35
2. Thefif
tht
ermo fanari
thmet
icpro
gres
sio
nis11andtwent
yfi
ft
hter
msis51.
Fin
dth
efi
rs
tte
rma
ndc
ommo
ndi
f
fer
enc
e.
(2ma
rks
)

3. Gi
vent
hatma
tri
cesP,
Q,Ra
res
ucht
hatP=QRa
ndP= a
ndQ= .
Fi
ndma
tri
xR. (
3ma
rks
)

4. Solvef
o rxintheequ a
ti
on.
Log(x+11)–2l og3=l og(9–x ) (3mark
s)
5. Giventhatthemean9, 8,5,5a nd8is7;fi
ndthest
an d
arddevi
ati
ono ft
hen u
mb erto2d .
p. (
3mark
s)
6. Fi
n dthee q
uati
ono fastr
aightli
nepas
singt
hrough(2,1)andisPar
a l
l
e lt
o3y=2x+6i nthefor
ma x+b+c=0 (
3mark
s)
7. Ab ustr
a ve
ll
ingat80km/ hle
a veast
ati
onat11.
15pm.An o
therbustrav
ell
in
ga t75k m/ hle
ave
sthesa
mes t
at
iona
t11.45pmi
nthe
samed i
recti
onasth ef
irs
tone .Atwhatt
imewil
l t
heirdi
st
anceapartbe55km? (
3ma r
ks)
8. Thefig
u rebel
ows howsp ar
to fach
urchbadgewhichhasarotat
io
na l
s y
mme t
ryo fo
rde r4aboutt
hepoi
ntmark
edwit
had o
t.Dra
w
th
ec omp le
tebadge. (3mark
s)

9. Si
mpl
i
fyt
hee
xpr
ess
ion
s (
3ma
rks
)

5 3
10.a
) Ex
panda
ndsi
mpli
fy(
2-x
) inas
cen
din
gp o
werso
fxupt
oandi
ncl
udi
ngt
het
ermi
nx (
2ma
rks
)
5
b
) Henc
eapp
rox
ima
teth
eva
lueso
f(1.
98)tofou
rsi
gni
f
ica
ntf
i
gur
es. (
2mar
ks)

F
ORMARKI
NGSCHEMESI
NBOX0724351706 Pa
ge|93
Ma
the
mat
ic
s121/
1,2
11.
Cho
rdQXa
ndYZi
nt
ers
ecte
xte
rna
ll
yatQ.
Thes
eca
ntWQ=11c
man
dQX=6c
mwh
il
eZQ=4c
m

5c
m

a) Ca
lcul
at
ethelen
gtho fcho
rdYZ (
2ma
rks
)
b) Us
etheansweri
ni )abo
vetofi
ndth
ele
ngt
hof
thet
ang
entSQ (
2mar
ks)
12.Makenthesu
b j
ectofth
eformul
ain

a(1-r)
n
S= (
3mar
ks)
1-r
13.Ama nd ep
o s
it
sSh.500,000inaninve
stmentwh
ichpa
ys12%pera
nnu
minter
estc
omp
oun
dedq
uar
ter
ly
.Fin
dhowman
yyearsi
t
t
ake
sforthemoneytodoubl
e.(
3ma r
ks)
14.Av a
ria
bleVvari
esjo
int
lyasth
evari
abl
eAa n
dh .WhenA=63an
dh=4, V=84,f
ind
;
(
a)T heval
ueofVwh enA=9a ndh=7 (1mar
k)
(
b)T heval
ueofAwh enV=4.5andh=0. 5 (
2mar
ks)
15.Rat
iona
li
zeandsimpl
if
y

(
3ma
rks
)

16.Gi
venthatx,ya ndza r
einte
gersandthat8≤x≤10, 5≤y≤7, 4≤z≤6.
x+y
Findthepercenta
g eerr
orin (
3ma
rks
)
z
SECTIONI I(50ma r
ks)
Answera nyFive(5)quest
io
nso nl
yinthissect
ion.
2
17.a.
) Us ingthefir
stPri
ncipl
esandas malli
n c
rementh,de
ter
minethede
ri
vat
iv
eo fy
=3x–2. (
4ma
rks
)
2
b.)F i
n dtheequatio
ntothen or
ma lof3x–y=2a tx
=1inthefo
rmy+mx =c. (
4ma
rks
)
c.) Determinethestat
ion
arypointandidenti
fyt
hen a
tur
eofpoi
nt. (
2ma
rks
)
18.Th
et a
blebe l
ows howsma r
ksob t
ai
n e
db y50s tud
entsi
nMa t
hemati
csExa
minat
io
n
32 64 68 55 52 68 37 46 65 26
45 87 44 58 39 54 21 44 76 23
65 42 82 87 75 44 47 48 52 32
23 76 74 91 28 33 27 48 56 66
45 56 98 21 34 31 83 65 77 76

(a
.)Sta
rti
ngwi th21a ndu s
inge qual
c l
assint
erv
alsof10,ma keafrequencydi
str
ibu
tionta
ble. (
2ma r
ks)
(b
.)Ong r
idp r
o vi
ded,drawthec umulat
ivefre
quencycurveforth
ed ata. (
4ma r
ks)
(c
.)Usi
ngt hegraph( b
.)aboveesti
ma te
:
(i
.)Theupp e
rq uart
il
e (
2ma r
ks)
(i
i
.)Thelowerq uar
til
e (
2ma r
ks)
19.Li
nnet’
sb asi
cs al
aryissh.
100,000.Sh eish o
u s
edbyh eremp l
oyera n
dp a
ysan omina
lrentofsh.
2000wh ichi
sd e
ducte
dfromher
sal
ary
.Sh ei senti
tl
edtoa ne n
ter
tainmental
lowanceo f
sh .
5,000a ndares
ponsi
b i
l
ityal
l
owa n
c eofsh.
10,000.Sheh a
sab an
kloan
andhirepurchaserepaymentswh i
chs herep
aysattherateof s
h.15,
000a ndsh.
3,000permo nth
. Sh
ea l
soma ke
scoopera
ti
veshar
e
cont
ri
b ut
ionso fsh.
5,000p ermon t
h.Ca l
cula
te:
(a
)He rgrosssalar
y (
1ma r
k)
(b
)He rtaxabl
ein c
o meinKsh. (
1ma r
k)
Duri
ngt hatmo nth
, t
hetablebelo
wwa susedtodete
rmineind i
vi
d u
al r
ateo
fincometax.
Inc
omeK£p .
m. Rat
e(sh.per£)
1 - 484 2
485- 940 4
941- 1396 6
1397- 1852 7
Over1852 9
FORMARKI NGSCHEMESI NBOX0724351706 Page|94
Ma
the
mat
ic
s121/
1,2

(c
)Uset heta
bletodet
ermine
;
(i
) Hermo n
thl
yg r
osst
a x (
5ma
rks
)
(i
i
)Hern e
ttaxgiv
enthatsh
eisen
tit
ledtoata
xrel
ie
fofs
h.1056permont
h. (
1ma
rk)
(i
i
i)Hernetsal
ary
. (
2ma
rks
)
20.
TheChordXYsubtend
sana ngl
eof88oatth
ecent
reO.I
ftherad
iusoft
heci
rc
lei
s10c
m,c
alc
ula
te:

(i
)T hear
eaoft
heci
rcl
e. (
2ma
rks
)
(i
i
)T hear
eaoft
hemajorse
ctorXPY, (
3ma
rks
)
(i
i
i)Thear
eaoft
ri
angl
eOXY (
2ma
rks
)
(i
v)Thear
eaofmaj
orse
g ment (
2ma
rks
)
(v
)T hear
eaoft
heMinorsegment (
1ma
rk)
21.a
) Co mple
tet
hetabl
ebelowfort
hec
urv
esy=3c
os2xa
ndy=2s
in(
2x+30)(
2ma
rks
)
x 0 15 30 45 60 75 90 105 120 135 150 165 180
3c
os2x 3 2. 598 1. 5 0 -
1.5 -
3 -2.
598 -1.
5 0 2.
59 3
8
2s
in(
2x  1
+30) 2 -
2 -
1.732 -
1 1

b) Onag r
a php a
pe r
,drawo nthes a
mea xesthegraphof y
=3c o
s2xandy =2si
n(2x
+30)for0≤x≤180 (
4ma r
ks)
a) Sta
tet
h eamp li
tudep eri
oda ndp hasea n
g l
eo feac
hc urve (
2ma r
ks
b) Useyourgr a
pht o:
-
i
) Es t
imat
et hevalueofxf orwh i
c h3co s2x–2s in(
2x+30)=0 (1mar
k)
i
i) Est
imat
et herangeo fvaluesofxf orwh i
c h3cos2x<2sin(
2x+30) (
1ma r
k)
22.Mungai
, KoskeiandKa ndiearep art
ici
p a
tinginana t
hlet
iccompeti
ti
on.T heprob
a b
il
ityt
hatMungai
,Koskei
,andKa
ndiecompl
ete
sth
e
31 4
ra
cein,, a
ndr esp e
c t
ive
ly.F i
ndt he;Probabi
lit
ythatinacompe t
it
ion.
56 7
(a
)On l
yo neo fthe
mc omp l
etesther ace. (3mar
ks)
(b
)Al lthethreecomp l
etestherace. (
1ma r
k)
(c
)No neo fth
e mco mp l
e t
estherace . (
1ma r
k)
(d
)T woo fthe
mc o
mp l
etestherace. (
3ma r
ks)
(e
)Atl eastonec omp le
testher ace. (
2ma r
ks)
23.Us
eap ai
ro fcomp as
se sandar u l
eron l
yinth i
sque s
ti
on.
(a
)Co nst
ructtri
angleABCi nwh i
c hAB=5. 8cm, AC=4. 2cma nd<BAC=450.
Measur
eBC. (
3ma r
ks)
(b
)( i
)Dr awe sc
ribed(external
)c i
rcl
eo ftri
angleABCwh i
chtouchesBC. (3mar
ks)
(i
i
)Dr awP1t helocuso fpointswh i
chmo vesuchthattheare
aoftri
an gl
eAPBish a
lfth
ea r
eaoft
ria
n g
leABC. (
3ma r
ks)
(
i) MarkP1andP2t hepo int
swh erePa n dthec i
rcl
eme et.Me as
ureP1P2. (1mar
k)
24.Apot
te
rwa n t
stoma k ean dsel
l ser
vingb owlsandp l
ates
.Ab owluses5k gofcl
ay.Ap l
at
eu s
es4kgo fcl
ay.T
hepot
terhas40kgof
cl
ayandwa nt
stoma k eatleast4b owls.Thep ro
fitabowl i
sksh35a ndtheprof
itonap l
at
eisk
s h
.30.
(a
.)For
ma lltheinequal
iti
es. (
3ma r
ks
(b
.)Ontheg ri
dp r
ov i
dedd rawt h
ei nequali
ti
es (4mar
ks)
(c
.)Howma nyb owlsandh owma n yplat
ess houldthepotte
rma kei
no r
d e
rtoma xi
mizeprof
it
? (3mar
ks)

F
ORMARKI
NGSCHEMESI
NBOX0724351706 Pa
ge|95
Ma
the
mat
ic
s121/
1,2
MUGOI
RI/
KAHUROJ
OINTEVAL
UAT
ION

121/1
MATHEMAT I
CS
.MOCKSKCSEPREDI CTI
ONS/..
2½HOURS
Sect
ionI(50mks)
Answera
llqu
est
io
nsi
nthi
sse
cti
on
1. Eval
uatet
hefoll
owi
ng(3mks)

2. Us
esq
uar
ero
ots
,re
cip
roc
ala
nds
qua
ret
abl
etoe
val
uat
eto4s
ign
if
ic
antf
i
gur
est
hee
xpr
ess
ion (
4mk
s)

3. Sol
vethef
oll
owi
nginequal
it
ie
sandrep
res
entt
hesol
ut
io
nonas i
ngl
enu
mberl
i
n e (3mks
)
3-
2x<5
4-
3x≥-
8
4. Th
ep o
intA,BandClieonastra
igh
tli
ne.
Thepos
it
io
nvect
orso
fAandCar
e2i+3j
+9ka
nd5i
-3j
+4kr
esp
ect
iv
ely
;Bdiv
ide
sAC
i
nte
rnal
lyi
ntherat
io2:1.
fi
nd
a) Pos
it
ionvec
toro
fB (2mark
s)

b
) Di
st
anc
eofBf
romt
heo
rig
in. (
1ma
rk)

5.
Wit
hou
tus
ingt
abl
es,
eva
lua
te g
ivi
ngy
oura
nswe
rins
tan
dadf
orm (
3ma
rks
)

6.
Ift
hei
nt
eri
ora
ngl
eofar
egu
larp
oly
go sm0a
ni nde
ache
xte
ri
ora
ngl
eis .
cal
cul
at
eth
esu
moft
hei
nt
eri
ora
ngl
es.

(
3mk
s)

7.
Simp
li
fy (
3ma
rks
)

8.Astr
aightl
inepass
esthro
ughthepoi
nts(-
3,-
4)an dispe
rpendi
cul
artotheli
newhoseequat
ionis3x+2y=11andint
ers
ectthexaxi
sand
yax i
satpointAandBr e
spe
cti
vel
y.Fi
ndthelengthofAB.(4ma r
ks)
9. AKe nyanbankbuysandsel
l
sfore
igncurr
en c
iesatth
ee x
changerat
esshownbelo
w.
BUYING( KSHS) SELLI
NG( KSHS)
1Eu r
o 147.
56 148.00
1U.SDo ll
ar 74.22 74.50
AnAme r
ic
ana r
riv
edinKeny
awith20,000Eu r
os.Hec o
nvert
edallth
eEu r
osi
ntoKenyanSh i
l
li
ngsatthebank.
Hes pe
nt
Kshs.2,
510,200wh i
l
einKenyaandconver
tedtheremai
ningKenyashil
l
ingsi
ntoU.SDoll
arsatth
eb ank.
Fin
dtheamo untindol
la
rs
thatherecei
ved. (3mks)
10.Afarmerhasap i
eceofl
andmeasu
rin
g840mb y396m. Hed i
vi
desiti
ntosqua
replot
sofequalsi
ze.Fi
ndthemaximuma r
eao fo
neplo
t
(3ma r
ks)
11.
Giv
ent
hat ,
wit
hou
tus
ingma
the
mat
ic
alt
abl
eorc
alc
ula
torf
i
nd:

a) Si
nA (
2ma
rks
)
b)T a
n(90-A) (
1ma
rks
)
12.
Inthefi
gur
ebe
low,
li
neABa
ndXYa
rep
ara
ll
el
.

I
fth
ear
eao
fth
esh
ade
dre
gio
ni m2,
s36c fi
ndt
hea
reao
ftr
ia
ngl
eCXY(
3ma
rks
)

F
ORMARKI
NGSCHEMESI
NBOX0724351706 Pa
ge|96
Ma
the
mat
ic
s121/
1,2

13.
Ali
qui
dspr
ayofmas
s384gi
spa
ckedinacyl
i
ndri
cal
conta
inero
fin
ter
nal
rad
ius3.
2cm.
Giv
ent
hatt
hed
ens
it
yoft
hel
iqu
idi
s
0.
6g/
cm3,
cal
cul
at
eto2dpt
hehei
ghtoft
heli
qui
dinth
ec on
tai
ner (
3ma
rks
)

14.
(a)F
indt
hei
nve
rseo
fth
ema
tri
x (
1ma
rks
)

b) Henc
esolv
eth
esi
mul
ta
neo
use
qua
ti
onu
sin
gth
ema
tri
xme
tho
d (
2mk
s)
4x+3y=6
3x+5y=5
15.Eval
uat
e: (
3ma
rks
)

16.
Thef
ol
lo
win
gda
tawa
sob
tai
nedf
romt
hema
sso
fac
ert
ai
nan
ima
ls.
Comp
let
eth
eta
blea
ndt
heh
ist
ogr
amb
elo
w.
(
3ma
rks
)

SECTI
ONII50mks
.Answeranyfi
veq
ues
ti
onint
hes
ect
io
n
17.
Ac i
rc
ula
rla
wnissu
rrou
n dedbyapat
hofun
if
ormwi
dt
hof7m.
thea
reao
fth
epa
thi
s21%t
hato
fth
ela
wn.
a) Cal
cul
at
ether
adi
usoft
h elawn (
4ma
rks
)

b
) Gi
venfu
rth
ertha
tthepat
hsur
rou
ndi
ngthel
awnisfenc
edonbot
hsid
esb
yb a
rbe
dwireonpos
tsa
tin
ter
val
sof10met
resa
nd11
met
resonthei
nneran
doute
rsi
desr
esp
ecti
vel
y.c
alc
ulat
eth
eto
taln
umbero
fpost
sre
q u
ire
dfo
rthef
enc
e
(4mar
ks)

c) Cal
cula
tet
hetot
alc
ostoft
hepost
sifo
n epos
tco
stsh105 (
2mk s)
18 Thedia
gra
mb el
owrep
rese
ntsasol
i
dco ns
is
ti
ngofahemi
sph
eri
cal
bot
toma
ndac
oni
cal
fru
stu
matt
het
op.O1O2=4cm,
O2B=R=4.
9cm
O1A=r
=2.1cm

F
ORMARKI
NGSCHEMESI
NBOX0724351706 Pa
ge|97
Ma
the
mat
ic
s121/
1,2

a) Dete
rminethehei
ghtofthecho p
pedo f
fconeandhen c
etheh e
ightofthebi
ggercone. (3mk s
b) Cal
cula
tethesur
fac
ea re
ao ft
h esol
id. (
7mks)
19 Thecoordi
nat
esofatri
angleABCa re A(1,1) B( 3,1)a nd C( 1,3).
(a
)Plotthetri
angl
eABC. (
1ma r
k)
(b
)T r
i
C'
.
angl
eABCu nder
g o
esat ra
n sl
at
ionvect
o ()
2
r .
2
Obtai
nt hei
mag eofA'B'C‘undert
hetra
ns
(
fo
r
2ma
ma
r
t
k
s
i
o
)
n,wr
it
eth ec o
o rdi
nat
esofA'B'

(c
)A'B'C'u nder
goesar ef
lect
iona l
ongthelin
eX=0,o bt
ai
nt hec oo
rdin
atesandp l
otontheg r
aphpoin
tsA"B"C" ,unde
rthe
t
ransf
ormati
on (
2ma r
ks)
(d
)T hetr
iangl
eA"B"C” ,undergoesanenlar
gementsc a
lefact
or-1,centeror
igi
n. Ob t
ai
nthec o
ordi
nat
esoft heima geA'"B"'C"'
.
(2mark
s)
(e)Thetr
iangl
eA"'B"'C" '
un de
rgoesarotat
io
nc ent
er(1,-2)ange1200.Ob
l tai
nthecoord
inat
esofth
eimageAivBivCiv.
(2ma rks
(f
) Wh i
chtr
iangl
esar
ed i
rect
lycon g
ruent
? (
1ma r
k)
20.Usi
ngar ul
erandcompasseson l
y,c
o n
str
uctatr
iangleABCs uc
ht hatBC=8cm, a
ng eABC=600
l
AndangleBAC=450
a)o ntheSamed i
agr
am, measurethel
engthof: (
5ma r
ks)

i
) AC
i
i
) BC
b
) Dr
awt
hec
ir
cumc
ir
cl
eoft
het
ri
ang
leABC. (
2mk
s)

c
) Cons
tr
uctt
hel
ocusofap
oin
tPwi
th
int
het
ri
ang
leb
ysh
adi
ngt
heu
nwa
nte
dre
gio
nin
sid
eth
eci
rc
umci
rc
lesuc
hth
att
hef
ol
lo
win
g
co
ndit
io
nares
ati
sf
ie
d: (
3ma r
ks)

i
) Pi scl
osert
oAt hanB
i
i) Angl
ePAB=a nglePAC
21 a) Drawth egraphofthefuncti
onbelowonthegri
dpro vi
ded
2
y=2x –7x–2f orthevalu
esof-1≤X≤6 (5mks)
2
b)F r
omy ourgraphd et
erminet
h eroot
softhefu
ncti
on.2x –7x–2=0. (
1mk)
2
c) Bydra
wingas ui
ta
b l
egrapho ff
unct
iony=2x–7o nthes a
mea x
is,
sol
vet
hesimult
ane
ousequat
io
nsy=2x –7x–2a ndy=2x–
7. (4mks)
22.Euni
cebough tora
ngeswo rt
hk s
h45, whil
eShar
ons p
en tthes
amea mo
untofmoneybutbo
u g
htatadi
sco
unto f75cen
tp e
roran
ge.
a)I f
Eunic
eb oug h
tano ran
gea tshx,wri
tedownasimpli
fi
e dexp
res
sionf
ort
hetota
lnumberofor
anger
sbough
tb yEuni
ceandSharon
.
(3mks)

b
)If
Sha
ronb
oug
ht2mo
reo
ran
gest
henEu
nic
e,f
i
ndh
owmu
che
achs
pen
tona
nor
ang
e (
5mk
s)

c
)Fi
ndt
het
ota
lnu
mbe
rofo
ran
gesb
oug
htb
yEu
nic
ean
dSh
aro
n (
2mk
s)

23.
a)Thef
igu
reb
lews
howsave
loc
ityt
imegra
phofano
bje
ctwh
icha
cce
ler
atef
romr
estt
oav
elo
cit
yVm/
sth
end
ece
ler
atet
ore
sti
na
to
talo
f54sec
ond
s.I
fth
ewholejo
urne
yis810m

a
)Fi
ndt
hev
alu
eofV (
2mk
s)

b
) He
ncef
i
nd:

i
) T h
eval
ueoftwhenth
ep a
rti
cl
eisi
nsta
ntane
ousl
yatr
est (
2mk
s)
i
i
)T h
eto
tald
ist
anc
etrav
ell
edbythepar
ti
cl
ed u
rin
gthef
ir
st4s
eco
nds
. (
2mk
s)
i
i
i)T
hemaxi
mumv el
oci
tyat
tai
nedbyth
ep a
rti
cl
e (
2mk
s)

F
ORMARKI
NGSCHEMESI
NBOX0724351706 Pa
ge|98
Ma
the
mat
ic
s121/
1,2

24.
Auma
,Ha
sss
ana
ndKa
mauc
omp
ete
dinag
ameo
fda
rts
.Th
eirp
rob
abi
l
it
ie
sofh
it
ti
ngt
het
arg
etsa
re r
esp
ect
iv
ely
.

a
) Dr
awap
rob
abi
l
it
ytr
eed
iag
ramt
osh
owa
llt
hep
oss
ibl
eou
tco
me (
2mk
s)

b
) Ca
lcu
lat
eth
epr
oba
bil
i
tyt
hat
:

i
) Noo nehi
tth
etarg
et (
2mk
s)
i
i
) On l
yoneoft
hemh i
tthetar
get (
2mk
s)
i
i
i) Atl
eas
toneoft
hemh i
ttar
get (
2mk
s)
i
v) Atl
eas
toneoft
hemmis s
edthet
arg
et (
2mk
s)

MUGOIRI/
KAHUROJOINTEVALUAT
ION
121/
2
MATHEMATI
CS
.MOCKSKCSEPREDI
CTIONS/
..
2½HOURS

SECTI
ONI
Answe
ral
lqu
est
io
nsinth
iss
ect
io
ninthesp
acepro
vid
ed
1.
Useloga
ri
th
mscor
rectt
o4dec
imal
spl
acetoev
alu
ate (
3mk
s)

2. So
lvet
hee
qua
ti
on

(
3mk
s)

3.
If f
i
ndt
hev
alu
eof
aan
dbwh
ereaa
ndba
rer
ati
ona
lnu
mbe
rs. (
3mk
s)

4. Gi
vent
hea
rit
hmeti
csequ
enc
e4,
11,
18…F
ind
a)T h
ecommondif
fe
renc
e (
1mk
s)

b
)Th
esu
moft
hef
i
rstt
ent
erms (
2mk
s)

4
5. a
) Ex
pan
d u
ptot
het
ermi
nx (
2mk
s)

8
b
) He
ncee
val
uat
e(0.
95) (
2mk
s)

6. Aq
uan
ti
tyPi
spa
rtl
yco
nst
anta
ndp
art
lyv
ari
esa
sth
ecu
beo
fQ.
IfP=25wh
enQ=1,
P=44wh
enQ=2.
Fin
dth
eval
ueofPwh
enQ=5
(
3mks
)

7. Ma
kePt
hes
ubj
ecto
fth
efo
rmu
la

(
3mk
s)

8. So
lvef
or i
nth
era
nge00≤ ≤3600g
ive
nth
at (
3mk
s)

9. Ti
sat
ran
sfo
rma
ti
onr
epr
ese
ntb
yth
ema
tri
x .
Und
erT
,as
qua
reo
far
e m2i
a10c sma
ppe
don
toas
qua
reo
far
ea

110cm2fin
dt h
ev al
ueofx. (
3mk
s)
10.T heg ra
die
ntfunct
ionofcur
vetha
tpass
est
hrought
hepoi
nt(
-1,
-1)i
s2x+3.
fi
ndtheeq
uat
ionoft
hecur
ve. (
3mk
s)
2 2
11.2x +2y -
6x+10y+9=0ist
heequat
ionofac
ir
cle
.Fin
dth
eradi
usandthecen
teroft
heci
rc
le. (
4mk
s)
12.I nwh atpr
oport
ionmu s
tte
ac o
sti
ngsh76andsh84perk
gbemi xe
dtopro
duceteac
ost
ingsh81p
erkg (3mk
s)
13.Ch ordsABandCDo faci
rc
lemeetatX (
3mk
s)

F
ORMARKI
NGSCHEMESI
NBOX0724351706 Pa
ge|99
Ma
the
mat
ic
s121/
1,2

I
fAB=8c
m,BX=5c
man
dDX=6c
m.Ca
lcu
lat
eth
ele
ngt
hof
cho
rdCDt
o2d
p (
3mk
s)

14.Adamcont
ai
ning4158m3o
fwate
rist
obedr
ai
n e
d.Apumpiscon
nect
edtoapi
peo
frad
ius3.5cma ndt
hemach
ineope
rat
esf
or8
ho
urspe
rday.Wate
rfl
owsatt
hepi
peatt
her
ateof1.
5mperse
cond.
Findth
enumbe
rofdaysitt
ake
stodra
int
hedam
(3mk
s)
15.Sol
vet
heequa
ti
onb e
low (3mks)

16. i
str
unc
ate
dto2d
.pc
alc
ula
tet
hep
erc
ent
agee
rro
rind
oin
gso (
3mk
s)

SECTIONII(a
nsweranyfiv
eq u
esti
on)
Answersanyfi
veequa
tioninth
issect
io
n
17.
Ab usi
ness
ma nobt
ainedaloanofksh450,
000fr
omaba
nkt
obuyamat
atuv
alu
edatt
hes
ameamou
nt.
Theb
an kc
harge
sin
ter
esta
t
24%p eran
n umcompo u
n d
edq ua
rte
rl
y
a) Cal
culat
ethetot
alamountofmo ne
ythebus
ine
ssma
npa
idt
ocl
eart
hel
oani
no nea
ndha
lfy
ear
s (4mks)

b
)T h
ea v
erag
ein
comerea
li
sedf
romt
hemat
atuperda
ywa sk
sh1500.
Thema
tat
uwo
rke
dfo
r3y
ear
sata
nav
era
g eo
f280d
aysp
ery
ear
.
Cal
cul
atet
het
ota
lin
comefro
mthema
tat
u,fo
rtheth
reeye
ars
. (2mks)

c
) Duri
ngth
ethr
eeyea
rs,
theval
ueo
fthemat
atude
prec
iat
edatt
hera
teo
f16%p
era
nnu
m.I
fth
ebu
sin
ess
mans
oldt
hemat
at
uati
tsn
ew
va
lue,
cal
cul
at
eth
etota
lpro
fi
there
ali
sedb
ytheendoft
hre
eyear
s. (
4mks)

18.
a)i
)T aki
n gtheradi
uso ftheeart
h,R=6370kma ndπ=22/7,
cal
cul
at
ethesh
orte
stdi
sta
n c
ebetwee
nthet
woc i
ti
esP(600N,
290)
0 0
andQ( 60N,31E)a l
on gth
ep ar
al
lelof
lati
tude. (3mks)
ii
)I fi
tis1200h r
sa tP,wh ati
stheloc
alti
mea tQ. (3mks)
ii
i
) Ana erop
lanefl
ewd u
es out
hfromap oi
ntA( 600N,
450E)t
oap o
intB.T
hed i
st
ancecov
eredbyt
heaer
opl
an ewas
8000k md et
erminethep os
it
ionofB. (4mks)
19.
(a)F il
lthetabl
eb el
o wforthecurv
esgiv
enb yy=3s i
n(2+30)andy=Co s2for
v
a l
uesintherangeO0180 . (
2mks)

 0 15 30 45 60 75 90 120 150 180


y=3Sin( ) 1.
2+30 50 2.
60 2.
60 1.50 0.00 -1.
50 -2.
60
y=Cos2 1.
00 0.
87 0.
50 -0.50 -0.87 -
0.87 -
0.50

(
b)Dr
awt h
eg r
aphsofy=3Si n(2+30 )=Cos2onsameaxes. (
2mk
s)
(
c)Us
ey ou
rg r
aphtosol
vetheequat
io
n3Si n(2+30)-Cos2=0. (
2mk
s)
(
d)Det
ermi
nethefol
lowi
ngfromy o
urg r
aph:
(
i) Amplit
udeofy=3Si n(2+30). (
1mk
)
(
ii
)Pe ri
odofy=3Si n(2+30 )
. (
2mk
s
(
ii
i
)Ph asedi
ff
eren
cefory=3Si n(2+30 )
. (
1mk
)

20.
Dra
wac
umu
lat
iv
efr
equ
enc
ycu
rvef
ort
hef
ol
lo
win
gda
tawh
ichs
howst
hema
rkso
bta
ine
dby80f
ormfou
rstud
ent
sinas
cho
ol.
(5mks)

Mark
s 1-
10 11-
20 21-
30 31-
40 41-
50 51-
60 61-
70 71-
80 81-
90 91-
100
f
req
uenc
y 3 5 5 9 11 15 14 8 6 4

Usey
ourgr
aphtofi
nd:
a
)T h
elowerq
uart
il
eQ1)
,media
n(Q2)a
n dupp
erqu
art
il
e(
Q3) (
3mk
s)
b
)T h
epassmar
kif85%ofth
esestu
dentar
etopa
ss (
2mk
s)
F
ORMARKINGSCHEMESINBOX0724351706 Pa
ge|100
Ma
the
mat
ic
s121/
1,2

21.
Thed
iag
ramb
elo
wsh
owst
ri
ang
leOPQi
nwh
ichMa
ndNa
rep
oin
tso
nOQa
ndPQr
esp
ect
iv
elys
ucht
hatOM= OQa
ndPN=

L
PQ. i
nesPMa
ndONme
eta
tX.
P

N
X

Givent hatvectorOPO=pa ndv e


ctorOQ=qEx pres
stM
hefoll
owinginter
mso fpanQq
d
a) Ve ctorPQ (1mk)
b) Ve ct
o rPM (2mks)
c) Ve ct
o rON (1mk )
Bye xp r
essingvectorOXi ntwod if
feren
twa y
s,det
erminetheval
uesofhan dksuchthatv
ect
orOX=kON and,PX=h PM
i)Expr e
ssOXi ntermo fpan dqintwod if
fer
entways. (2mks)
ii
)F i
n dthev al
ueo fhan dk (2mks)
ii
i)Findt herati
oPX:XM (1mks)
22.a)T h efir
sttermo fana ri
thmeti
cp rogr
essi
on(AP)is2.Th
es umo ft
hefi
rst8te
rmsofAPis256.
i) F indthec ommo nd i
ffe
renceof AP (2mks
)
i
i
) Givent hatthes umo ft
h efi
rstntermso ft
heAP416. Fi
ndn (
2mks)
b)Th e3rd,5thand8thtermso fanot
h e
rAPf ormsthefi
rs
tthreeter
mso fageometr
icprogr
essi
on(GP)
.I
ftheco
mmo ndif
fer
enc
eoft
heAP
is3
Find
i
) T h ef
irsttermo fGP (4mks
ii
) Thes u mo ft
h efi
rst9te r
mso ftheGPt o4s .
f (2mks)
23.I nthef i
gureb el
ow, VABCDi sar i
gh tpy
ramidonasq u
arebase.Po i
ntVisver
ti
cal
l
yabo v
ethemi
ddleoft
heba se
.
PQ=10c m, VR=13c m.Mi sthemid po
intofVR.

F
ind
(
a) (i)T
heleng
thPR. (
2mk
s)
(
ii
)Thehei
ghtofth
epyra
mid (
2mk
s)
(
b)Theangl
ebetween
(
i) VRandthepla
nePQRS. (
2mk
s)
(
ii
) T h
ean g
lebet
weenMRandb
asePQRS. (
2mk
s)
(
ii
i)Thepl
anesQVRa n
dPQRS. (
2mk
s)

24.
Thev
ert
ic
esA(
-2,
-2)
,B(
-4,
-1)
,C(
-4,
-3)a
ndD(
-2,
-3)i
sun
derat
ran
sfo
rma
ti
onb
yma
tri
x
E=

a
)i) f
i
ndt
hec
oor
din
at fA1B1C1D1
eo (
2mk
s)

i
i
) An
oth
erma
tri
xS= t
ran
sfo
rmsA1B1C1D1.
Fin
dth
eco
ord
ina
t fA11B11C11D11
eo (
2mk
s)

F
ORMARKI
NGSCHEMESI
NBOX0724351706 Pa
ge|101
Ma
the
mat
ic
s121/
1,2

b
) Ma
tri
xR= f
urt
herg
ivsA11B11C11D11at
e ran
sfo
rma
ti
on.
fi
ndt
hec
oor
din
at fA111B111C111D111a
eo ndp
loto
nth
egr
aph

pr
ovid
ed (2mks
)
c
) Fi
ndthema t
ri
xrep
res
e nt
in
gthetrans
for
mati
onmappin
gtheimagef
oun
din(a
)(i
i
)ab
ovebac
ktot
heobj
ectABCD.
(
2mks)
d
) A(
2,2)B(
2,0)an
dC(3,2)ar
ecoordina
teoft
heve
rti
cesoft
ri
angl
eABC.Tr
ia
ngl
eABCund
erg
oesas
hearf
acto
r3p a
ral
le
ltoxa
xis
.
Det
ermin
ethecoor
din
a t
esofA1B1C1. (2mks
)

F
ORMARKI
NGSCHEMESI
NBOX0724351706 Pa
ge|102
Ma
the
mat
ic
s121/
1,2
GAT ANGAF ORM4MAT HEMAT ICSPAPER1
MAT HEMAT ICSPAPER1
FORMF OUR
ENDT ERM2EXAM.
TIME2½HOURS.
SECT IONA( 50MARKS)
1. Witho utu singma the ma ti
cal t
ableso rac al
culatoreval
uate :
- (
3mks)
5184
6x -18÷9+( 5--3)
2. Simp l
ifyc o mp letel
y:
2 2
12x -11x y+2y
3 2
18x -8x y
3. Ma r
yb ou gh tma i
zea ndb e
an sfromawh olesa
lerwh osoldt hemtoheratshs.36a ndshs.60p erkgresp ect
ive
ly.Marymixedthe
ma i
zea n dme a nsint her ati
o5:3. Ifsh ehastoma keapro fi
tof30%,atwh atpri
ceshouldshes el
l1k go fthemixt
ure?
(4mk s)
4. Atrian gleh ass idesAB=8c m, BC=10c ma ndAc =13cm. Ca l
cul
ateth
ea r
eao ftr
iangl
eABCc orrec
tto1d .p. (3mks)
5. Ah alfo ft hei nteri
o ra n g
leso fann -si
d edirr
egularpo l
ygo nmeasurs1350e
e ach.Therema i
ningh al
fme a su
res1650each. F
indn .
(
2mks)
6. Evaluate: (3mks)
1 1 253
o f
2 + (-)
9 3 332
2 1 1
o f3 ÷
5 3 3
0 0
7. Findxi fc o s( 3x-30) =sin( 7x+50) (
3mk s) (3mks)
8. Ana rco fl eng th12c ms ubt
en dsana n gleof5/ 6πca tthec ent
reofacir
cle.Fi
ndtheradiuso ftheci
rclei nter
mso f
.
(
2mks)
9. Alad derwh ichi s6c ml ongl eansag ai
n stawa l
l.Ifthetopo fth
eladderi
s4ma bovethegroun d,howf arfromthewa l
listhefootof
thelad der. (
Gi vey ou ra nswe rto1.d.p.) (
3mks)
10.Wh atin tergra lvalueso fxsatisf
yth efo l
lowingp ai
rsofineq ua
li
ti
es? (
3mks)
5/2–x /2≤3
½x+2>x- 1
11.Findth ev alu eo fyint h eequ ati
on . (4mks)
243x32y =81
729x3y÷( 32y-1)
12.Juman ee dst oi mpo rtac arf romJ apanwh oseco s
tisUS$5000o uts
ideKeny a
.Heintendstob uythec arthroughana gentwhod ea
ls
i
nJ ap ane seYe n.Th ea g e
ntwi llchargeh im20%c ommi ssionofthepri
ceofthec a
randaf u
rthersh.80325J a
paneseYenf o
rshipment
ofthec a r.Ho wma nyKe nyaSh il
li
n gswi llbespenttoo bt
a i
nthec a
rgive
nt hefol
lowingexcha ngerate
s?
(3mk s
)
Curren cy Bu ying sel
li
n g
1USDo l
lar 81.70 82.50
100J apan eseYe n 67.45 70.35
13.InaKi n yo zish op,ac u stome rma yb es erv
edb yo neofthet wobarber
s.Ona ver
ageo nebarbertakes10mi nute
swh i
letheo t
hertak
es
12mi nute stos er
veo nec ustome r.Ifthetwob arbersstarttoser
vecusto
me rsatthesamet i
me ,fi
ndt hes hor
tes
tti
mei ttake
stos e
rve
44c us t
o me rs. (3mk s)
14.Gi
a
vent
) Co
h
l
a
u
tth
mnv
ep
e
o
c
s
t
o
i
ti
rp
onv ect
o rsofpo i
n tPisp =5a–2bwh () ()
er
ea= a
3
2
ndb= f
4
1
ind
:-
(
2mks)
b) P’
15.Ab u
t
s18.
h eima
8ml o
geo
n
fPu
gmo vi
n
n
de
ga
rt r
an
t50k
sla
m/
ti
o
ho
nv
v
()
e
e
c
r
t
t
a
o
k
r
e
-
-
4
6
sas ec
(
o
1mk
ndb
)
usmo vi
ngat45k m/hin22½s .Fi
n dthelengtho ft
hes e
condb us.
(
4mks)
16.Diana hb o ug ht15s hirtsand8t r
o usersfors h.
11250. He rbrot
herJamesb ought20%l e
sss i
mi l
arshirt
sa nd25%mo resimil
ar
tr
ou sersf romt hesa mes hopf orshs11, 700. Findthec ostofashir
t.
(4mk s
)

SECTI
ONB( 50MKS)
Answeronl
yfivequest
ionsf
romt hi
ssec
ti
on
17.Arhombusha sit
svert
ice
sPQRS. Theco-
ordi
nat
eso
fthev
ert
exP,Q,
andSo
fth
erh
omb
usa
reP(
-1,
3)Q(
2,4)a
ndS(
0,y
).Th
e
di
agonal
sQSa ndPRme eta
tp o
intM.
Giv
enthatt
hee quati
onofli
nePRi sy=x
+4,
a)F i
ndtheequatio
no fdi
agonalQS. (2mks
)
b)F i
ndthecoordinat
esofthemidpoi
ntMo fQS. (3mks
)
c)F i
ndthecoordinat
esofS. (3mks
)
d) Pl
otther
h o
mb u
so nthegri
dprovi
d e
dandfi
ndthea
reaof
therh
ombus. (3mks
)

F
ORMARKI
NGSCHEMESI
NBOX0724351706 Pa
ge|103
Ma
the
mat
ic
s121/
1,2

18.I
nane xper
imentthele
ngthof100rat
swe r
eme as
uredt
othen e
are
st1cman
dt a
bula
tedasfol
l
ows;
-
Lengthincm 20-
24 25-29 30-34 35-
39 40-
44 45- 49 50-54 55- 59 60- 64
Frequency 1 4 x 20 25 26 7 1 1
a)F in
dt heval
u eofx
. (1mk)
b) Using42a stheass
umedmeanc a
lcu
lat
etheme a
nleng
th. (
5mks)
c) Calcul
atethest
andar
ddevi
ati
onoftheabo
ved a
ta. (
4mks)
19.Asol
idisma deupofaconic
alf
rust
rumandah e
mispher
ic
altopassho
wninthef
igur
eb e
low.Thed
ime
nsi
onsar
easin
dica
ted
.(Us

=22/7)

7c
m

8c
m

4.
2cm
a)F i
ndtheareaof:
i
)T h
ec i
rcul
arbase (
2mk
s)
i
i)Thecurvedsur
fac
eo ft
hefr
ustr
um. (
3mk
s)
i
ii
)Theh e
mispher
ic
alsurf
ace
. (
2mks
)
b) Asi
milarsol
idhasatot
alsur
fac
eareaof81.
5cm2.Dete
rmi
nether
adi
usofi
tsba
se. (
3mks
)
20.I
nthefi
g ur
eb e
low(notdr
awntoscale
)AB=8.4c
m, AC=6.5c
m,AD=7.2cm,CD=5.6c
man
dan
gleCAB=500.

Ca l
culat
ec orrectto2d .p.
a)T helengthBC (2mks
)
b)T hesizeofa n
g leABC (3mks
)
c) Si
zeo fangleCAD (3mks
)
d) Cal
cu l
a t
ethea reao ft
ria
ng l
eACD. (2mks
)
21.Fourtown sC, S,Ta ndYa res uchthatSi s150kmo nab ear
ingof0250fromC. Ti
s300kmo nabea
ri
ngof1450f
romCandYisd
ue
Westo fCa ndo nab ear
ingo f3000fromT .
a) Usi
n gas cal
eo f1c m=50k m, dra
wa na ccurat
escaleshowingtheposit
io
nsofS,C,TandY. (5mks
)
b) Byme a s
uremen tfromy oursc a
ledrawingd ete
rmine:-
i
) T hed i
stancean db ear
ingofYf romS. (3mks
)
i
i)T hed i
stanceST . (2mks
)
22.Pet
erb oughttwob ull
sandn inesheepfo ratotalo
f Ksh98,
200.Ifheh a
db o
ughtth
reebul
lsandf
oursh
eep
,hewouldh
avesp
ent
Kshs2,200less.
a)F or
mt woe quati
on storepresentth
ea bovei nf
ormati
o n
. (1mk)
b) Usema t
ri
xme thodt od e
terminethec osto f
ab ul
landth a
tofasheep. (4mks
)
c) Pet
erla t
ersoldth ea n
imalsh ehadbo ughtma ki
ngap r
ofi
tof25%pe rbul
land30%p e
rsheep
.
i
) Ca l
cu l
a t
etheto t
al amounto fmo ne
yh er ecei
ved. (2mks
)
i
i) Determinecorrectto1d .
pth eperce
n ta
g eprofi
tPetermadefro
mt hesale
softheani
mals
. (3mks
)

F
ORMARKI
NGSCHEMESI
NBOX0724351706 Pa
ge|104
Ma
the
mat
ic
s121/
1,2

23.T
hefl
oorofabal
conymea
sure
s5.4mb y1.
8m.Squa
resl
absa
repl
acedi
napat
hofuni
f
ormwi
dt
haro
undtheb
alco
nya ss
hown
.The
s
had
edp a
rtsa
retho
sewit
hcorne
rslab
s.Thes
ide
softh
ecorn
ersl
absar
eeq
ualt
oth
ewidt
hoft
hep
ath.Al
lot
hersl
absar
esmal
le
r
a
ndofsi
de12cm.Thear
eaofre
ctan
glepar
twi
thnosl
abs50,
544c
m2.

a)F i
ndt
hewidthofth
ep a
rtwi
thn osl
abs
. (
4mk
s)
b)F i
ndt
hetota
lnumberofsl
absinth
eb a
lco
ny. (
4mks)
c)T h
ecosto
fcornersl
abiss
h.60andthecos
tofe
achs
mal
l
ers
labi
sKs
h.8.
Fi
ndt
het
ota
lco
sto
fal
lsl
abs
.( 2mk
s)
2
24.Gi
vent
hat
:-y =(
x –9)(x+3)

d
y
a
)F i
nd (
1mk
)
dx
2
b
)F i
ndthegra
dientoft
hefunct
io
ny=(x–9)(x+3)a
tth
epo
int
swh
erei
tcu
tst
hex
-ax
is. (
2mk
s)
c
)F i
ndthecoor
dinat
esofth
eturni
ngp
oint
s. (
3mk
s)
2
d
) Ske
tchthecur
vey=(x -
9)(
x+3) (
3mk
s)

GATANGAFORM4MAT HEMATICSPAPER2
MATHEMATI
CSPAPER2
FORMFOUR
ENDTERM2EXAM.
TI
ME2½HOURS.
SECT
IONA(50MKS)
1. Usel
oga
ri
th
mt a
blet
oeva
lua
te. (
3mk
s)

3
0.
0246X142

0.
002X1.
14

2. Sol
vet
heequa
ti
on;2cos(3x+12)=- 1f
or00 ≤x≤ =1800 (
3mk
s)
3. Sol
vef
orxint
heequat
ion: (
3mk
s)
Lg8(x+6)-L
o og8(x–3)=2/3
4. Int
hef
igu
rebel
owPQRSi sac y
cli
cquad
ril
at
era
l.L
inePSi
spar
al
le
ltoQRa
nda
ngeQPS=1050.
l
QRist
hedi
amete
rofthecir
clece
ntr
eO.

R
P
P

Det
ermineth
esiz
eo fan
glePRS. (
3mks)
5. Ea
chs i
deofaregu
larhexa
gonmeas
ures20.
5cm.
Wh ati
sth
emax
imu
mpe
rce
nta
gee
rro
rinc
alc
ula
ti
ngt
heh
exago
n’spe
rime
ter
.
(3mks)
6. MakeAt hesub
jec
to ft
hefor
mul
a. (3mks
)
FORMARKINGSCHEMESI NBOX0724351706 Pa
ge|105
Ma
the
mat
ic
s121/
1,2
2p L-A
W=
n 3k
7. Simpl
ify 2+√3 a n dgiv
ey ouranswerintheform ab+ cdwh er
ea,b,
candear
eInteger
s.
4+√6 e (
3mks)
5
e5tht 5
8. Expa
n d(3–½x )upt
ot h erm. Henceu s
et heexpa
nsi
ontoeval
uate(
3.25)cor
rec
tto1d ec
imalpl
ace.(4mks)
9. Theco-ord
inat
esoft
h elong
estcho r
do f
ac i
rcleareA(0,2)andB(6,10).F
indthee
quat
ionoftheci
rc
leinthef
orm
2 2
x +y +ax+b y+c=0wh erea,b,andca reconstant
s. (3mks)
8. TapAc anfi
llata
nkin20mi nut
eswh il
eT a
pBc anfil
lt
hesametanki
n15mi nute
s.Whentheta
nkisful
l
,itcanbeempt
iedbyCin30
minute
s.Ini
ti
all
ywhent het
ankise mpty
,allta
p sareopene
d.Aft
er5minute
stapBiscl
ose
d .F
indth
etotalt
imeta
kenfo
rthetankt
o
beful
l.(3mks)
11.Inth
ef i
gurebel
ow,BTi sata
n g
en tt
othec i
rcl
ea tB.AXCTa n
dBXDa rest
rai
ghtl
ine
s.
AX=6c m,CT=8cm,BX=4.8cma ndXD=5c m. (2mks)

C T

Fi
ndthel engt
ho fBT
12.Twob a
gsAa ndBe a
chc ont
ai
nami xtur
eofyel
l
o wandgree
nb a
n a
nasallofthes
ames i
ze.BagAc ont
ai
ns12yell
owa n
d15g re
en
bana
n a
s. BagBc o nt
ains8yel
lowand13g re
enb an
anas.F
indtheproba
bil
ityofpi
cki
ngag ree
nb an
anaatran
dom.
(
3mk s
)
13.Fi
ndtheq uarti
ledevi
a t
ionoft
hedatagive
nbelow. (3mks
)
12,
17,6,9,8,
9,16,15,10

14.Theposi
tionv e
c t
orsofPa ndqis3i-2j+kand2i+j –3kres
pecti
vel
y.Determi
nethecol
umnv ect
orPQa ndhenc
ecal
cula
tet
he
l
engthto2d e c
ima lpl
aces. (
3mks)
15.Aquanti
tyPv a
riespart
lyasnandp a
rtl
yasthesquar
eofn.Wh e
nP=- 3,n=-1andwh e
np =18,n=2.Fi
ndpwh enn=1.
(3mks
)
16.With
o u
tu singas etsquar
eorap r
otr
a c
tor
,cons
truc
ttri
ang
leABCs uchtha
tAB=AC=5. 4c mandangleABC=300.Det
e r
min
e
shor
tes
td ist
ancefromAt oBC. (
4mk s
)

SECTIONB(50MKS)
Answeranyf
i
veques
ti
onsf
romth
isse
cti
on
17.Thein
c o
metaxr
ate
soface
rta
inye
arwer
eass
howni
nth
eta
bleb
elo
w:-

Month
lyi
ncome(Kshs
) T
axr
atei
nKs
hsp
erKs
hs20
1 - 18000 2
8401 -30000 3
30001 -36000 4
36001 -48000 5
48001 -a ndabo
ve 6
7

IntheyearMr .Wander
a’
smo nthlyearni
ngwer
easfol
lows;
-
Basi
cs a
lary Ks h
30000
Houseall
o wance Ks h
s12000
Commu terall
owance Ks h
a2500
Me d
icala
llowance Ks h
s3500
Hewa sentit
le
dt of
amil
yrel
iefo f
Ks h
s1240p e
rmo n
th.
a
) Calc
u l
at
eh ist
axabl
eincomepe rmo nt
h. (
2mks)
b
) Hisnetta
xp ermo n
th. (5mk
s)
c
) Apartf
romi ncometaxt
hefol
lowin gmonthl
ydedu
cti
onsweremad
e;-NHIFofKs
hs250;s
erv
icecha
rgeo
fKshs150;L
oanre
pay
men
t
ofKsha6000a ndco-or
pera
ti
v esharesofKs
hs2600.Cal
cul
ateMr
.Wa nd
era
’smont
hlyn
etinc
o mef
romhise
mployme
nt.

F
ORMARKI
NGSCHEMESI
NBOX0724351706 Pa
ge|106
Ma
the
mat
ic
s121/
1,2
(
2mk
s)
18.At
a) (i
ran
)Onag
s
forma
ri
d
t
i
,
d
o
r
nr
awt
ep
r
r
e
i
a
s
n
e
g
n
l
tedb
ePQRa
yth
n
ema
dit
( )
si
t
ri
x
mag
-2 0
0 -
eP’ Q’
2
R’
ma
o
p
nt
sP(
h
es
1,
a
3)
mes
,Q(
e
3,
to
fa
3)a
xes
n
.
dR( 2,1)ont
oP’ Q’andR’res
pec
(
t
iv
e
3mk
ly
s
)
.

(i
i
)De t
erminet hea r
eao ft ri
angleP’Q’R’. (
2mk s
)
b
i
) An
) Plo
o t
h
ttr
e
rt
i
ang
r
an
l
sf
eP”
orma
Q”
t
i
R”o
onr
nt
e
h
pr
es
es
a
ent
mea
edb
xesa
( )
yma
sa
t
r
b
i
o
x
ve
0 -
-
1 0
.
1
mapsP’Q’R’ont
oP” Q”R”.
(
2mk s
)
i
i
c
) De
) De
s
t
c
er
r
ib
mi
et
n
h
et
et
h
ra
ema
ns
t
f
o
r
r
i
ma
xo
t
ft
i
o
h
nr
es
e
i
p
n
r
g
e
l
s
et
en
r
a
t
n
e( )
db
sf
or
y
ma
0 -
-
1 0
t
io
1

nwh ichma psP”Q”R”o n


toPQR.
(
1mk
(
2mk
)
s)
19.Ac onstruct
ionc omp anyh astotransport450b agsofcementa nd360ironsheet
stoac onst
ructi
onsit
e.Hehastwov ehi
cl
esathi
s
di
sp osal
. Alorrywh ic
hc a nc arr
y90b agso fceme ntand30i r
ons heet
spertri
p.Ap i
ckupwh ichcancarr
y45b ag
so fc
ementand60
i
rons heetsp ertr
ip.Eac hv ehicl
ema ke satl
ea stonetri
p .I
tprofi
tshim900p ertr
ipfort
helorryandKshs600pertr
ipfo
rthepi
ckup.
IfXa ndYr epresentlorriesa ndp i
ckupsr es
pect i
vely
,
a) Writedo wnt woi nequali
tiesinXa ndYa p
artfromX≥1,a ndY≥1 (
2mk s
)
b) Representthei nf
o r
ma ti
o ng raphica
ll
y . (
4mks)
c)I ft
h econ st
ructioncomp a nyistoma xi
mi z
ep rofi
t,fi
nd:
i
)Nu mb eroft r
ipsforea chv ehicl
e. (
2mks)
i
i)T heto t
alma ximump ro f
it. (
2mks)
20.Thep osit
iono ftwot own sPa n dQa reg iv
ent othen eare
stdegreeasP(45oN,200W)a n
dQ( 450N,1600E).
Find
a) Sho r
testdist
an ceb e
twe ent hetwot own sin:
-
i
) Kilome tres(t
ak eradiuso fthee ar
tha s6370k m) (
3mks)
i
i) Nau t
icalmiles.(Takeπ=22/ 7)a ndthee ar
th’sradius=6370k m. (
2mks)
b) Ash ipleavestownPa nds ai
lsdu eeastfor120h ourstoanothertownRa tanavera
gespeedo f27knots.
i
) Ca lcula
tet hed i
stanceb etwe e
nt hetwot own sinn auti
calmil
es. (
2mks)
i
i)F i
ndt hep osit
iono ftownR. (
3mks)
21.The2nda nd5tht ermso fa na r
it
h meti
cp rogressionare8a nd17r espec
ti
vely
. T
hesecond,10thand42ndte
rmsoftheAPformth
efir
st
th
re etermso fag eome tr
icp rogressi
on .Fi
nd

a)Thefi
rs
tter
ma ndthecommondi
ff
eren
ceoft
heAP.(
3mks
)
b)Thefi
rs
tthr
eeter
mso ftheGPa
ndthe5tht
ermoft
heG.
P (
4mk
s)
c)Thesumofthef
irs
t4t e
rmsoft
heG.P. (
3mk
s)
22.(
a)Co
mp l
et
ethetabl
ebelow (
2mk
s)

X 0 30 60 90 120 150 180 210 240 270 300 330 360


Si
nx 0 0.
5 0.
87 1 0.
5 0 -0.
87 -1 -0.
5 0
2si
n(X+30 1 2 1.
73 0 -2 -
1 1

b) Drawthegra
pho fy=si
nxa ndy=2si
n(x+30)o
nthesa
meax
isus
ingas
cal
eof1cmtorep
rese
nt300ont
hex
-ax
isa
nd4cmto
re
pres
ent1u n
itontheya xi
s. (
4mk
s)
c)F i
ndtheperi
odanda mpli
tudeofy=2si
n (
x+30) (
2mks)
d)F r
omt hegr
aphfindtheroot
sof 2si
n(x+30)–sinx=0 (
2mk
s)
23.I
nthefigu
rebelo
w, Eisthemid-po
intofAB,OD:DB=2:
3andFist
hepoi
ntofi
nt
ers
ect
ionofOEan
dAD.
A

E
F

O
D
B
a
) Gi
vent
hatOA=aa
ndOB=b
,ex
pre
ssi
nte
rmso
faa
ndb
.
(i
)OE (
1mk
)
i
i
) AD (
1mk
)

b) Gi
venfur
the
rtha
tAF=tADandOF=s OE,f
indt
heval
uesofsandt. (6mk
s)
c) Showtha
tO,FandEarecol
l
ine
ar (
2mks)
24.Th
efig
urebel
owshowsari
ghtpy
ramidPQRSV.PQRSisar
e c
tang
lewi
thPQ=8c
man
dQR=12cmVis25c
mfr
omt
hev
ert
ic
es
P,
Q,RandS.
FORMARKINGSCHEMESI NBOX0724351706 Pa
ge|107
Ma
the
mat
ic
s121/
1,2

F
indt
o2d
.p

a
) T
heh
eig
hto
fthepy
rami
d. (
3mk
s)
b
) T
hea
ngl
ebet
weenPVandt
hebasePQRS. (
2mk
s)
c
) T
hea
ngl
ebet
weenQVandSV. (
3mk
s)
d
) T
hea
ngl
ebet
weenth
epl
anesSRVandPQRS. (
3mk
s)

KASSUJ ETEXAMINATI
ONS
121/1  
  
  
  
  
  
Mathemati
csPape
r1
2½Ho ur
s
Ter
m2, .
SECTIONI50MARKS
0.
51x5700
1. Wi
th
outu
sin
gta
ble
s,e
val
uat
e gi
vi
ngt
hea
nswe
rins
tan
dar
dfo
rm. (
3mk
s)
6.
8x0.
0095

()
t
1 1
2. F
indt
hev
alu
eofti
nth
efo
ll
owi
nge
qua
ti
on81-1× = (
3mk
s)
27 243-1

3. Si
mpl
i
fyt
hee
xpr
ess
ion (
3mk
s)

×2-9y
2

2 2
2x-7×y +3y
4. Ev
aluateu
sin
gtab
leso
fre
cip
roc
alsa
ndc
ube
son
lye
xpr
ess
ingy
oura
nswe
rto4s
ign
if
ic
antf
i
gur
es

(
3mk
s)

2 2 3
5. Si
mpl
i
fy; o
f5 -2 (
3mk
s)
3 5 10

3 1 3
÷4 +1
5 2 5

6. F
indt
heequ
ati
ono
fal
i
newh
ichp
ass
est
hro
ught
hep
oin
t(2,
3)a
ndi
spe
rpe
ndi
cul
art
oy=3x-
1.Gi
vi
ngy
ourans
werint
hed
oub
le
i
nt
erc
ept
sfo
rm (
3mks)

7. Ge
org
ere
cei
ved10,
000Euro
sf r
omhisbr
oth
erwhos
taysi
nFr
anc
e.Hes
entt
ohi
ssi
st
erwh
ost
aysi
nJa
pan10,
000Ye
n.I
nad
dit
io
n
Ge
org
eboughtaca
rwort
hsh.200,
000.Exc
hang
era
tes:

Bu ying Se l
l
ing
1Euro 73.4226 73.52953
100Japa
n e
seyen 62.8011 62.8822
Howmu chwasle
ft
. (
3ma
rks
)
8. Wri
tedownfouri
nequ
ali
tieswh i
chf
ul
lyde s
cribet
heun
sha
dedr
egi
onRi
nth
efi
gur
eSb
elo
w (
4mk
s)

F
ORMARKI
NGSCHEMESI
NBOX0724351706 Pa
ge|108
Ma
the
mat
ic
s121/
1,2

9. Al
i
nePQ=12.
5cm.Byu
sin
gan
oth
erl
i
ne,
div
idePQi
nt
oni
nee
qua
lpa
rts
. (
3mk
s)

10.T
hes
umo
fth
ein
ter
iora
ngl
eso
fap
oly
go s19800.
ni Fin
dth
enu
mbe
rofs
ide
sth
epo
lyg
onh
as. (
3mk
s)

11.Th
ediagr
amb
elo
wisas
ket
cho
far
ic
efi
el
d(n
otd
rawnt
osc
ale
),u
sei
ttog
ene
rat
eas
urv
eyo
r’sf
i
eldboo
k.(Al
lme
asu
reme
ntsa
rei
n
met
res
) (
4mks)

12.Abo ywalkdir
ect
lyfro
mp ointQtowa r
dsthefoo
to faver
ti
calf
l
a gpost200ma wa y
.Afterconveyi
ngadist
anc
eo f140m, heob s
erv
es
th
ea ngl
eo fel
eva
tionofthetopofthefl
agposti
s75°.Ca lc
ula
tethea ng
leofdep r
ess
iono fpoi
ntQfromthet
o po ft
hefla
gp o
s t
.
(
3mks)
13.Twos i
mil
arb l
ock
sh avemassesof729ga n
d216gr espe
cti
vel
y.Ift
hes ur
fac
ea r
eao ft
hes mall
erblo
cki m2,
s300c c a
lcul
at
ethe
sur
faceareaoft
helargerbl
ock. (3mk s)
3
14.Usi
n gtr
apezoi
dalr
u l
ewith6o rd
inat
es,fi
ndtheareabounde
db yth
ec urvey=2x –5,thex -
axi
sandtheli
nesx=2a ndx=8.
(3mk s)
15.Kassi
mh asamo neyboxcontai
ning100mi xe
dshs5a ndshs10c oi
nswithat o
talval
ueo fshs600.Howma nyo feachtyp
eo fcoi
n
doestheboxcontai
n. (3mk s)
16.Thefig
ureb e
lowshowsar egul
artet
rahed
ronPQRSo f
edges4cm.
Drawitsnetandme as
urethelengt
ho ft
hestr
aightpat
hofPSthroughthemi dp
o i
ntTovert h
eedgeQR. (3mk s)

SECTIONII(50MARKS)
(Ans
we rANYFIVEque
sti
onsi
nthespa
cespr
ovided
)
2
17.
Completet
hetabl
ebe
lowfort
heequ
ati
ony=2x +2x
-8b
yfi
l
li
ngt
heb
lan
ksp
ace
. (
2mk
s)
x -5 -4 -3 -2 -1 0 1 2 3 4
y 32 -4 -
8 16
2
a)
.Ont heg r
aphp r
o v
ide
db el
o w, p
lott
h egraphofy =2x +2x-8. (
4mks)
(
u s
ethescal
e :y -
axi
s=1unit,x-axi
s=1unit
)
b) Useyourgraphtosolve:
2
) 2x
i +2x-8=0 (
1mk)
2
i
i) 2x +x+2=0 (
3mks)
18.a)abuslef
tKisumua t9.
30a mt owa r
dsNa i
robia
ta nav e
ragespe
edof81km/hr
.Ama t
atul
eftNai
rob
iforKi
sumuat10.10a.
mata
n
aver
agespeedof72km/ hr.Th edis
tancebetwee
nKi sumua ndNair
obii
s360km.Deter
mine
:
i
) T h
etimet a
kenbe f
orethetwov ehic
lesmet. (
3mks)
i
i)T h
ed i
stancebet
we e
nt wov ehi
cles40mi n
utesaft
e rme e
ti
ng. (
2mks)
i
ii) Acarle
ftKisumutowa r
d sNair
o bia
t9.50ama tana ve
ragespe
edof90km/hr
.Deter
minethet
imeth
ec a
rcaug
h tupwit
h
F
ORMARKI
NGSCHEMESI
NBOX0724351706 Pa
ge|109
Ma
the
mat
ic
s121/
1,2
t
hebus. (3mks
)
(
b)T
hefi
gur
ebelowsho
wsspe
edt
imeg
rap
hof
ajo
urn
ey.I
fto
tal
dis
tan
cet
rav
ell
edi
n80s
eco
ndsi
s920m.F
indt
hed
ist
ancet
rav
ell
ed
i
nth
efin
al40sec
onds.

(
2mk
s)
16m/
s

s
pee
d

0 15 t
ime(
sec
ond
s)
19.TheFi
gur
esh
owsaf
rus
tumo
far
igh
tpy
rami
dop
enc
ont
ai
nerf
ors
tor
ingwa
ter
.

Cal
cul
ate
:
a)T hehe
ightofth
ep y
ramidfr
omwh i
cht
hefru
stu
mwa sc
utf
rom. (
2mk
s)
b)T hes
urfa
cea r
eaofthefr
ust
um (
4mk
s)
c)T heca
pacit
yofthef
rustuminl
it
res (
4mk
s)
20.Thet
abl
eb el
owrepre
s e
ntmark
sinper
cen
tagesc
ore
dby50st
uden
tsi
nac
las
s

Mar
ks Fr
equ
enc
y
40-
44 6
45-
49 4
50-
54 7
55-
59 6
60-
64 12
65-
69 4
70-
74 5
79-
79 3
80-
84 3

a
) St
at
eth
emo
dal
cla
ss (
1mk
)

b
) Est
imat
e:
i
) Themeanma rk (
2mk
s)
i
i
) Themedi
a n (
3mk
s)
c
) Onthegri
dp ro
vid
eddr
awafre
que
ncypol
ygo
ntorep
res
entt
hea
bov
ein
for
mat
io
n. (
4mk
s)
21.Giv
enthatthecol
umnve
cto
rs

̂
() ( ) ()
-
4
3
̂
16
p= ,q= ,r= a
-4 ̂
9
6
ndt
ha
̂
3
ta=2p- q+ r
̂ ̂ 3̂
4
2

F
ORMARKI
NGSCHEMESI
NBOX0724351706 Pa
ge|110
Ma
the
mat
ic
s121/
1,2
Expressasac o l
umnv ect
ora ndh encec alcul
ateitsma gnit
ud e (4mks)
b) Gi venthatthemi dp oi
n t
sofPQi s(-
3, 1)a ndQ( 7,5),obtainth eco-
ordinat
eso fP (3mks)
c) At
o
r
ans
11
fAB a
la
ti
o
ndC.
()
1
2
nT .ma
1
pstri
ang l
eABCo ntotri
a ngl 11 1
eABC.Gi v
entheco -
ordinatesA(2,
3), B(2,
5)andC( 4,4)
.
(
Fi
n
3mk
dt
s)
hec o-
ordin
ate
s

22.Thec oor
dinateso ftri
angleABCa reA( 1,1),B(3, 1)andC( 1, 3).
(a
)Pl otthetr
iang l
eABC. (1mk)
(b
)T ri
an gl
eABCu ndergoesat r
an sl
ati
o nv ec()
to
2
r .Ob
2
tai
nt heima geofA’B’C’un derthetr
ans
(
for
2mk
ma
s
t
)
i
on,writethecoordi
n at
eso fA’
B’C.

(c
)A’ B’Cunde r
go esar efl
ect
iona longth elineX=0, ob t
ainthec oordi
natesandp l
o tonthegraphp oint
sA”B”C”u nderthetrans
formati
on.
(2mk s)
(d
)T hetriang
leA” B”C”u nder
go esane nlargemen tscalefact
o r-1,centr
eo r
igi
n.Ob t
ainthecoordinat
esoft
h eimag eA”’
B”’C”’
.
(2mks)
(e
)T hetriang
leA” ’B”’
C” ’undergoesar otationcentre(1, -
2)a ngle1200.Ob t
ainthec oordi
nate
so ftheimageA” B”C”.
(2mks)
(f
) Wh ichtr
ianglesared ire
ctl
yc on gr
ue nt? (1mk)
3 2
Ap art
icl
emo vesa l
on gas t
rai
g htli
n es uchthatitsd i
spl
a ceme ntsmetresfro
mag iv
enp oi
ntiss=t –5t +3t+4wh eretisti
mei n
s
e conds.
Fi
n d:
(
a)T hed i
splaceme ntofthep arti
clea tt=8. (2ma r
k s
)
(
b )T heve l
oc i
tyo ft
h epart
iclewh ent=10. (3ma r
k s
)
(
c)T heva l
ue sof twh e
nt hep art
icl
ei smo men tari
lyatrest. (3ma r
ks)
23.Api l
otint
e n
d stoflyfromAt oDt hro ughBa n dC, Bi s750k mf romAa ndo nab e
a r
ingofN50°E. Cisonab eari
ngofN40°Wf ro
m
Ba ndtheirdistan
c ea pa
rtis600k m.Di sonab eari
ngo fS85°Wf romCa nda tadist
anceo f1050km.
a) Us ingthes caleof1c mfor100k m,sh owt hefli
g htroute
. (4marks)
b)I f thepil
oto nitswa yb ac
kd ecidestof lydir
ectlyfromDt oA;
i
) Byu seo fcomp assbearingfindt hed i
rect
iono f Afr
o mD (1mk)
i
i)F indthed ist
an cefr
omDt oAi nk il
ome t
ers. (1mk)
d)T hep l
anef l
i
e sat500k m/ h.If i
tleavesDa t9.00a .
ma twh att i
med i
ditarri
vea tA. (2mks)
d)i ) Us ingy ourdia g
rami n(a)a bove,(i)fi
ndth ed i
stanceb e
twe enAa ndC (1mk)
i
i)F indthec omp assbearingo fAf r
o mp ointC (1mk)

KASSUJ ETEXAMINATI
ONS
121/2  
   
 
  
  
  
Mathemati
csPape
r2
2½Ho u
rs
Ter
m2, .
SECTIONI50MARKS
2
1. Sol
veforxintheequ
ati
o 2x
n- +x+36=0u
sin
gco
mpl
et
in
gth
esq
uar
eme
tho
d (
3mk
s)

2. Si
mpl
i
fyb
yra
ti
ona
li
zi
ngt
hed
eno
min
ato
rin l
eav
ingy
oura
nswe
rant
hef
orm wh
erea
,ba
ndca
rei
nt
ege
rs.

(
3mk
s)

3. F
indt
hev
alu
eofxi
nth
eeq
uat
io
n i
nth
era
ngel
i
ste
dbe
low.

0o≤x≤360o (3mks
)
4. Drawal
in
eAB=4cm,
Pisavar
ia
blepo
inti
nthepl
aneof
thep
a p
er,ab
oveAB,
suc
hth
ata
ngeABC=600a
l ndt
hear
eaoftr
ia
ngl
e
m2.
APB=6c Usi
ngar
ule
ran
dap ai
rofcompa
sse
sonlyf
i
ndtheloc
usofP. (3mar
ks)

5. Ex
pan
dan
dsi
mpl
i
fyt
heb
ino
mia
35
l2+ .He
x
nceu
set
h
()
ef
i
rstf
ourt
ermso
fyo
ure
xpa
nsi
ont
ofi
ndt
hev
alu
eof(
2.
(
5
5)
3mks)

6. T
heleng
thandbr
ead
tho
far
ect
ang
ula
rfl
oorg
ard
enwe
reme
asu
reda
ndf
oun
dtob
e4.
1ma
nd2.
2mres
pect
iv
ely
.Fi
ndt
hep
erc
ent
age
e
rro
rinit
sar
ea. (
3ma r
ks)
2 5
7. So
lvef
orxi
nth
eeq
uat
io
n3(
log
x) -l
ogx+2=0 (
3ma
rks
)

8. Ma
keSt
hes
ubj
ecto
fth
efo
rmu
laei
nth
efo
ll
owi
ng; (
3mk
s)

F
ORMARKI
NGSCHEMESI
NBOX0724351706 Pa
ge|111
Ma
the
mat
ic
s121/
1,2
9. Eachmemb ero facl
asstakeoneandonlyoneoft
hethre
eforei
gnlan
guages:Fr
ench
,GermanandSpani
sh.15pupi
l
stakeFr
ench
,9
ta
keGe r
ma na nd6takeSp an
ish.T
wop upi
l
sa r
echosenatra
ndom.Repre
sentthei
nf
ormati
oninatr
eedia
g r
amhencefi
ndthe
pro
b a
bil
it
ytha tbot
hp upi
lstak
ed i
f
fer
entsubj
ect
s. (3mks)
10.Theamo un
to fo i
lus
edb yashiptr
ave
ll
ingataunif
ormspeedvar
ie
sjoi
ntl
ywi t
hthedi
sta
nceandth
es q
uareoft
hespee
d.Ift
heshi
p
uses200barrelsofoi
lintrav
ell
in
g200mi l
esat36mileperhour
,det
erminehowma nybar
rel
sofoi
lar
eusedwhentheshi
ptr
ave
ls
360mi l
esat18mi le
sp er
. (3marks)

11.Usel
oga
rit
hmstoev
alua
te:
345.
3×0.00697
(
4mar
ks)
0.
534
12.Th
efi
gurebel
owshowsaci
rcl
eof
dia
met
erXY.
Cho
rdT
Zin
ter
sec
tsXYa
tC.
Ata
nge
ntt
oth
eci
rc
lea
tYme
etsT
Zpr
odu
ceda
tS.
T y

X Z
S

Gi
venth
atT
C=14c m,CY=4c
ma n
dYS=7.5cm.ca
lcu
lat
eth
eleng
tho
f:
a) CS (
1ma
rk)
b) XC (
2ma
rks
)

( )
2
2x x
13.De
termi
net
hev
alu
esofxfo
rwh
icht
hema
tri
x hasnoi
nve
rse (
3ma
rks
)
2 1
14.Mr.Ki
mbo,al
ocal
ret
ai
le
rbo
ughti
mp o
rte
dric
eats
h.56pe
rkil
ogra
ma ndl
oca
lri
ceatsh.
48p e
rkil
ogra
m. Hewant
stomixt
het
wo
t
ypesofr
ic
esoastomak
eapro
fi
tof20%.If
hesol
dthemi
xt
ureats
h.120p
er2kil
ogra
mp ac
ket,
fi
ndtherati
oth
etwoty
pesofr
ic
e
wasmix
ed. (3mks)

15.T
hes
ixt
hter
mo fage
omet
ri
cpro
g r
essi
oni
s16a n
dthet
hi
rdte
rmis2.De
ter
minethecommonrat
ioandt
hef
i
rstt
erm.
(3marks
)
2 2
16.T
hee
quati
onofaci
rc
lei
sgi
venbyx +4x+y -2y-4=0.De
ter
minet
hecen
trean
dr a
diuso
fth
ec i
rc
le. (3marks
)

SECTIONII
(Ans
we rANYFI
VEques
ti
onsinthes
pac
espro
vid
ed)
17.Theta
b l
ealo
ngs
ides
howsth
er a
teso
fta
xat
io
ninacert
ai
nye
ar.

I
nco
mei
nK£p .a Ra
te(
Sh.
PerK£)
1-3900 2
3901- 7800 3
7801- 11700 4
11701- 15600 5
15600- 19500 7
Ab ov
e19500 9

Int
hatyea
rMr .Kar
iuk
iatte
ache
ratKangaruHighSchoo
l wasear
nin
gabas
ics
ala
ryofKs
h.27000permo
nth.I
naddit
io
nhewa s
ent
it
le
dtootherta
xabl
eall
owance
stot
all
i
n gto11000p ermo n
thandap
ers
onal
rel
ie
fofKsh1056p
ermont
h.Heli
vesi
ntea
chers

qua
rte
rswhereheispayi
nganominal
rentof Ks
h.3500p ermont
h.
(
a)Cal
cul
atehowmu chinc
ometaxMr.Kar
iukiispai
dpermon t
h. (
4marks
)

(
b)Mr.Kar
iuk
i’
sot
herd
educt
io
nsp
ermo
nthwe
rec
o-o
per
ati
ves
oci
et
yco
ntr
ibu
ti
ono
fsh2500a
ndl
oanr
epa
yme
ntofs
h.3000,
cal
cul
at
e
hi
snetsal
ar
ypermont
h. (3marks
)

(
c)L
atert
hesameyearMr
.Kar
iuki
wastr
ansf
err
edt
oa n
othe
rsc
hoolwhe
reheear
nedhar
dsh
ipal
lo
wanceequi
val
entt
o30%ofhi
sbas
ic
s
ala
ry.Ontopoft
hede
duct
ionin(
b)abo
ve,hea
lsohadade
duc
ti
o no
fsh2700permont
htoKCT.Ca
lcul
at
etheper
cen
tag
echa
nge
i
nhisnets
alar
ypermo
nth (
3ma r
ks)
o o
18.Gi
vent
haty
=2x+c
os½x,c
omp
let
eth
eta
bleb
elo
wfo
rth
emi
ss
ingv
alu
eso
fy,
cor
rec
tto1d
eci
mal
pla
ce (
2mk
s)

Xo 0o 30o 60o 90o 120o 150o 180o 210o 240o 270o 300o 330o 360o
Y=si
n2x
+co
s½x 1.
0 1.
8 -0.
4 -0.
6 0.
4 -0.
7 -1.
0
F
ORMARKI
NGSCHEMESI
NBOX0724351706 Pa
ge|112
Ma
the
mat
ic
s121/
1,2
o o o o
(b
)Ont hegri
dp r
ovi
debelow,dr
awtheg
rapho
fy =s
in2x+c
os½xf o
r0x360 T
aket
hesc
ale1c
mf o
r30 o
nt hex
-ax
is.
2c mfo
r0.
5
uni
tsonthey–axi
s. (4mks
)
(c
)Us eth
eg r
aphtoest
imate
(i
)T hemini
mu mval
ueofy (1mk)
(i
i
)T heval
ueofXforwhich½s i
n2x+½c o
s½x 0.25 (3mks
)
13
19.Quadr
il
ate
ralOMNPi ssuc
hthatOM=mOP=pa ndPN= m. PNisp
rod
uce
dtoQsucht
hatPN:PQ=13:
15.Tisapointo
nMN
4
su
c ht
hatMN=3T N

Showtha
tO,Tan
dQa rec
oll
i
near
. (10ma
rks
)
20.Th
efig
urebe
lowre
pre
sent
sari
ghtpy
rami
donas
qua
reb
asePQRSo
fsi
de12c
m.Oi
sth
ece
ntr
eoft
heb
asea
ndVO=14cm.

Cal
cul
at
e;
(a
)T hel
engt
hofVPto1d
ecima
lpl
ace (
3ma
rks
)
(b
)T hean
glewhi
chVPmake
swit
htheba
sePQRS (
2ma
rks
)

(
c)T
hes
urf
acea
reao
fth
epy
rami
dto1d
eci
mal
pla
ce (
3ma
rks
)

(
d)T
hev
olu
meo
fth
epy
rami
d (
2ma
rks
)

21.T
het
abl
esh
owstheg
oods(P)pr
odu
cedbyac
ert
ai
nfa
cto
ryi
nti
me(
t)s
inc
e2010i
sbe
li
eve
dtoo
beyal
awo
fth
efo
r Atwh
mP=k erek
a
ndAar
econs
tan
tsa
ndtisti
meinyea
rs.

t 2010 2011 2012 2013 2014 2015 2016


P 5000 6080 7400 9010 10960 13330 16200

a) Ex
pres
sthelawP=kAtwherekandAa r
econs
tan
tsa
ndtist
imeinye
arsi
nthel
i
nearf
ormy=mx+candfi
ll
theta
bleapp
ropr
iat
el
y.
(3mks)
b) Pl
otasui
ta
blest
rai
ghtli
neg r
aph (4mks)
c) Us
ethegraphabov
etofindtheva
lue
so fAa
ndk. (3mks)
22.Asmal
lscal
efar
merwishestobuysomeshe
epandgoa
tsf
orre
ari
ng.Ashe
epcos
tssh
.400a
ndago
atc
o s
tssh.
300.Thef
armerhas

F
ORMARKI
NGSCHEMESI
NBOX0724351706 Pa
ge|113
Ma
the
mat
ic
s121/
1,2
enoughsp a
cefo ronly20a nima l
sandma yspendatmo s
tsh.6800. T
h enumbero fgoatsshoul
dnotexc
eedtwicethenumbero
fshe
ep.
a) Bylett
ingxa ndyt orepresentthenumb erof s
heepandg oat
sh ecanb uyresp
ecti
v el
y,writ
edownalli
nequa
li
tie
sf r
omtheabove
in
formati
o n
. (
4mks)
b) Represe
ntthein equali
ti
esont hegridp r
ovided
. (
4mks)
c)F romy ourgra
p h;fi
ndt hema ximumn u mberofanimal
sh ecanb uyatthelowestcost. (2mks)
23.(a
)Ana eropla
n efl
iesfromt ownA( 200N,600N)totownB(200N,200E)
. (
Taki
ngR=6400k m,π=3.142)
Ifi
tthenfli
esduenor
thfr
om
townBt otownC, 420kma wa y,calc
u l
atecorr
ecttotheneares
td egr
ee,thela
ti
tud eoftownC. (3marks)
(b)Calcul
at
et hesho r
testdist
anceink mb etweentownsP(600N,400W)a ndQ(600N,300E)g i
vi
ngyourans
werto2d ecimalp
lac
es
(
2ma r
ks)
(c)Thelocalti
mea ttownT (330N,150W)i s1045h our
s.Wh a
tisthelocalti
mea tQ(500N,300E)? (2ma rk
s)
24.UseT r
apezoi
dal ru
letof i
ndth eare
ab etweenthec ur
ve.
2
y=x +4x+4, thex-axisandt heordi
n at
esx=- 2andx=1( Us e7o r
d i
nat
es)
a)Co
mp le
tethetablecorrectto2d .p. (2ma rk
s)
x -
2 -
1. 5 -
1 -0.
5 0 0.5 1
y

b
)F i
ndthear
eaenc
los
edb
yth
ecurv
e,th
ex-
axi
s,l
i
nesx
=-2andx=1. (
3mark
s)
c
) Usei
nt
e g
rat
io
ntofi
ndt
hee
xac
tare
aandhen
cefi
ndt
hepe
rcen
tag
eer
rori
nyo
ura
ppr
oxi
mat
io
n. (5ma
rks)

F
ORMARKI
NGSCHEMESI
NBOX0724351706

F
ORMARKI
NGSCHEMESI
NBOX0724351706 Pa
ge|114

You might also like